Download as pdf or txt
Download as pdf or txt
You are on page 1of 106

TOPIC 1: LEPROSY DERMATOLOGY 1

19 Major topics from which 6 MCQs arise 2. Commonest nerve involved in leprosy in
1 Dermatophyte infection 38 A. Ulnar
2 Inchen planus 21 B. Median
3 Psoriasis 18 C. Radial
4 Pemphigus 18 D. Sciatic
5 Syphilis 16 A
6 Acne 16 .........(AIIMS PGMEE - DEC 1994)
7 Tuberculosis skin 13
Nerve involvement in leprosy
8 Alopecia 12
• Ulnar
9 Scabies 11
• Posterior auricular
10 Atopic dermatitis 11
• Peroneal
12 Contact dermatitis 10
• Posterior tibial
13 Lgv 8
14 Chanroid 6
15 Dermatitis herpetiformis 5
16 Pityriasis rosea 5
17 Tinea versicolor 5
18 Pityriasis alba 4
19 Tuberous sclerosis 4
226

19 T opics 226 MCQs


48 T opics 83 MCQs Claw hands of a patient with Hansen disease
Misclleneous topics < 5 MCQs
21 Acanthosis nigricans 3
22 DLE 3
23 Donovaniosis 3
24 Drug reaction 3
25 Epidermolysis bullosa 3
26 Erythema multiforme 3
27 Gonorrhea 3
28 HSV 3
29 Molluscum contagiosum 3 • Nerve involvement is more common in tuberculoid
30 Urticaria 3 maculoanaesthetic and neuritic types of leprosy.
31 Actinitic keratosis 2 • In tuberculoid type the involvement is usually restricted
32 Chloasma 2 to a single nerve which is usually a small cutaneous nerve
33 Darier’s disease 2 twig supplying cutaneous patch or large nerve trunk.
34 Exanthems childhood 2
35 Fordyce spot 2
36 Impetigo 2
37 melanoma 2

TOPIC 1: LEPROSY
1. Best method of treatment of ulnar Nerve abscess in a
case of leprosy is:
A. Incision and drainage
B. High dose of steroid
C. Thalidomide
D. High dose of clofazimine in lepromatous leprosy the nerve are infrequently involved
A but whenever involved the involvement tends to be
.........(AIIMS PGMEE - SEP 1996) bilateral and symmetrical
In tuberculoid type of leprosy the nerve involvement is
manifested as cutaneous loss of sensations and in
lepromatous type it is manifested as glove and stocking
type of peripheral neuropathy.
The cutaneou s sensation first lost i n leprosy is
temperature (but this does not happen in all cases)
• Incision & drainage of abscess, followed by Antileprotic t/ Motor nerve involvement is rare.
t is done for nerve abscess. Autonomic involvement of nerves can occur.
Lagophthalmos seventh nerve of a patient with Hansen
disease
Hansen disease. Corneal scar seventh nerve exposure

3. A patient presents with single hypopigmented


anaesthetic patch with satellite lesion on forearm.
The most likely diagnosis is

HELP LINE NO. 9391567707


TOPIC 1: LEPROSY DERMATOLOGY 2

A. Indeterminate leprosy • Indeterminate leprosy: This early form causes one to a


B. Tuberculoid leprosy few hypopigmented or, sometimes, erythematous
C. Neuritic leprosy macules.
D. Lepromatous leprosy – Sensory loss is unusual.
B – Approximately 75% of affected persons have lesions that
.........(AIIMS PGMEE - DEC 1994), .........(AIIMS PGMEE heal spontaneously.
NOV - 1999), AIPGMEE - 1999 – In some, the disease may persist in this indeterminate
form. In those with weak immunity, the disease progresses
Satellite lesions are a features of borderline tuberculoid leprosy. to one of the other forms.
In borderline tuberculoid leprosy ,the features are some
what similar to tuberculoid leprosy in that they have a • Ill-defined macular lesion of indeterminate leprosy, without
single or few lesions only. any clear loss of sensation
The difference is that the margin of the lesion is not sharply
demarcated from the surrounding normal skin (In
tuberculoid lesions the margins are sharply demarcated
i.e., they have a distinct border)

• Borderline Tuberculoid Leprosy (BT)


• Another example of borderline tuberculoid leprosy. There
are multiple annular lesions on the back .
• Some of the lesions have satellite lesions. Skin smear
showed a bacteriological index of 1+ • Tuberculoid leprosy:
– Skin lesions are few.
– One erythematous large plaque is usually present, with
well-defined borders that are elevated and that slope
down into an atrophic center.
– The lesions can become arciform or annular.
– They can be found on the face, limbs, or elsewhere,
but they spare intertriginous areas and the scalp .
– Lesions can be dry and scaly, hypohidrotic, and
hairless.
– Another presentation involves a large, asymmetric
hypopigmented macule. Both types of lesions are
anesthetic and involve alopecia
• In borderline tuberculoid the characteristic features is
that there are few small satellite lesions around the main
lesion.

Classification : The Ridley-Jopling classification is used


to differentiate types of leprosy and helps in determining
the prognosis.
Purely neuri tic leprosy (asymmetrica l peripher al
neuropathies with no evident skin lesions), with or without
tenosynovitis and symmetric polyarthritis, is also possible.
A general classification of disease is based on the
number of skin lesions present and the number of bacilli • Well defined annular lesion of Tuberculoid Leprosy with
found on tissue smears. loss of sensation
Paucibacillary disease (indeterminate leprosy and TT)
has fewer than 5 lesions and no bacilli on smear testing.
Five or more lesions with or without bacilli
(borderline leprosies and LL ) is considered multibacillary
disease.

• Ridley-Joplin Classification of leprosy


• Well defined lesion of Tuberculoid Leprosy with loss of
sensation

HELP LINE NO. 9391567707


TOPIC 1: LEPROSY DERMATOLOGY 3

• Spontaneous resolution can occur in a few years, leaving • A case of borderline leprosy
pigmentary disturbances or scars. Progression can also
occur, leading to borderline-type leprosy. In rare instances
in which a patient is untreated for many years, the
lepromatous type can develop.
• Neural involvement is common in persons with TT; it
leads to tender, thickened nerves with subsequent loss
of function. The great auricular nerve, common peroneal,
ulnar, and radial cutaneous and posterior tibial nerves are
often prominent. Nerve damage can happen early, • Borderline lepromatous leprosy:
resulting in wrist drop or foot drop. – Lesions are numerous and consist of macules, papules,
plaques, and nodules.
• Borderline tuberculoid leprosy: – Annular punched-out–appearing lesions that look like
– Lesions in this form are similar to those in the tuberculoid inverted saucers are common.
form, but they are smaller and more numerous. – Anesthesia is often absent.
– The nerves are less enlarged and alopecia is less in – As with the other forms of borderline leprosy, the disease
borderline tuberculoid leprosy than in other forms. may remain in this stage, it may improve, or it may regress.
– Disease can remain in this stage, it can convert back to
the tuberculoid form, or it can progress to LL. • A case of borderline lepromatous leprosy with plaques on
the back of arms and nodules on the back, distributed
quite symmetrically

• Borderline tuberculoid Leprosy with infiltrated edges that


are not so well defined and the satellite lesions

Lepromatous leprosy
narrow zone beneath the epidermis
free of organisms (Grenz zone)

• Borderline borderline leprosy: Cutaneous lesions consist


of numerous, red, irregularly shaped plaques that
are less well defined than those in the tuberculoid type.
Their distribution may mimic those of the lepromatous
type, but they are relatively asymmetric. Anesthesia is Lepromatous leprosy, skin, acid fast stain
only moderate.
• Regional adenopathy may be present.
• Disease may remain in this stage, it may improve, or it
may worsen.

A case of borderline leprosy

4. E. N.L. is seen in which type of leprosy


A. BT
B. LL
C. TT
D. Indeterminate
B
.........(AIIMS PGMEE - MAY - 1993)

HELP LINE NO. 9391567707


TOPIC 1: LEPROSY DERMATOLOGY 4

• Erythema nodosum leprosum is the chief manifestation • Lepromin test is a test for detecting cell mediated
of type II lepra reactions. immunity in a leprosy patients. It is widely used as an aid
• Type II lepra reaction occurs in lepromatous leprosy to classify the type of disease.
Important features of type II lepra reaction - • The test is strongly positive in the typical tuberculoid
1) 90% cases occur after the chemotherapy has started cases and the positivity getting weaker as one passes
(generally within 2 years) through the spectrum to lepromatous end.
2) Type II reaction is caused by sudden release of the • The lepromatous cases are lepromin negative i.e., cell
bacterial antigen which induces the formation of IgG mediated immunity is deficient.
antibodies and immune complexes. (It is type III
hypersensitivity reaction)

• Lepromin test is not a diagnostic test but it is of great


value in estimating the Prognosis of leprosy.
• Lepromin negative individual are at high risk of
developing progressive multibacillary leprosy and
The most common manifestation of ENL are crops of painful
those who are lepromin positive either escape the clinical
erythematous papules.
disease or develop paucibacillary disease

6. Average duration of treatment in multibaciilary leprosy


is
A. 1 year
B. 2 years
C. 3 years
D. Life long
B
.........(AIIMS PGMEE - NOV - 1993)
Hansen disease. Facial nodule and erythema nodosum
leprosum • Duration of t/t for multibacillary leprosy — 2 years
• Duration of t/t for paucibacillary leprosy — 6 months

7. Cell mediated immunity is maximally suppressed in


A. Borderline leprosy
B. Lepromatous leprosy
C. Tuberculoid leprosy
D. Indeterminate type
B
.........(AIIMS PGMEE - NOV - 1993)

• The type of leprosy that develops in an individual depends


upon the cell mediated immunity of the individual against
4) It is also often associated with- the leprosy bacilli.
• Neuritis or nerve palsies
• Orchitis
• Individuals with better cell mediated immunity develop a
• Iritis
localized form of the leprosy while those with very poor
• Arthritis
immunity develop a generalized form of leprosy
• Albuminuria due to glomerulonephritis (Lepromatous leprosy).
T/T of type II lepra reaction -
Mild — Antipyretics • Since most individuals have immunity varying between
Severe — Glucocorticoids
these two extremes and this immunity also varies from
Recurrent E.N.L. -> Thalidomide
time to time a large number of leprosy patients show a
wide spectrum of clinical manifestations which vary b/w
5. Lepromin test is strongly positive in
tuberculoid and lepromatous varieties. These patients are
A. BB
said to have dimorphous or borderline leprosy.
B. TT
C. BL
Type of leprosy with intact cell mediated immunity -
D. LL
Tuberculoid leprosy
B
Type of leprosy in which cell mediated immunity is maximally
.........(AIIMS PGMEE - MAY - 1993)
depressed - Lepromatous leprosy
.........(AIIMS PGMEE JUNE - 2000)

HELP LINE NO. 9391567707


TOPIC 1: LEPROSY DERMATOLOGY 5

Indeterminate leprosy • Erythematous plaque with xerosis

• Corticosteroid treatment is aimed at controlling acute


inflammation, relieving pain, and reversing nerve and eye
8. Treatment of Lepra 1 reaction is damage. With treatment, approximately 60-70% of the
A. Thalidomide patient’s nerve function is recovered.29 If neuritis is
B. Corticosteroids absent, NSAIDs may be helpful.
C. Increase in Dapsoe dose • MDT should be continued during type I reactions.
D. Clofazimine
B • Lepra type II reactions, or ENL , occur in approximately
.........(AIIMS PGMEE - MAY - 1994) 10% of patients with borderline lepromatous leprosy and
in 20% of patients with LL.
Reactional states occur in approximately one third of – These reactions are type III humoral (antibody-
patients and are acute inflammations of the disease. antigen) hyper sensitivities , with a systemic
They may be induced by MDT, physical or mental stress, inflammatory response to immune complex deposition.
trauma, pregnancy, or surgical procedures . – The most common presenting symptoms are crops of
A leprous reaction should be considered a medical painful erythematous nodules of the skin and
emergency and mandates immediate care. subcutaneous tissue.
These states can result in permanent neurologic sequelae, – Bullae, ulcers, and necrosis also may occur.30 Nerve
resulting in disability and deformity. damage is slower than in reversal reactions.
P atients at the highest risk a re those with Ulcers and papular lesions on the trunk
multibacillary leprosy and/or preexisting nerve
impairment.
• Lepra type I (reversal) reactions usually affect patients
with borderline disease.
– Reversal reactions are a shift toward the tuberculoid
pole after the start of therapy , and they are type IV
cell-mediated allergic hypersensitivities, indicating an
improvement in cell-mediated immunity.
– Puberty, pregnancy, and childbirth can also precipitate type
I reactions.
– These reactions usually result in skin erythema, with edema
and tenderness of peripheral nerves.
– New skin lesions are common, and the patient may
have an acute febrile illness.
– The peak time for type I reactions is during the first
2 months of therapy and for up to 12 months.

• multiple, well-defined, erythematous, tender plaques on


the chest, lower abdomen, buttock and right calf and a
few of these showed satellite lesions • Histopathology slide from ulcer margin (ZN Stain,
1000X) showing heavy infiltration of dermis by lepra
bacilli

• Caseating granulomas along the neuro-vascular


bundle with infiltration of nerve
• The reaction usually manifests after a few years of
therapy, and, although a single acute episode is possible,
relapses occur intermittently over several years.
• Associated fever, malaise, arthralgias, neuralgia, iridocyclitis,
dactylitis, orchitis, and proteinuria may be present.
– The use of clofazimine in MDT substantially reduces
the incidence of ENL to 5%. Clofazimine has also been
used to treat ENL.

HELP LINE NO. 9391567707


TOPIC 1: LEPROSY DERMATOLOGY 6

– Thalidomide is effective except in the case of neuritis prevalent state favours the diagnosis of Indeterminate
or iritis, in which case corticosteroids should be used. leprosy.
– Other treatment therapies reported to be effective include
colchicine, pentoxifylline, cycl osporine A, 12. A young boy presented with lesion over his right
intravenous immunoglobulin, and infliximab buttock which had peripheral scaling and central
– Lowering the dose of dapsone may decrease the scarring. The investigation of choice would be
severity of bullae and ulcers. A. Tzank Smear
B. KOH preparation
• Lucio phenomenon is a cutaneous n ecrotizing C. Biopsy
vasculitis that is sometimes designated a type II D. Saboraud’s agar
reaction. C
– It is common in Mexico and Central America and is .........(AIIMS PGMEE NOV - 2001)
characterized by erythematous, geometric, irregular-
shaped macules that rapidly progress to ulceration and Boy is having lupus vulgaris because the lesions have central
necrosis on acral areas or extremities of patients with diffuse scarring.
LL. Central Scarring is a feature of lupus vulgaris.
– Systemic symptoms such as hepatosplenomegaly, fever, Diagnosis of lupus vulgaris is confirmed by Biopsy.
arthritis, and nephritis are usually present.
– Thalidomide is ineffective in treating this type of Feature Disease Investigation
reaction; however, no consensus on treatment had been Central clearing Tinea corporis KOH Smear
determined. Central Scarring Lupus Vulgaris Biopsy
– Most patients with Lucio phenomenon have not received Central Crusting Leishmaniasis ID body demonstration
MDT or were treated irregularly; therefore, MDT is
recommended.
Lupus vulgaris
– Azathioprine or cyclophosphamide with corticosteroids
• Lupus Vulgaris à It is T.B. of skin
with or without plasmapheresis has also been used.

Hansen disease. Lucio phenomenon

The real challenge in managing leprosy is the Lupus vulgaris. A hyperkeratotic, crusted, granulomatous
treatment of reactional states. plaque (or plaques) on the face or elsewhere is
– If the course of MDT is not complete, continue taking characteristic. An apple-jelly color is seen on diascopy
those medications as directed. (pressure to remove the blood with a glass slide
– Systemic steroids are effective in reducing inflammation
and edema in reversal reactions; therefore, they are the
most helpful medications in preventing nerve damage.
– Prednisone at 40-80 mg/d should be given for 5-7 days
then tapered slowly over 3-6 months. This long course is
necessary to decrease the severity of disabilities and
deformities. One study recommended a low-dose (30 mg/
d) regimen for 20 weeks for controlling type I reactions.
– Clofazimine can also be used as a steroid-sparing agent
for reversal reactions, alone or with corticosteroids. • A young girl, 8 years old,
– Although the WHO does not support its use for ENL, • The loss of eyebrows is an indication of diffuse lepromatous
thalidomide is highly effective with ENL . It is leprosy
ineffective for the treatment of reversal reactions.

11. A boy comes from B ihar with anaesthetic,


hypopigmented, atrophic patch over face. The
diagnosis is:
A. Borderline leprosy
B. Pityriasis versicolor
C. Pityriasis alba
D. Indeterminate leprosy
D
.........(AIIMS PGMEE NOV - 1999), AIPGMEE – 1998,
AIIMS PGMEE - JUNE 1998

• Hypopigmented, anaesthetic, non scaly patches with


epidermal atrophy and the boy coming from leprosy

HELP LINE NO. 9391567707


TOPIC 1: LEPROSY DERMATOLOGY 7

• Most patients with leprosy can be cured with multi-drug • Photomicrograph to show skin with atrophic epidermis
therapy in just six month and a granuloma composed of well formed tubercle
composed of focal collection of epithelioid cells, Langhan’s,
giant cells and only few scattered lymphocytes, features
of BT leprosy

• man with active lepromatous leprosy • lepromin test site showing dense collections of
lymphocytes with focal areas of epithelioid cell
collection indicating a positive lepromin reaction

Lupus Vulgaris
Progressive form of cutaneous tuberculosis occurring
in a person with a moderate or high degree of immunity.
The characteristic lesion is a reddish-brown plaque,
• Deformity due to nerve damage with its consequent ulcers composed of nodules which show an ‘apple-jelly’ colour
and resorption of bone. Such deformities can be worsened when pressed with a glass spatula (diascopy). The disease
by careless use of the hands often affects the face leading to disfigurement due to
the destructive skin lesions.

• The individual lesions begin as reddish brown papules that


coalesce to form a plaque with a serpiginous border. The
plaque grows by peripheral extension, while healing at
one end.
• In western countries LV is common on the face, while in
India, lesions are more often encountered on extremities
and trunk.
• lepromatous leprosy patient with diffuse infiltration of the
face, ear lobes and chin
with few papulonodular lesions on the chin

• a sheet of macrophages in the dermis separated from


the flattened atrophic epidermis by a clear zone, a typical Lupus Vulgaris -Asymptomatic slowly growing purple
feature of lepromatous leprosy. scaly bumps on his right wrist for 3 years . A skin
biopsy demonstrated hyperplasia and hyperkeratosis of
the epidermis and caseating granulomas in the mid dermis.
Acid fast organisms were found on tissue sections. Central
scarring

HELP LINE NO. 9391567707


TOPIC 1: LEPROSY DERMATOLOGY 8

Lupus Vulgaris • Cutaneous leishmaniasis on dorsum of hand prior to


• asymptomatic ulcer on the back of his left elbow for 18 therapy.
months. It started as a small painless red papule which • Lesions were typically 0.6–3 cm diameter with central
slowly enlarged by peripheral extension and occasional crusting and ulceration with an erythematous indurated
ulceration with healing in the center margin
Lupus Vulgaris - Multiple, confluent,fib rotic, scaly,
hypopigmented plaques with central atrophy, peripheral
hyperpigmentation and crusts. 2 year history of slowly
progressive minimally pruritic confluent plaques involving
the perioral and centrofacial areas. The eruption was
preceded by or al mucosal plaques. Skin biopsy
demonstrated granulomatous inflammation, and cultures
grew Mycobacterium tuberculosis.

• Same lesion 3 weeks after completion of 20 days of sodium


Tinea corporis stibogluconate treatment.
• Large patches of erythema with central clearing

Tinea corporis -Central clearing

• Cutaneous leishmaniasis
• The presentation varies depending on the stage of disease.
Lesions are usually found in exposed areas. The skin lesion
begins as a nontender, firm, red papule several centimeters
in size at the site of the sandfly bite. In time, the lesion
widens with central ulceration, serous crusting, and
13. A 16 year old student reported for the evaluation
granuloma formation. The border often has a raised
of multiple hypopigmented macules on the trunk and
erythematous rim known as the volcano sign.
limbs. All of the following tests are useful in making
a diagnosis of leprosy, except
Cutaneous leishmaniasis
A. Sensation testing
This lesion was a 3-cm by 4-cm nontender ulceration that
B. Lepromin test
developed over the course of 6 months at the site of a
C. Slit smears
sandfly bite
D. Skin biopsy
B
.........(AIIMS PGMEE NOV - 2003)

Lepromin test is not a diagnostic test. It is a test used to


assess the immune status of an individual used for
detecting the cell mediated immunity of a patient. useful
in classifying the type of leprosy
• cutaneous leishmaniasis located on the right arm also useful in assessing the prognosis.
Drawbacks of lepromin test in its use as diagnostic
• 2-cm by 3-cm lesion was located at the exposed
test It may give
area where the sleeve ended. Note the satellite lesions.
1. positive result in non cases and
2. negative results in lepromatous and near lepromatous
cases.

Diagnosis of leprosy
• History & clinical examination includes: -
- Examination for sensory loss
- Examination of peripheral nerves
• Demonstration of Acid Fast Bacilli in skin smears

HELP LINE NO. 9391567707


TOPIC 1: LEPROSY DERMATOLOGY 9

Demonstration of causative organisms in skin smears Characteristics of skin lesions in lepromatous leprosy
prepared by the slit and scrape method and in nasal - Numerous, small, wide spread (involving, face, ear lobes)
swabs by the modified ziehl-nelsen method for staining - Bilaterally symmetrical
acid fast bacilli. Ill defined, macules which gets nodular
- Early involvement of face, nasal stuffiness, epistaxis
• Histology - A skin biopsy is indicated if the diagnosis is - Sensations are unimpaired. They do not manifest the
in doubt as in sensory loss which are so characteristic of tuberculoid
(i) Indeterminate leprosy leprosy.
(ii) Other granulomatous disorders like lupus vulgaris or
sarcoidosis cannot be ruled out.

Other auxiliary aids


- tests for sweat function
- histamine test
- nerve conduction studies.

14. A 45 year old male had multiple hypoaesthetic mildly


erythematous large plaques with elevated margins
on trunk and extremities. His ulnar and lateral
popliteal nerves on both sides were enlarged . The
most probable diagnosis is:
A. Lepromatous leprosy
B. Borderline leprosy Ear nodules of a patient with Hansen disease.
C. Borderline tuberculoid leprosy
D. Borderline lepromatous leprosy
D
.........(AIIMS PGMEE NOV - 2003)

Skin lesions show the characteristics of Tuberculoid


leprosy and the nerve involvement shows the features
of lepromatous leprosy.
This case neither belongs to lepromatous leprosy
(absence of characteristic lesions) nor to tuberculoid type
(presence of B/L nerve involvement with numerous Facial nodules of a patient with Hansen disease.
lesions)
So it should be some dimorphous case i.e. either it is a
case of Borderline Tuberculoid, Borderline lepromatous or
midborderline (BB) leprosy.

Hansen disease. Tuberculoid hand.

Lagophthalmos of a patient with Hansen disease.

Characteristics of skin lesions in Tuberculoid leprosy


• One or few lesions
• Well defined border
• Lesions are erythematous and indurated plaques
• The plaque mey be uniformly raised but sometimes it is
annular and has raised margins.
• Lesions are Hypoaesthetic Nerve involvement in the Tuberculoid type

- Nerve involvement is common in tuberculoid type.

Nerve involvement begins early and progresses rapidly.


- The involvement is usually restricted to a single nerve
which may either be small cutaneous nerve twig supplying
cutaneous patch or it may be a large nerve trunk such as
ulnar, median, radial, lateral popliteal, great
auricular etc.

HELP LINE NO. 9391567707


TOPIC 1: LEPROSY DERMATOLOGY 10

• Tuberculoid leg ulcer and biopsy site arm of a 15. Drug of choice for type II Lepra Reaction is:
patient with Hansen disease. A. Thalidomide
B. Steroids
C. Clofazamine
D. Rifampicin
B
.........(AIPGMEE - 2008)

16. An 8-year-old boy from Bihar presents with a 6


months history of an ill defined, hypopigmented
slightly atrophic macule on the face. The most likely
diagnosis is:
A. Pityriasis alba
Nerve involvement in the lepromatous type B. Indeterminate leprosy.
- Nerves are infrequently involved but whenever involved C. Morphoca
the involvement tends to be B/L and symmetrical. D. Calcium deficiency
B
Borderline leprosy is not same as Mid borderline (Borderline ..........(AIPGMEE - 2005)
Borderline or BB).
The term Borderline leprosy includes all the three - 17. A boy aged 8 years from Tamil Nadu presents with a
- Borderline Tuberculoid white,non anesthetic,nonscaly,hypopigmented
- Borderline (B.B.) macule on his face; likely diagnosis is:
- Borderline lepromatous A. Pityriasis alba
B. Pityriasis versicolor
The following features favours the diagnosis of Borderline C. Indeterminate leprosy
lepromatous leprosy. D. Pure neuritic leprosy
• Multiple lesions with wide distribution in the body C
(Trunk and extremities are involved) .........(AIPGMEE - 2001)
• B/L and symmetrical peripheral nerve enlargement.
Though the presence of hypoaesthetic lesions with
characteristic shape, erythematous plaques with
raised margin is not typical of lepromatous, yet it can be
seen in Borderline lepromatous.

Hansen disease skin biopsy Fite stain

Borderline lepromatous leprosy presenting as macular


erythematous generalized rash, anterior trunk.

Why indeterminate Leprosy,


- non scaly leision

If ‘scaly’ hypopigmented macule,


- diagnosis will be ‘Pityriasis alba’.

Hansen disease plasmoid (plastic) iridocyclitis Indeterminate Leprosy :


• One or more hypopigmented or hyperpigmented macular
a plaques may be seen
• Often an anaesthetic or praesthetic patch is the first
symptom noted by the patient.
However sensation is often preserved in early
lesions, particularly those on the face (as in this one)
• The lesion may clear spontaneously in a year or two, but
specific treatment is recommended.

HELP LINE NO. 9391567707


TOPIC 1: LEPROSY DERMATOLOGY 11

For mild reactions of both types, administration of


salicylates and other NSAIDs is adequate.

21. In leprosy nerves commonly involved are:


A. High ulnar, low median
B. High median, low ulnar
C. Triple nerve palsy
D. High radial, low median
A
.........(PGI - 1997 - Dec)
Pityriasis alba: White scaly, hypopigmented macule on the
cheeks and chin (face of a child) is: Pityriasis alba • In leprosy, although any peripheral nerve may be enlarged
Thisisusually solitary , harmless, non scarringusually non (including small digital & supraclavicular nerves), those most
infectious. commonly affected are the ulnar, posterior auricular,
peroneal & posterior tibial nerves, with associated
hyperesthesia & myopathy.

22. WHO regime for paucibacillary leprosy


A. 100 mg Dapsone daily + Rifampicin month (600 mg)
B. Dapsone daily + Rifampicin daily
C. Dapsone + Rifampcin + Clofazemine daily
D. Rifampcin + Clofazamine daily
A
.........(AIPGMEE - 1996)

Treatment of Leprosy
18. Treatment of Acute neuritis in Lepra I reaction is A/ Paucibacillary Multi bacillary
E
• Rifampcin 600 • Rifampcin 600
A. Dapsone
mg monthly and mg & clofazimine
B. Steroid
Dapsone 100 mg 300 mg monthly
C. Thalidomide
daily Dapsone 100 mg &
D. Incision and Drainage
clofazimine 50 mg
D
daily
.........(AIPGMEE - 1994)
• Treatment for 6 • Treatment of 2
In type I lepra reaction Thaladomide has no role. months follow up years, follow up for
Incision & drainage is indicated once nerve abscess for 2 years 5 yrs
develops following Acute nuritis.
23. Skin pigmentation & Icthyosis like side effects a seen
19. Multidrug therapy is given for in
A. Syphilis A. Rifampcin
B. leprosy B. Clofazimine
C. Herpetiformis C. Dapsone
D. Icthyosis Vulgaris D. Steroid
B B
.........(AIPGMEE - 1994) .........(AIPGMEE - 1996)

In T.B. & leprosy multiple drugs are used in combination for Clofazimine is a dye with leprostatic and anti inflammatory
treatment properties
Adverse effects include:
20. Drug of choice in type I Lepra reaction with severe • Reddish brown discolouration of Skin more on
neuritis exposed parts,
A. Thalidomide • Discolouration of hair and body secretion
B. Clofazimine • Dryness of Skin & itching (Icthyosis)
C. Dapsone
D. Systemic Corticosteroid Purple skin from clofazimine of a patient with Hansen disease
D
.........(AIPGMEE - 1995)

‘In Type I reaction where nerve paralysis is likely, corticosteroids


ought to be given.
In Type II reaction ,thalidomide is the treatment of choice in
moderate to severe reactions.’
DOC for Type I Lepra Reaction
= Corticosteroid
DOC for Type II Lepra Reaction = Thalidomide

HELP LINE NO. 9391567707


TOPIC 1: LEPROSY DERMATOLOGY 12

24. Skin biopsy in leprosy is characterized by: Tuberculoid AFB may be found at times
A. Pariappendegeal bacilli from the margin usually not
B. Pariappendegeal Iymphocytosis infective.
C. Perivascular Iymphocytosis Borderline May be found depending on
D. Any of the above nature of lesion where from it is
B taken Infective for all practical
.........(AIPGMEE - 1997) purposes.
Indeterminate ±
• Infiltration consists of epethiloid cells, Lymphocytes Neuritic Ordinarily Negative.
and Giant cells.
• Infiltrate is localized around blood vessel and other skin 26. All of the following lesions may be seen in leprosy
appendages except
A. Erythematousmacules
Hansen disease skin biopsy foam cells B. Vesicles
C. Hypopigemented patches
D. Flat & raised patches
B
.........(AIPGMEE - 1997)

All of the lesions mentioned except ‘vesicles’ may be seen in


Leprosy.

28. Leprosy does not involves:


Hansen disease skin biopsy tuberculoid. A. CNS
B. Testes
C. Skin
D. Cornea
A
.........(PGI - 1998 - Dec)

• Lepromatous leprosy involves the skin, peripheral nerves,


anterior eye, upper airways (down to the larynx), testes,
Hansen disease corneal scraping acid-fast bacilli. hands and feet, bones, muscles, internal organs etc.
• The vital organs and CNS are rarely affected in
leprosy, presumably, because the core temperature is
too high for growth of M. Leprae.

30. Thalidomide is the drug of choice for:


A. Lepra type I reaction
B. Lepra type II reaction
C. Nerve abscess
D. Nerve-excision
B
.........(AIPGMEE - 1999)
Hansen disease anterior chamber tap acid-fast bacilli globi.
type II Reaction is also known as Erythema Nodosum
Leprosum.
LEPRA type I Reaction is also known as Reversal Reaction.Q

LEPRA Reaction type II (ENL):


• occurs in Leprornotous Leprosy.
• Is a Jarish Herxheimer (arthus type of reaction)
i.e. type III hypersensitivity
• Thalidomide is the most effective drug for ENL. Q

25. Skin smear reports negative following pattern of However other drugs are more commonly used in the
leprosy management of ENL.
A. Indeterminate leprosy These include clofazamine, glucocorticoids, analgesics,
B. Neuritic type leprosy antipyretics and antibiotics.
C. Lepromatous leprosy Thalidomide is absolubly C/I in women of child bearing
D. Border line leprosy age.
B
.........(AIPGMEE - 1997) LEPRA Reaction type I (Reversal Reaction):Q
• Seen in TT.
Type Slit Smear • It is a manifestation of delayed H.S. to Mycobacterium
Lepromatous Teeming with AFB. Leprae antigens.
Infective

HELP LINE NO. 9391567707


TOPIC 2: DERMATOPHYTE INFECTION DERMATOLOGY 13

• Reversal reactions DO NOT RESPOND TO THALIDOMIDE nodosum lepro sum follows institu tion of
Mild episodes are managed with NSAIDs, while chemotherapy.
glucocorticoids are used in severe reactions.
Immunopathology of Erythema Nodosum Leprosum
31. Most common type of leprosy in India: • ENL is thought to be a consequence of immune complex
A. BT reaction.
B. TT Elevated levels of tumor necrosis factor (TNF) have
C. LL been demonstrated in erythema nodosum leprosy.
D. BL • It is thought that TNF plavs a central role in the
B pathobiology of this syndrome.
.........(PGI - JUNE 1997)
37. A 27-year-old patient was diagnosed to have
• TT (polar tuberculoid) leprosy is the most common form borderline leprosy and started on multibacillary multi-
of disease encountered in India &• Africa, but virtually drug therapy. Six weeks later, he developed pain in
absent in South East Asia, where BT type is frequent. the nerves and redness and swelling of the skin
lesions. The management of his illness should include
32. Erythema leprosum nodosum is seen in all of the following, EXCEPT
A. Lepromatous leprosy A. Stop anti-leprosy drugs
B. Borderline tuberculoid B. Systemic corticosteroids
C. Borderline lepromatous C. Rest to the limbs affected
D. Tuberculoid D. Analgesics
A and B A
.........(PGI - JUNE 2004) .........(AIIMS PGMEE - MAY 2004)

33. In Leprosy which of the following is not seen: TOPIC 2: DERMATOPHYTE INFECTION
A. Abnormal EMG
B. Voluntary muscle wasting 39. A 30 yr old female presents with history of itching
C. Decreased Proprioception under right breast. On examination annular ring
D. Decreased response to tactile sensation lesion was present under the breast. The diagnosis
C is:
.........(PGI - June -2000) A. Trichophyton rubrum
B. Candida albicans
• Leprosy affects mainly cultaneous nerves. It causes both C. Epidermophyton
sensory and motor changes. In polyneuritic leprosy sensory D. Microsporum
changes are more marked than motor changes. A
• Sensation of temperature and pain lost earlier than .........(AIIMS PGMEE MAY - 2002)
sensation of touch and pressure.
• Myopathy, muscle weakness and atrophy can occur • The findings of itchy annular ring lesions on the trunk
• Proprioception is carried by tract of Goll (Fasciculus gracillis point towards the diagnosis by Tinea corporis.
) and tract of Burdech (Fasciculus cuneatus ) , which is • Though Tinea corporis can be caused by all the three -
not involved in leprosy. Trichophyton Epidermophyton and Microsporum ;
• Tactile sensation is carried by peripheral nerves to spinal Trichophyton rubrum is the most common cause.
cord and then to brain. So, lesion of the nerves affect
the sensation.

34. The following drug is not used for the treatment of


type II lepra reaction:
A. Chloroquin
B. Thalidomide
C. Cyclosporine
D. Corticosteroids
C
.........(AIIMS PGMEE - MAY 2006)

35. The main cytokine, involved in erythema nodusum


leprosum (ENL) reaction is:
A. Interleukin-2
B. Interferon-gamma
C. Tumor necrosis factor-alpha
D. Macrophage colony stimulating factor 40. Wood’s lamp light is used in the diagnosis of:
C A. Tinea capitis
.........(AIIMS PGMEE - MAY 2006) B. Candida albicans
C. Histoplasma
Erythema nodosum leprosum or Type II lepra reaction D. Cryptococcos
occurs exclusively in patients near the lepromatous end A
of leprosy spectrum. In about 90% cases Erythema .........(AIIMS PGMEE MAY - 2002)

HELP LINE NO. 9391567707


TOPIC 2: DERMATOPHYTE INFECTION DERMATOLOGY 14

• Tinea capitis is ringworm infection of the scalp mainly due Kerion, a severely inflammatory, boggy, indurated, tumor-like
to M. Canis. mass that may occur in tinea capitis.
• Under Wood’s lamp light the infected areas give a light
green fluoresence.

• Wood’s lamp is used in the evaluation of certain other


skin disorders as listed below -
Condition Colour
• Tuberous Sclerosis Blue white
• Vitiligo Totally white
• Erythrasma Coral red or pink Characteristic features of Tinea cruris : -
• Pseudomonas inf Greenish white or pale blue The lesion shows the following characteristics.
• Porphyria cutanea tarda Pinkish red (Urine) • Well defined margin, raised borders
• Tinea versicolor Golden yellow, totally white • Multiple erythematous scales, can be papulovesicular
• Clear centre
Erythrasma. Corynebacterium minutissimum is the bacterium • Lesions are seen in the genital area and on the medial
responsible for this mild, chronic superficial infection. Red– aspect of upper thigh
brown patches, commonly of the groin or axilla , with • The lesions are usually bilateral but asymmetrical
distinct border and little to no scale occur. Symptoms
• Itching is the predominant feature
• Age group
It is a disease of usually young men

Corynebacterium minutissimum creates a water-soluble


porphyrin that fluoresces coral pink under Wood’s
light examination. This finding may be absent if the Causative agent
patient has bathed recently. • It is a dermatophytoses caused mainly by
Trichophyton rubrum
Epidermidophyton floccosum
In India, Trichophyton rubrum is the most common cause of
Tinea cruris and in western countries epidermidophy- ton
floccosum is the most common cause

Differential diagnosis
Candidiasis
• More common in females
41. A 22 years old male patient presents with a complaints • Characteristic clinical features are :
of severe itching and white scaly lesions in the groin Presence of several pustules
for past month. Which of the following is most likely Numerous small satellite lesions that have typical
to be the causative agent frayed peeling edge on rupture
A. Trichophyton rubrum Lesions do not have a distinct raised margin
B. Candida albicans
C. Candida glabrata • A moist, erosive, pruritic patch of the perianal skin
D. Malassezia furfur and perineum (with satellite pustule formation) is
A demonstrated in this woman with extensive
.........(AIIMS PGMEE NOV - 2003) candidiasis
The classical appearance of a Tinea cruris infection is red
scaly lesion

• Candidiasis. Discrete superficial pustules developed


within hours of birth on the hand of an otherwise
healthy ne wborn. A potassium hydroxide
preparation revealed spores and pseudomycelium,
and culture demonstrated the presence of Candida
albicans.

HELP LINE NO. 9391567707


TOPIC 2: DERMATOPHYTE INFECTION DERMATOLOGY 15

It is a case of Pityriasis versicolor


Clinical features of pitvriasis versicolor
• The lesions consist of large scaly hypopigmented
macules
• They usually start around the hair follicles and then merge
with each other to form large areas
• The hypopigmented lesion is covered with Branny scales.
• Upper trunk is the most common site of involvement,
(upper back, shoulders with upper chest, upper arms)
• Candidiasis. Dry, red, superficially scaly, pruritic
macules and patches on the penis represent • Coup’d onle or stroke of the nail is seen — This is
candidal balanitis loosening of scales with fingernail

Diagnosis of pitvriasis versicolor


Direct examination
(i) Direct examination of scales in 10% KOH shows
characteristic yeast and mycelia
(ii) Direct examination under wood’s lamp show pale yellow
fluorescence.
Culture
• Culture is rarely needed to establish the diagnosis
• It can be cultured in the following media
Sabraud’s dextrose agar with, chloramphenicol,
• Candidiasis. Erythema, maceration, and satellite
pustules in the axilla, accompanied by soreness and Acti-dione, Tween — 80, and Layered olive oil.
pruritus, result in a form of intertrigo. Produces yellowish colonies within 5-7 days

Treatment It
(a) Topical therapy
(i) 2.5% Selenium sulfide in detergent base
(ii) 20% Sodium hyposulfite solution of 50:50 propylene
glycol in water
(iii) Zinc pyrithione
(iv) Keratolytic creams containing 3-6% salicylic acid
Pityriasis versicoior (Malassezia furfur) (v) Retinoid acid creams
It is asymptomatic (there will be no itching) Not
inflammatory • Most individuals with tinea versicolor complain of
cosmetically disturbing, abnormal pigmentation.
• The involved skin regions are usually the trunk, the back,
the abdomen, and the proximal extremities . The
face, the scalp, and the genitalia are less commonly
involved.
• The color of each lesion varies from almost white to
reddish brown or fawn colored.
• A fine, dustlike scale covers the lesions.
42. A 24 year old man had multiple, small hypopigmented • Patients often complain that the involved skin lesions fail
macules on the upper chest and back for the last to tan in the summer.
three months. The macules were circular, arranged • Occasionally, a patient also complains of mild pruritus.
around follicles and many had coalesced to form large
sheets. The surface of the macules showed fine
scaling. He had similar lesions one year ago which
subsided with treatment. The most appropriate
investigation to confirm the diagnosis is;
A. Potassium hydroxide preparation of scales
B. Slit skin smear from discrete macules
C. Tzancktest
D. Skin biopsy of coalesced macules
A
.........(AIIMS PGMEE NOV - 2003)

HELP LINE NO. 9391567707


TOPIC 2: DERMATOPHYTE INFECTION DERMATOLOGY 16

Pityriasis versicolor: (a) pink, hyper- or hypopigmented


patches on the upper back of a young adult;

Wood’s light examination of the same patient. The areas Pityriasis versicolor: a KOH preparation of the scale shows
that previously appeared brown on a pale background short hyphae and budding cells (‘spaghetti and
now appear cream-colored due to fluorescence. meatballs’ or ‘grapes on a vine’

• Tinea versicolor is a benign skin disease that causes scaly


macules or papules on the skin. As the name implies (versi
Pityriasis versicolor in dark-skinned patients. These patients means sever al), the condition c an lead to
may develop either hypopigmented or hyperpigmented discoloration of the skin, with colors ranging from
areas white to red to brown. The condition is not considered
to be contagious because the causative fungal pathogen
is a normal inhabitant of the skin.

• Most cases of tinea versicolor occur in healthy individuals


Pityriasis versicolor in dark-skinned patients. These patients with no immunologic deficiencies. Nevertheless, several
may develop either hypopigmented or hyperpigmented factors predispose some people to develop this condition.
areas These factors include genetic predisposition; warm, humid
environments; immunosuppression; malnutrition; and
Cushing disease.

43. A 36-ye ar-old factory worke r developed


A fine scale when scraped is characteristic of pityriasis versicolor itchy,annular scaly plaques in both groins. Application
of a corticosteroid ointment led to temporary relief
but the pla ques continued to ex tend at the
periphery.The most likely diagnosis is:
A. Erythema annulare centrifugum.
B. Granuloma annulare.
C. Annular lichen planus.
D. Tinea cruris
D
..........(AIPGMEE - 2005)
Face uncommon
• Erythema annulare centrifugum.
• Pityriasis versicolor affecting the face (a relatively
• Erythema Annulare Centrifugum (EAC) is a descriptive
uncommon site) in a patient who also had seborrheic term for a skin lesion.[1] The lesions form consists of
dermatitis. Both are caused by the same yeast
redness (erythema) in a ring form (annulare) that
spreads from the center (centrifugum).

HELP LINE NO. 9391567707


TOPIC 2: DERMATOPHYTE INFECTION DERMATOLOGY 17

The diagnosis here is Tinea capitis (Dermatophytosis) or


fungal infection.

• It is case of tinea capitis



• Tinea capitis can present in several ways. It is the
inflammatory type of tinea capitis.
It presents with boggy, soft
• Occurring at any age these lesions appear as raised pink- mass with loose easily pluckable hairs (kerion formation)
red ring or bulls-eye marks. They range in size from 0.5–8 along with scales.
cm (0.20–3.1 in). The lesions sometimes increase size
and spread over time and may not be complete rings but Other 3 types of Tinea capitis
irregular shapes. Distribution is usually on the thighs • Non inflammatory - circular patches of partial alopecia
and legs bu t can also appear on the upper with broken off hairs.
extremities, areas not exposed to sunlight, trunk • Black dot variety - Areas of partial alopecia with hair
or face. Currently EAC is not known to be contagious, which has been broken off at the level of scalp giving the
but as many cases are incorrectly diagnosed as EAC, it is appearance of multiple black dots.
difficult to be certain • Favus - Yellowish crusts within the hair follicle with a mousy
odour.

Tinea capitis

Often no specific cause for the eruptions is found. However,


it is sometimes linked to underlying diseases and conditions
such as:

• Food (including blue cheese or tomatoes).


• Fungal, Bacterial and Viral infections such as sinusitis,
tuberculosis, candidiasis or tinea.
• Drugs including finast eride,[5] chloroquine,
hydroxychloroquine, oestrogen, penicillin and amitriptyline.
• CANCER-sensitivity to malignancy (especially the type
known as erythema gyratum perstans, in which
there are concentric and whirling rings).
• Primary biliary cirrhosis.
• Graves disease. Tinea capitis – Cicatrical alopecia
• Appendicitis.
• Lupus
• Pregnancy (EAC usually disappears/stops soon after
delivery of baby).
• Hormone

44. A man aged 30years presents with, alopecia, boggy


scalp swelling and easily pluckable hair. Next step in
establishing Diagnosis would be - • Kerion
A. KOH smear
B. Culture sensitivity
C. Biopsy
D. None of the above
A
.........(AIPGMEE - 2001)
AIIMS PGMEE - MAY – 1994
AIIMS PGMEE - DEC 1998
AIIMS PGMEE - SEP 1996

HELP LINE NO. 9391567707


TOPIC 2: DERMATOPHYTE INFECTION DERMATOLOGY 18

T.CAPITIS - Kerion Tinea capitis caused by Trichophyton verrucosum


lesions may be highly inflammatory which show a well-developed kerion of the posterior scalp , which is
swollen, edematous, oozing and crusting lesion in often mistaken for a bacterial abscess by those unfamiliar
the form of boggy inflammation of the scalp called with it.
“kerion”.
— mis-diagnosed and treated as an abscess of the scalp
Hair loss may be permanent causing cicatricial alopecia .

Favus –
- clinical picture of favus is characteristic
where solid crust is formed on the infected area later stages of a resolving kerion after treatment with oral
- spread to cover the whole scalp griseofulvin. The lesions may produce significant hair
- scalp has special mouse smell. loss but regrowth is the rule in almost all cases.
- condition is very chronic and may end with cicatricial alopecia

Tinea capitis, black dot. The fungus may invade the hair
(endothrix) and cause breakage at the base , resulting
in multiple black dots.
Tinea capitis occurs commonly in children as adults are
protected due to fungistatic properties of sebum.

Diagnosis of Tinea capitis - The diagnosis of Tinea capitis is


made by demonstrating the fungal hyphae in
scrapings obtained from the hair.
• The material obtained from the scrapings is mounted in
microscope slide in 10% KOH (KOH dissolves the
keratin and leberates the fungal hyphae.) black dots
• If the fungal hyphae are not visible in this way then the
material should be cultured on Saboraud’s media to
confirm the diagnosis.
• For other tinea infection of nails and skin the material is
obtained from skin and nails and procedure is
followed in the similar way.

45. Most common organism causing tinea capitis is:


A. Trichophyton tonsurans Tinea capitis, white dot. If the hair is white, multiple white
B. Microsporum dots may result from endothrix infection.
C. Epidermophyton
D. Candida albicans
A
……….(AIPGMEE - 2001)

Dermatophyte infection of the scalp as Tinia capitis has


returned in epidemic proportions. The predominant
organism in Trichophyton Tonsurans. • Moccasin-type” tinea pedis caused by E. floccosum

HELP LINE NO. 9391567707


TOPIC 2: DERMATOPHYTE INFECTION DERMATOLOGY 19

vesicular type tinea pedis caused by T. interdigitale

Tinea incognito” or steroid modified tinea pedis


caused by T. rubrum .

Tinea capitis and corporis caused by M. canis

Tinea is transmitted via the feet by desquamated skin


scales in substrates like carpet and matting.

Favus usually caused by T. schoenleinii, produces favus-like


crusts or scutula and corresponding hair loss.

Tinea pedis caused by T. rubrum. Sub-clinical infection (left)


showing mild maceration under the little toe and more severe
infection showing extensive maceration of all toe web spaces

46. Griesofulvin is given for the treatment of fungal infec


tion in finger nail dermatophytosis for:
Tinea of the groin showing typical erythematous lesions on A. 4 weeks
the inner thighs B. 6 weeks
C. 2 month
D. 3 month
D
.........(AIPGMEE - 2002)

Onychomvcosis (dermatophytosis/fungal infection of nails):


Tinea of the nails caused by T. rubrum
- most often is due to trichophyton rubrum
- must be treated systemically until all infected portions of
the nails grow out and are trimmed off Deformed nails
may grow more slowly than usual.
- Treatment should continue for at least 3 months.

• Tinea corporis caused by T. rubrum

Tinea Corporis
Central clearing

HELP LINE NO. 9391567707


TOPIC 2: DERMATOPHYTE INFECTION DERMATOLOGY 20

• Greisofulvin is DOC for dermatophvtosis 51. What does Not cause Tinea capitis
A. Epidermophyton floccosum
• Infected Part Duration of t/t
B. Microsporum
Skin 3 weeks C. Trichophyton violaceum
Palm & soles 4-6 weeks D. Trichophyton rubrum
Finger nails 4-6 months A
Toe 8-12 months .........(AIIMS PGMEE - DEC 1994)

47. Tinea ungium effects 52. About Dermatophytes, all are TRUE, EXCEPT
A. Nail fold A. Candidia albicans usually causes systemic infections
B. Nail plate B. Superficial layer of skin involved
C. Joints C. Microsporum does not involve nail
D. Inter digital space D. Epidermophyton does not involve hair
B A
.........(AIPGMEE - 1995) .........(AIIMS PGMEE - MAY - 1993)

Tinea Unguium involves both nailplate & nail bed but • Candida alb icans usually causes cutaneous
more common and more charecterstic involvement infections
is of nailplate. • Systemic infections with Candida albicans is seen only
Tinea unguium is characterized by asymmetrical nail in immuno compromised individual such as patients
involvement in the form of discolouration, thickening, taking steroids or suffering from diabetes.
subungual collection and partial separation of nail
plate’ Dermatophvtosis
• It is superficial fungal infection of the keratinised skin.
There are three genera of organisms involved
i) Trichophyton
ii) Epidermophyton
iii) Microsporum
• These organism live on the keratinous structures and can
infect epidermis, hair and nails.
48. An eleven year old boy is having tinea capitis on his
scalp. The most appropriate line of treatment is: • Epidermophyton does not involve the hairs and scalp.
A. Oral griseofulvin therapy
B. Topical griseofulvin therapy
C. Shaving of the scalp
D. Selenium sulphide shampoo
A
.........(AIPGMEE - 2003)

49. The test likely to help in diagnosis of a patient who


presents with an itchy annular plaque on the face is: Microsporum does not involve the nails.
A. Gram’s stain
B. Potassium hydroxide mount
C. Tissue smear
D. Wood’s lamp examination
B
.........(AIPGMEE - 2003)

50. TINEA Incognito Is seen with:


A. Steroid treatment
B. 1% BHC
C. 5% permethrin 53. A 8 year old child has localized, non-cicatricial alopecia
D. Antibiotics over scalp with itching and scales. The diagnosis is
A A. Tinea barba
.........(PGI - 1999 - Dec) B. Alopecia areata
C. Tinea capitis
• Steroid treatment can lead to masked infection particularly D. Lichen planus
Ring worm, resulting in extensive and unusual appearing C
ring worm (Tinea incognito). .........(AIIMS PGMEE - NOV - 1993)

55. Dermatophytes are:


A. Trichophyton rubrum.
B. Tinea versicuJaris.
C. Sporathrix.
D. Micosporidium
A
..........(PGI - DEC 2003)

HELP LINE NO. 9391567707


TOPIC 2: DERMATOPHYTE INFECTION DERMATOLOGY 21

Trichophyton rubrum Sporotrichosis causes subcutaneous inf, enough sepatate


hyphae, not demonstrable in pus or tissues. Diagnosis is
• The findings of itchy annular ring lesions on the trunk by culture.
point towards the diagnosis by Tinea corporis.
• Though Tinea corporis can be caused by all the three –
Trichophyton Epidermophyton and Microsporum ;

• Chromomycosis causes subcutaneous lesion, In KOH moist


skin scrapping, crusts, and pus, the brown pigmented
fungal structures present as hyphae.

Trichophyton rubrum is the most common cause of


Tinea corporis.

• Histoplasmosis is a dimorphic fungi, it causes systemic


infection. In tissues it remains in yeast phase.

56. Fungal filaments on skin scrapping or on section is


seen in:
A. Dermatophytosis
B. Chromomycosis
C. Mucormycosis
D. Histoplasmosis
E. Sporotrichosis
A and B 57. Treatment of tinea unguis:
.........(PGI - JUNE 2003) A. Flurticasone
B. Itraconazole
• Filamentous fungi are : C. Oleamine oil
- Dermatophytes D. Terbinafine
- Aspergillus E. Neomycin
- Zygomycetes B and D
- Penicillium .........(PGI - JUNE 2005)
• Superficial mycoses (surface infections and cutaneous inf”)
are : Treatment of Tinea unguium:
- Tinea versicolor, nigra Local Systemic
- Piedra - Miconazole, Econazole,
- Dermatophytes Clotrimazole - Griseofulvin
- Candida - Allylamines such as Terbinafine - Ketoconazole
• So, on skin scrappings or section, superficial mycoses - Itraconazole
are demonstrated by the presence of their filaments - Terbinafine.
(hyphae, mycelium).
58. Kerion is seen in:
• Mucormycosis causes subcutaneous infection, it has A. Pityriasis
septate hyphae. B. Dermatophytosis
C. Candidiasis
D. Trichomoniasis
B
.........(AIIMS PGMEE - DEC 1998)

• Dermatophvtosis is Superficial fungal infection of


the skin.
• Theses infections are restricted to the invasion of horny
structures, Stratum corneum, Nail & Hair.

HELP LINE NO. 9391567707


TOPIC 3: LICHEN PLANUS DERMATOLOGY 22

• Tinea is group name for highly contagious mycelial fungus. 62. A10 year old child has violaceous papule and
There are 3 distinct genera in this group. pterygium of Nails, the diagnosis is:
• 1. Epidermophyton (No hair involvement) A. Psoriasis
• 2. Microsoprium (No Nail involvements) B. Lichen Planus
• 3. Trichophyton C. Pemphigus
D. Pemphigoid
Tinea Capitis — is caused mainly by Trichophyton and B
microspora. The lesion produced are of 4 types. .........(AIIMS PGMEE JUNE - 1999), AIIMS PGMEE - JUNE
• 1. Scaly Type 1998
• 2. Kerion — usually caused.by Trichophyton. Initialy
boil like lesions later painless boggy swellings are produced. Lichen planus
• 3. Black dot Pruritic Papular disorder involving
• 4. Favus • Flexural surface
• Mucous membrane
• Genitalia
Clinical Features
• Age group 30-60 yrs
Lesions are
• Polygonal
• Plain topped
• Papules
• Purple

the involvement of Mucosa ,Thinning of nails - strongly


suggests the diagnosis of Lichen Planus

Dermatophyte infection with and without a Wood’s light


exam.

TOPIC 3: LICHEN PLANUS


The lesions are Symmetrical and usually effect
• Forearm and wrists
61. Generalized Lichen planus, treatment of choice is
• Lower legs & thigh
A. Topical salicyclic acid
• Genitalia
B. Ultraviolet rays
• Palms and soles
C. Systemic steroids
• Mucosal involvement (especially the buccal mucosa)
D. Erythromycin
occurs in 2/3 cases.
C
.........(AIIMS PGMEE - MAY - 1993)
Other characteristic features :
• Wickham’s Striae — Delicate white lines on surface of
T/T of lichen planus -
lesion
• Generalized lichen planus — Systemic corticosteroids
• Koebner phenomenon
If the patient canno|.be given corticosteroids then
• Nail Involvement — Occurs in 10% cases (pterygium,
Dapsone in given
Any chid)
Combination of dapsone and corticosteroid will give better
• Scalp lesions — Scarring Alopecia 1" occurs.
results than either drug used alone
(a) Lichen planus, typical flat-topped polygonal papules at a
• Lichen planus with few lesions -
typical site on the wrist. (b) Lichen planus lesions with
i) Local corticosteroid
the typical purple color.
or
ii) Lotion containing
Semiconfluent papules of lichen planus
a) Liquor picis carbonics
b) Tincture bensoin
c) Acetone
d) Chloroform

Single lesions in oral mucosa only - Corticosteroids tablets


which can be kept in mouth
• For Resistant single lesion - Intralesional injection of
corticosteroids‘

HELP LINE NO. 9391567707


TOPIC 3: LICHEN PLANUS DERMATOLOGY 23

Multiple eruptive small spots of lichen planus, with typical • Lichen planus is associated with Hyperpigmentation
purple color. Hyperpigmentation in Lichen planus is due to the
shedding of melanin from the damaged epidermis
into the dermis, whe re it is engulfed by
macrophages.

63. The characteristic Nail finding in lichen planus is:


A. Pitting
B. Pterygium
C. Beau’s lines
Lymphocytic infiltration is present. A band of lymphocytes
D. Hyperpigmentation of the Nails
and histocytes are present immediately subepidermally.
B
• The typical lesion of LP is a pruritic, purple, polygonal, plain
.........(AIIMS PGMEE MAY - 2001), AIPGMEE - 2006
topped, papular lesion which have a whitish lace work
pattern on its surface Wickham’s stria.
Lichen planus. Longitudinal ridging is seen on the fourth
• Sites involved: • Skin : front of the wrist alongwith skin of
fingernail. Atrophy with the beginning of pterygium is
limbs and trunk.
seen on the fifth fingernail
• Mucus membrane : often affected and lesion in mouth
are seen in upto 30%
A white lacework pattern on buccal mucosa is the most
frequently observed pattern.
• Nails : are affected in 5 to 10% of patients.

Changes include:
- thinning, pterygium formation and complete loss of nail
plate. Remember:
Onycholysis from psoriasis • Involved of scalp may lead to hair loss
Psoriasis nails • Lichen planus has been found to be associated with viral
Psoriasis of the nailbed may manifest itself as onycholysis, hepatitis due to Heptatitis C.
oil spots, and nailbed thickening.
Psoriasis of the matrix results in nail surface pits. Wickham’s Striae------> Delicate white lines on surface of lesion
• Whickham’s striae on the surface of a lesion of lichen
planus. These can be accentuated by the application of
oil (or even water) to the lesion.

64. A pt. presents with scarring Alopecia, thinned nails,


• The volar aspect of the wrist is a typical site for lichen
hypopigmented macular lesions over trunk and oral
planus, and is an area where Koebner reactions are
mucosa. The diagnosis is:
often seen.
A. Psoriasis
B. Leprosy
C. Lichenplanus
D. Pemphigus
C
.........(AIIMS PGMEE NOV - 2001)

66. Regarding Lichen Planus all are true, except:


A. Hypopigmentation in residual disease
B. Lymphatic infiltration in supraderma! Layer
C. Ithcy polygonal, purple Papule
D. Skin, hair and oral mucosa commonly invovled
A
.........(AIPGMEE - 2000)

HELP LINE NO. 9391567707


TOPIC 3: LICHEN PLANUS DERMATOLOGY 24

The Koebner reaction in lichen planus due to a scratch on • LP of the nailfold is important, as the inflammatory process
the arm. Lesions take about a week to appear, about may lead to scarring and adherence of the nailfold
the time the wound would normally take to heal. to the dorsal nailplate , a process known as
• Koebner phenomenon pterygium. This is irreversible, and warrants aggressive
therapy to preserve the normal nail.

Multiple small scr atches on the dorsum of the hand,


demonstrating the Koebner reaction in lichen planus.

• Scalp involvement by lichen planus is relatively uncommon,


but may occur in isolation. It may be primarily around the
follicles and hyperkeratotic, producing a nutmeg-grater-
like feel to the scalp. This is known as lichen planopilaris.
The clinical differential diagnosis is usually from
discoid lupus erythematosus . Both of these are
Pigmented Koebner reaction of lichen planus due to an scarring disorders at this site, and carry a risk of permanent
abrasion incurred alopecia, so prompt diagnosis and aggressive therapy are
required.

Postinflammatory pigmentation is often prominent in lichen Erosive genital lichen planus may occur in either sex, in
planus this case only becoming apparent following circumcision
for treatment of chronic balanitis.

• Generalized lichen planus, with abrupt ‘exanthem-like’


onset. This pattern is associated with hepatitis infection

LP may cause nailplate damage


• Vulval lichen planus, in this instance with macerated keratin
producing a white color but still with a purplish background
component.

LP-periungual inflammation

HELP LINE NO. 9391567707


TOPIC 3: LICHEN PLANUS DERMATOLOGY 25

Perianal lichen planus may resemble intertrigo, seborrheic


dermatitis, or psoriasis , due to the occluded skin at
this site.

• Muercke’s lines
– Transverse, arciform, white lines which are related to
changes in the nail bed (and thus do not grow out with
67. Which of the following is wrong statements: the nail) are characteristic.
A. Koilonychia in Vit B12 deficiency – Edema of the underlying connective tissue seems to be
B. Oncholysis in Psoriasis the cause. They are most commonly associated with
C. Mees lines in Arsenic poisoning hypoalbuminemia (e.g. nephrotic syndrome) but may occur
D. Pterygium of nails in Lichen Planus after trauma.
A – These changes reverse when the edema resolves or the
.........(AIPGMEE - 2000) albumin is restored to normal.

• Koilony chia is a feature of Iron deficiency Anemia & not


megaloblastic anemia

• Oncholysis is not a disease but a physical sign in which the


terminal nail plate separates from the underlying
nail bed.
It is observed in Psoriasis, Thyrotoxicosis, Chronic
Paronychia, Yellow nail syndrome.

Psoriasis charcteristically causes “thimble pitting” of the • Yellow nail syndrome


finger nails. – In yellow nail syndrome (YNS), the nails become thickened,
• Mees lines are transverse white striae on the finger nails yellow, and grow very slowly. This condition is associated
seen in Arsenic poisoning. with pulmonary disorders (e.g. pleural effusion,
• Pterygium of nails can be seen in Lichen Planus. chronic bronchitis, and bronchiectasis) and dysplasia
of the lymphatics.
Darrier’s disease. This shows typical longitudinal white streaks – One study suggested that the underlying cause of YNS is
and notched splitting of the free margin of the nail. not primarily a lymphatic abnormality, and that the
lymphatic im pairment appears secon dary and is
predominantly functional and not structural in nature

Terry’s nails
The distal 1–2 mm rim of the nail (which is still in contact Yellow nail syndrome. The patient also had chronic
with the nailbed) is pink while the rest of the nail is white. lymphedema of the face and the nails were dark, with
This appears to be a non-specific sign longitudinal and transverse overcurvature.

Nail–patella syndrome
The lunula is triangular and the patellae are absent or
Half and half nails hypoplastic in this syndrome with autosomal dominant
The proximal half of the nail is white, while the distal half inheritance. Hyperpigmentation of the pupillary margin
is red or pink in half and half nails – a marker of renal of the iris (Lester iris) may also be seen, as well
disease as glomerulonephritis with renal failure

HELP LINE NO. 9391567707


TOPIC 3: LICHEN PLANUS DERMATOLOGY 26

Muehrcke’s nail
---> Severe hypoalbuminemia as in Nephrotic
syndrome

Nail involvement is not seen in DLE.

Blue nails
• Wilsons disease
• hemochromatosis
the radiograph of the knees shows absence of the left • Antimalarial drug
patella and a hypoplastic right patella . Brown nail
• Addison’s disease
• Arsenic poisoning

Leuconychia - Liver disease


• Leukonychia. These marks are common, nonuniform,
white lines found in differing spots on one or more nails.
They should not be confused with Mees’ and Muehrcke’s
lines, which parallel the lunula across the entire nail bed
and occur in more than one nail.

Disorders of Nail
– Beau’s Line
• Any severe systemic illness.
– Oncholysis • Psoriasis
• Fungal infection
• Thyrotoxicosis
• Tetracyline
• Trauma
Nail Pitting • Mees’ lines
• Psoriasis Arsenic poisoning, Hodgkin’s disease, CHF, leprosy, malaria,
• Alopecia Areata chemotherapy, carbon monoxide poisoning, other systemic
• Lichen Planus insults
• Eczema

68. Wickiham’s stria are seen in:


• Ridging • Lichen Planus A. Lichen niditus
B. Lichenoid eruption
C. Lichen striates
D. Lichen planus
D
.........(AIPGMEE - 2002)

Wicham’s striae- This is a network of gray lines on papules of


• Koilonychia lichen planus.
• • Iron deficiency Anaemia Lichen planus
• • Lichen planus intensely pruritic, scaly, violaceous, flat-topped papules
• • Repeated exposure to detergents on the wrists

• Lichen planus
Hypertrophic plaques
Koebners phenomenon

HELP LINE NO. 9391567707


TOPIC 3: LICHEN PLANUS DERMATOLOGY 27

pseudopelade’.

nail plate tends to thin


become grooved and ridged.
Lichen planus nail may darken, thicken up or lift off the nail bed (onycholysis).
Guttate plaques cuticle is destroyed and forms a scar (pterygium).
nails may shed, stop growing altogether and rarely, completely
disappear.

Lichen planus
Wickham’s striae

69. Mouth Lesion are seen in


A. Psoriasis
B. Lichen Planus
C. Basal Cell CA
D. Tcthyosis Vulgaris

.........(AIPGMEE - 1994)

Plaquelike oral lichen planus on the buccal mucosa on the left


Oral lichen planus side

Lichen planopilaris
• Reticular oral lichen planus on the buccal mucosa
Follicular lichen planus
on the left side
also known as lichen planopilaris
results in
tiny red spiny papules around a cluster of hairs
Permanently bald patches may develop.
Sometimes no follicular scaling or inflammation is present
=
but bald areas of scarring slowly appear
=
often looking rather like footprints in the
snow • lcerative oral lichen planus on the dorsum of the
= tongue.
known as ‘pseudopelade’.

HELP LINE NO. 9391567707


TOPIC 3: LICHEN PLANUS DERMATOLOGY 28

70. Which of the following are pruritlc lesions: The morphological features of LP are :
A. Lichen planus • Lymphocytic infiltration along the dermoepidermal junction
B. Sun burns - Lymphocytes are intimately a/w basal keratinocytes with
C. Pemphigoid basal cell degeneration.
D. Psoriasis - Anucleate, necrot ic basal cells may b ecome
E. SLE incorporated into the inflamed papillary dermis ,
A,B,C and D where they are referred to as colloid or civatte bodies.
.........(PGI - 2000 - Dec) - Chronic cases of LP shows epidermal hyperplasia &
thickening of the granular cell layer & stratum corneum.
— Lichen planus, psoriasis cause itching.
— SLE : skin lesions are NOT itchy or painful
— Sunburns : Burning, itching, blistesing, redness occur
— Pruritus may be non-existent or severe in pemphigoid.

71. Itchy polygonal violaceous papularlesion is seen in:


A. Lichen planus
B. Psoriasis
C. Pitriasis rosea
D. Pitriasis rubrapilaris
A
.........(AIPGMEE - 1998)
74. Basal cell degeneration seen in;
72. Kobner’s phenomenon seen in A. Lichen planus
A. Lichen plannus B. Psoriasis
B. Warts C. Pemphigus
C. Behcet syndrome A
D. Psoriasis .........(PGI - DEC 2002)
E. Vitiligo
A,D and E Reticular oral lichen planus: typical lesions on the buccal mucosa
.........(PGI - DEC 2004)

• Kobner response is the development of lesions in


previously normal skin that has been subjected
to trauma
.Diseases showing this response are :
- Carcinomas
- Darier’s disease
- Erythema multiforme
- Hailey-Hailey disease
- Kaposi’s sarcoma Reticulat oral lichen planus: a more intense lesion on the buccal
- Leukemia mucosa
- Lichen planus & sclerosus
- Necrobiosis liopoidica
- Peroforating collagenosis & folliculitis
- Psoriasis
- Vasculitis
- Vitiligo
- Xanthoma

• Kobner response is best not used to dermatosis that occurs


• Histopatholo gy of reticular or al li chen planus:
due to spread of an infective agent like molluscum
parakeratosis, acanthosis, vacuolar degeneration of
contagiousum or warts; for this phenomenon, the term
basal cells, typical lichenoid lymphocytic infiltrate
pseudo-kobner is used.

73. True about lichen plannus:


A. Basal cell degeneration
B. Colloid bodies seen
C. Epidermal hyperplasia in chronic cases
D. Wickham striae seen
E. Autoimmune disease
All
.........(PGI - DEC 2004)

• The exact pathogenesis of LP is not known. Cell mediated


immune reactions secondary to release of antigens at the
levels of the basal cell layer & the dermoepidermal
junction might be the causative factor.

HELP LINE NO. 9391567707


TOPIC 3: LICHEN PLANUS DERMATOLOGY 29

75. Most characteristic feature of lichen planus is: Lichen planus. There is basal layer vacuolization. The epidermis
A. Thinking of nail plate is most common is slightly acanthotic, and there is a tendency to a saw-
B. Non scarring alopecia tooth pattern of the lower epidermis.
C. Violaceous lesions on skin and mucous membrane
D. Wickham striae
C
.........(PGI - June -1998)

• Most characteristic of lichen planus is


— Violaceous, polyhedral papules with shiny, flat tops in skin
and mucous membrane.
— In lichen planus, there is SCARRING ALOPECIA
Nail involvement is seen in minority of cases only.

76. The most characteristic finding in lichen pianus is:


A. Civatie bodies
B. Basal cell degeneration
C. Thinning of nail plate Where as violaceous lesion other than lichen planus also found
D. Violaceous lesions in lupus perinio, lymphoma cutis, cutaneous lupus
B etc.
.........(PGI - June -1999)
• Thinning of nail is seen in
• The typical finding in lichen planus : — Lichen planus
— Band of lymphocytes and histiocytes in the subepidermal — Trauma
region (Lichenoid band) — Vascular or neural disorder
— Epidermal thickening with hypergranulosis. — Systemic disease
— ‘Sawtooth’ pattern of erosion, cytoid bodies and dense
lymphocytic infiltrations of basal epidermal region. Longitudinal ridging of Nail — in Lichen planus
The basal keratinocytes shows degeneration and necrosis.

• Horizontal ridging of Nail — Chronic ECZEMA

Lichen planus . This lesions shows hyp erker atisis,


hypergranulosis, some acanthosis, and a bandlike infiltrate
that fills the papillary dermis and obscures the deramal
epidermal junction.

• Thimble pitting of Nail — Psoriasis “

Lichen planus. The most notable feature is a bandlike infiltrate


of lymphocytes that obscures the dermal epidermal
junction.

HELP LINE NO. 9391567707


TOPIC 4: PSORIASIS DERMATOLOGY 30

• Civatte bodies - a nucleated, necrotic basal cell may


become incorporated into the inflammed papillary dermis
called civatte bodies. It is important feature of Lichen
planus.

77. Max. Joseph’s space is a histopathological feature


of:
A. Psoriasis vulgaris
B. Lichen planus
Other important features of psoriasis are :
C. Pityriasis rosea
Auspitz sign : multiple minute bleeding points when
D. Parapsoriasis
scale is lifted from plaques.
B
.........(AIIMS PGMEE - MAY 2006)

Lichen planus is characterized by degeneration of basal


keratinocytes and disruption of epithelial basement
membrane.
As a result of this the epithelial connective tissue interface
weakens which may lead to histo logical cleft
formation.
• These cleft are called Max Joseph Space.
Spongioform pustules of Kagoj : Neutrophils form small
Other histological characteristic of lichen planus.
aggregates within slightly spongiotic foci of superficial
Saw toothing - Redefinition of the normal smoothly epidermis.
undulating configuration of the dermoepidermal interface — Stratum granulosum is thinned or absent.
to a more angulated zig-zag contour due to infiltration
of the lymphocytes along the dermoepidermal junction.

Civattee bodies - These are anucleate necrotic basal cells


which may become incorporated into the inflamed
papillary dermis.
• Remember that the basic pathologic process in lichen
planus is immunological attack on the basal layer while
the presence of inflammatory cells and the other epidermal
alterations are believed to be secondary events.

TOPIC 4: PSORIASIS

79. Munro micro abscess is seen in:


A. Dermal tissue
B. Stratum basale
C. Stratum corneum
D. Stratum malpighi
C
.........(PGI - 1999 - Dec)

• Munro-micro abscesses are minute micro-abscesses formed


by Neutrophils aggreegation within the parakeratotic
STRATUM CORNEUM found in psoriasis.

Munro Microabscesses
• Munro microabscesses are composed of degenerated
polymorphonuclear leukocytes (PMN’s) in the horny
layer (stratum corneum) and are seen in psoriasis and
seborrheic dermatitis.
Example of Munro microabscesses in Psoriasis.

HELP LINE NO. 9391567707


TOPIC 4: PSORIASIS DERMATOLOGY 31

81. Treatment of pustular psoriasis is:


A. Thalidomide
B. Retinoids
C. Hydroxyurea
D. Metholtrexate
B
.........(AIIMS PGMEE MAY - 2002)

Both methotrexate and retinoids are used in t/t of pustular


psoriasis.
Pautrier Micro-abscess formed in epidermis isseen in Mycosis Methotrexate Retinoid
fungoides. • It is mainly suitable for those who • Effective against all types of
would otherwise be disabled by the psoriasis but more for
disease and particularly suitable for erythrodermic and pustular
some elderly patients with severe psoriasis
psoriasis

• Most useful when used in


combination with ultraviolet light
• Serious side effects • Its major drawback as

• Hepato toxicity • T eratogenic

Pautrier Microabscess • Suppression of bone marrow • Hepatdtoxicity

• Pautrier microabscesses are a collection of 3 or more atypical • Hyperlipidemia


mononuclear cells within the epidermis in mycosis • Bone toxicity
fungoides.
Example of Pautrier microabscesses in Mycosis Fungoides. Isoretionoid.
• Pustular and erythrodermic psoriasis usually respond more
rapidly than commmon plaque psoriasis at does
of10-25 mg/ day excellent contro l of these
conditions usually can be achieved with isoretinoin ”
• The t/t of choice for pustular psoriasis — Retinoids.

82. A patient presents with erythematous scaly lesions


on extensor aspect of elbows and knee. The clinical
diagnosis is got by:
Microabscess at tips of dermal papilla —— Dermatitis A. Auspitz sign
herpetiformis B. KOH smear
C. Tzanck smear
D. Skin biopsy
A
.........(AIIMS PGMEE MAY - 2002)

• Erythematous scaly lesions on extensor aspect of elbow


and knee favours the diagnosis of Psoriasis.
• The clinical diagnosis of psoriasis can be made by
demonstrating Auspitz sign.
• It is a characteristic finding of psoriasis in which removal
of scale leads to pinpoint bleeding.
80. Photochemotherapy is useful in: Psoriasis - silver-white scales
A. Tinea capitis
B. Psoriasis
C. Lichen planus
D. Icthyosis vulgaris
B
.........(AIIMS PGMEE - Dec - 1995)

• In psoriasis PUVA (Photochemotherapy with ultraviolet


radiation of the A type) is used. Silvery scales

Other skin disorder helped by ultravoilet Radiation :


• Atopic Dermatitis —Few patients benefit
• ACNE—Only occasionally used
• Mycosis Fungoides —Only used as initial t/t
• Pityriasis lichenoides chronica –
Good treatment for rare disease.

HELP LINE NO. 9391567707


TOPIC 4: PSORIASIS DERMATOLOGY 32

• Auspitz sign Guttate psoriasis is characterized by


small red dots (or drops) of psoriasis.
often appears on the
trunk, arms and legs
lesions may have some scale
Guttate psoriasis frequently appears suddenly following a
streptococcal infection or
viral upper respiratory infections
events that can precipitate an attack of guttate psoriasis
tonsillitis
a cold, chicken pox
immunizations
physical trauma
psychological stress, illness
chronic plaque psoriasis accounts for almost 90 percent of administration of antimalarial drugs
psoriasis in adults.
Plaques range from coin-sized to much larger.

Inverse Psoriasis:
localized in the flexural surfaces of the skin
e.g.,
armpit, groin
under the breast
other skin folds
Typically, it appears as
smooth inflamed lesions without scaling
and is particularly subject to irritation due to rubbing and
sweating

Pitting of the nails


occurs in 50% of patients.
Histologic features include
hyperkeratosis
parakeratosis
persistent nuclei in the stratum corneum
focal absence of the granular layer
a regular pattern of elongation of the rete ridges
downward extensions of the basal layer
extension of the papillary dermis close to the surface
epithelium,
collections of neutrophils in the stratum corneum
Munro’s microabscesses

HELP LINE NO. 9391567707


TOPIC 4: PSORIASIS DERMATOLOGY 33

Psoriasis –Pitting nails

Parakeratosis
Total nail destruction -Psoriasis Persistence of the nuclei of the keratinocytes into the stratum
corneum of the skin.
This is a normal state only in the epithelium of true
mucous membranes in the mouth and vagina

Subungual hyperkeratosis

Hyperkeratosis and Parakeratosis

Onycholysis

83. A pt. with psoriasis was started on systemic steroids.


After stopping t/t pt. developed generalied pustules
all over his body, the most likely cause is:
A. Drug induced reactions
B. Pustular Psoriasis
C. Bacterial infection
D. Septicemia
B
.........(AIIMS PGMEE NOV - 2001), AIPGMEE - 2002

(1) Hyperkeratosis & parakeratosis Pustular Psoriasis — It is generalized pustule formation


(2) Neutrophils in the epidermis which can complicate as
(3) Thinning of the epidermis overlying the dermal papillae 1) Arthritis
(4) Vessels close to the epidermis 2) Exfoliation
(5) Elongated rete ridges 3) Constitutional Symptoms
Conditions which precipitate pustular psorialsis
1) Steroids
2) Salicylates
3) Iodides

84. The Drug of choice for a pregnant woman in 2nd


trimester with pustular psoriasis is:
A. Prednisolone
B. Dapsone
C. Acitretin
D. Methotrexate
A
.........(AIPGMEE - 2008)

Collections of neutrophils in the stratum corneum 85. The only definite indication for giving systemic
Munro’s microabscesses corticosteroids in pustular psoriasis is:

HELP LINE NO. 9391567707


TOPIC 4: PSORIASIS DERMATOLOGY 34

A. Psoriatic enythroderma with pregnancy. At these sites lesions are seen as Red patches whose surfaces
B. Psoriasis in a patient with alcoholic cirrhosis. are not as scaly as in ordinary psoriasis as the moisture
C. Moderate arthritis. here decreases the scaling and produces a moist
D. Extensive lesions and glazed appearance.
D
..........(AIPGMEE - 2005) 88. Treatment of choice in Pustular psoriasis
A. Psorialin + uv therapy
Systemic corticosteroids are not recommended for B. Systemic steroid
psoriasis, because the disease tends to mcur in more C. Methotrexate
severe form after cessation of therapy. D. Estrogen
Nevertheless, systemic corticosteroids have dramatic C
short term effectiveness. These can thus be used in .........(AIPGMEE - 1994)
extensive or severe form of generalized pustular
psoriasis as a life saving measure when other drugs Drug of choice for psoriasis is PUVA-
are contraindicated or ineffective. Psoralen + UV.A (320-400)
‘Whether or not corticosteroids worsen the long-term Drug of choice for Psoriasis changes in certain special
prognosis, they can be a useful means of controlling associations. These include :
this disease in the short term’ • Psoriatic Arthropathy
• Ps. Erythroderma
86. All are true regarding Psoriasis, except: • Pustular Psoriasis
A. Head, neck and face are not involved Under these conditions the drug of choice becomes
B. Arthritis is seen in 5% of cases Methotrexate.
C. Abscess are seen in lesions
D. Non-scaly, red lesions are seen in infra mammary and natal 89. Least common site involvement in psoriasis is
regions, A. Scalp
A B. Nail involvement
.........(AIPGMEE - 2000) C. CNS involvement
D. Arthritis
• Scalp (head) is involved in almost all cases. But Remember C
that psoriasis of scalp never causes loss of hair and .........(AIPGMEE - 1998)
baldness
• About 5-10% of patients have associated joint complaints Sites involved in psoriasis include :
Psoriatic arthritis affects 5-42% of people with psoriasis. 1. Skin: Skin surface may be involved virutally anywhere,
• Abscesses may be seen in lesions, specially of the although mucosa seem sparedQ.
‘Pustular’ variant . ‘Sometimes though rarely psoriatic • Most usual form : Extensor aspects of trunk and limb
patient develop generalized pustule formation. Pustules ,knees, elbows and scalp are especially frequently affected.
may coalasce to form lakes of pus (Abscesses) • Unusual ‘Reverse Psoriasis : In this form flexural lesions
• In the most usual form, psoriasis affects the extensor are more prominent.
aspect of the trunks and limb preferentially . The (Face is not usually severely affected although the scalp
knees, elbow, and scalp being the most common. margin, paranasal folds, and retroauricular folds are
inverse psoriasis quiet often involved.)
flexural lesions are more prominant.
This is most often seen in : - major body folds in the 2. Nails : Nails are often affected: May show: Thimble pitting,
elderly oncholysis (separation of nail plate from nail bed)
- groins and genitalia, axilla and infmmammary folds in deformities, discolouration and subungual hyperkeratosis
women. • Joints : Arthritis : About 5-10% of patients with which
- skin of abdominal folds and umbilicus in either sex. psoriasis have associated joint complaints. ‘psoriatic arthritis’.

Classic distal interphalangeal joint involvement in psoriatic


arthritis

90. Uses of PUVA:


A. Pigmented purpuric lesion
B. Herpes zoster
C. Mycosis fungoides
D. Lupus panniculitis
E. Lichenoid dermatitis of Gougerot and Blum
A,C and E
.........(PGI - DEC 2004), PGI - DEC 2002

HELP LINE NO. 9391567707


TOPIC 4: PSORIASIS DERMATOLOGY 35

PUVA is used in
- Atopic dermatitis
- Eosinophilic folliculitis
- Pompholyx
- Vitiligo
- Psoriasis

• The pigmented purpuric dermatoses are a group of chronic


diseases of mostly unknown etiology that have a very
distinctive clinical appearance. They are characterized by 92. HPR finding in psoriasis:
extravasation of erythrocytes in the skin with A. Micromunro abscess
marked hemosiderin deposition B. Suprapapillary mining
C. Grenz zone present
• The term pigmented purpuric dermatoses includes D. Pautrier’s abscess
Schamberg disease (ie, progressive pigmentary E. Hyperkeratosis
dermatosis), purpura annularis telangiectodes A,B and E
(Majocchi disease), lichen aureus, itching purpura, .........(PGI - DEC 2006)
eczematidlike purpura of Doucas and Kapetanakis,
and the pigmented purpuric lichenoid dermatosis • Some important Histopathological (HPR) changes in
of Gougerot and Blum psoriasis are :
- Vasodilatation, papillary oedema and leukocyte infiltration
• Pigmented purpuric dermatitis affecting the trunk . in early changes.
• Some of the lesions show the characteristic orange- - Compact hyperkeratosis, disappearance of the granular
brown, speckled, cayenne pepper–like discoloration layer and slight epidermal hyper plasia.
that is the hallmark clinical sign of a capillaritis . - Intraepidermal spongiform pustule.
• Men are more frequently affected than women. If the - Munro microabscess formation in lesional stratum
lesions are pruritic, then the term itching purpura is corneum.
sometimes used. Early cutaneous T-cell lymphoma, purpuric - Irregular epidermal hyperplasia with suprapapillary thinning.
clothing contact dermatitis, and drug hypersensitivity
reactions should be considered in the differential diagnosis. Irregular epidermal hyperplasia with suprapapillary thinning.

Pigmented purpuric lichenoid dermatosis of Gougerot Grenz zone reffers to a narrow zone of normal dermis
& Blum occurs specially in men aged between 40 & 60 between the epidermis and pathological changes
with characteristic clinical features of dermatosis with in the underlying dermis . It is found in Granuloma
presence of lichenoid papules in association with purpuric faciale, an uncommon vasculitic skin disease.
lesion. Psoralen & UVA (PUVA) & Cyclosporin is helpful.

• Phototherapy with both UVB & psoralen with UVA


(PUVA) is the current treatment of Mycosis
fungoides

• Pautrier micro abscesses are seen in Mycosis fungoides or


cutaneous T-cell lymphoma.

93. Causes of erythroderma:


A. Pityriasis alba
B. Pityriasis versicolor
Lupus panniculitis is a rare condition in which
C. Psoriasis
inflammatory changes primarily affects the deep dermis &
D. Lichen planus
subcutaneo us fat. Treatment w ith potent
E. Eczema
corticosteroids & intralesional triamcinolone is
C,D and E
beneficial. f&
.........(PGI - JUNE 2005)

HELP LINE NO. 9391567707


TOPIC 5: PEMPHIGUS DERMATOLOGY 36

• Erythroderma is the term applied to any inflammatory skin Grattage test


disease which affects more than about 90%of the body Grattage Test: Gentle scraping of the surface of a
surface area. psoriasis plaque with a glass slide will remove the loosely
Causes of Erythroderma: attached scales and reveal a shiny surface
• Hereditary disorders- Ichthyosiform erythroderma, pityriasis
rubra pilaris. TOPIC 5 : PEMPHIGUS
- Psoriasis.
- Eczema of various types. 97. Intra epidermal Bulla is seen in:
- Drugs - organic arsenic, gold, mercury, occasionally, A. Pemphigoid
barbiturates etc. B. Pemphigus
- Pemphigus foliaceus. C. Dermatitis Herpetiformis
- Lymphoma & Leukemias D. Light reaction,
- Other skin diseases like Lichen PIanus, Dermatomyositis. B
- Unknown. .........(AIIMS PGMEE - Dec - 1995)

“Pemphigus causes intraepidermal blistering because of


loosening of desmosomal links b/n> epidermal cells caused
by immunological attack”.

94. Treatment of psoriasis:


A. PUVA
B. Methotrexate 98. A 56 year old male having painful bullous lesion in
C. Systemic steroids lower extremity, the most likely diagnosis is:
D. Femicycline A. Pemphigus vulgaris
E. Terbinafine B. Bullous pemphigoid
A and B C. Necrotising pemphigus
.........(PGI - JUNE 2005) D. Contact ezema
B
95. Koebner’s phenomenon is characteristic of: .........(AIIMS PGMEE - JUNE - 1997)
A. Psoriasis
B. Pemphigus vulgaris Pemphigoid is commonly seen in old age (60-80 yrs) as
C. Pityriasis rosea bullous eruptions on lower limb.
D. Lupus vulgaris
A
.........(PGI - June -1999)

• Koebner’s phenomena is linear lesions produced by


scratching a primary lesion, which results in new lesions
developing along the line of scratch It is found in : Lichen
planus, Warts, Psoriasis

96. Auspitz sign is seen in:


A. Pustular psoriasis 99. Nikolsky sign is positive in all Except:
B. Plaque type psoriasis
A. Pemphigus
C. Lichen planus
B. Bullous Pemphigoid
D. Pityriasis rubra pilaris
C. Toxic epidermonecrolysis
B
.........(AIIMS PGMEE - MAY 2008) D. Stapylococcal scalded skin syndrome
B
• Auspitz sign is a feature of plaque psoriasis . .........(AIIMS PGMEE JUNE - 2000)
• Plaque psoriasis or psoriasis vulgaris in the most common
form of psoriasis. Nikolsky sign —> Sliding of superficial layers of skin over
Auspitz sign deeper layers when a tangential force is applied with a
It is a bedside test to confirm the clinical presence of psoriasis. pulp of thumb especially over bony prominences.
There are 3 steps to this test It is positive in
• Step A -Gently scrape the lesioin with a glass slide. This • Staphylococcal scalded skin syndrome
accentuates the silvery scales Scrape off all the scales. • Toxic epidermonecrolysis
• Step B -As you continue to scrape the lesion, a. glistening • Pemphigus
white adherent membrane appears. • Epidermolysis Bullosa congenita
• Step C -On removing the membrane, punctate bleeding • Nikolsky sign is due to Acantholysis so it is positive in
points become visible. pemphigus but not in pemphigoid (Acantholysis is absent
The other bedside test for Psoriasis is:- in pemphigoid)

HELP LINE NO. 9391567707


TOPIC 5: PEMPHIGUS DERMATOLOGY 37

• Acantholysis means lack of cohesion between epidermal 6. Nicolasky sign present


cells, with subsequent separation. 7. Age Group 40-50

100. A 24 years old female has flaccid bullae in the skin Pemphigoid
and oral e rosions. Histopatho logy shows 1. Vescicles seen on the lower part of body, limbs >trunk.
intraepidermal acantholytic blister .The most likely 2. Lesions are non itchy and painless
diagnosis is: 3. Oral mucosa not involved
A. Pemphigoid. 4. Lesions are subepidermal
B. Erythema multiforme. 5. Acantholysis absent
C. Pemphigus vulgaris. 6. Nicolsky sign absent
D. Dermatitis herpetiformis 7. Age Group 60-80
C
..........(AIIMS PGMEE MAY - 2003) Dermatitis Herperiformis
1. Vescicles seen on extensor surface
• Bullae involving oral mucosa with intraepidermal lesions & 2. Vesicles are itchy and painless
acantholysis is characteristic of pemphigus. 3. Villous atrophy usual
• Here is a D/d of commonly asked vesico bullous disorders 4. Lesions are subepidermal
5. Young adults are involved
Vesicobullous disorders
INFECTIOUS
Herpes Simplex
1. Cluster of vesicles usually on face
2. Painful
3. Recurrent
Two types
a)TypeI-
Seen in childhood
gingivostomatitis, fever, vesciles on lips
Erythema multiforme
b)Type H-In young adults after sexual contact usually involves
1. Characterstie vesicular lesions on hands and feet (target
genitalia
lesion)
2. non itchy and painless.
3. face and upper limbs involved
4. mucosal involvement can be seen

Herpes Zoster
1. Vesicular eruption occurs in dermatomal pattern.
Thoracic dermatome is the most commonly involved (50%)
2. Involvement of ophthalmic division of trigeminal N.
3. Painful and tender
4. 2/3 of patients over 40 yrs of age
101. A 40 yr old female developed persistent oral ulcers
followed by multiple flaccid bullae on trunk and
extremeties. Direct examination of a skin biopsy
immunoflurescene showed intercellular IgG deposits
in the epid ermis and suprabasal split with
acantholytic cells. The probable diagnosis is
A. Pemphigus vulgaris
B. Pemphigoid
C. Ery theme multiforme
D. Dermatitis herpetiformis
A
.........(AIPGMEE - 2008), AIPGMEE - 2000), AIPGMEE -
2003
NON-INFECTIOUS
Pemphigus Pemphigus Vulgaris (PV)
1. Vescicles are seen on the upper part of body. Trunk > Pemphigus is an autoimmune blistering disorder resulting
limbs. from the loss of integrity of normal intercellular attachments
2. Lesions are non itchy arid painless within the epidermis.
3. Oral mucosa involvement in 50% cases. Most individuals are in fourth to sixth decades of life.
4. Lesions are intraepidennal Men & women are affected equally.
5. Acantholysis present Associated with

HELP LINE NO. 9391567707


TOPIC 5: PEMPHIGUS DERMATOLOGY 38

Increased incidence of HLA -DR4 and HLA - DRW6 of calcium dependent adhesion molecules
serological halotype Early PV (i.e. mucosal involvement only) have only anti -
Drugs (penicillamine, rifampicin), neoplasm (thymoma & Dsg3 autoantibodies
lymphoma), and mysthenia gravis are rarely associated Advanced PV (i.e. mucosal & skin involvement both) have
both anti Dsg3 and Dsg - 1 autoantibodies.

Prognosis ‘
• Can be life threatening with mortality of ~5%.
Infection & complication of steroid treatment are most
common cause of morbidity & mortality.
Treatment
Systemic glucocorticoid are the mainstay of treatment.

(1) Suprabasal, intraepidermal bullae


(2) Superficial perivascular inflammatory infiltrate
(3) ”Tombstoning” of basal keratinocytes
(4) Normal stratum corneum with basketweave appearance
Flaccid blisters on eithe r normal appea ring or (5) Acantholysis
erythematous skin. These blisters rupture easily, leaving
denuded areas that may crust and enlarge peripherally.
• Manual pressure to the skin elicit the seperation of the
epidermis (Nikolsky sign)Q.
• This finding, is characteristic of Pemphigus Vulgaris but
is not specific

• Lesions typically present on oral mucosa, scalp, face,


neck, axilla, and trunk (upper half of body). In most • Oral ulcerations –pemphigus vulgaris
patients, lesions begin in the mouth. Involvement of
other mucosal surfaces (eg. pharyngeal, laryngeal,
esophageal, conjuctival, vulval or rectal) can occur in
severe disease.
• Pruritis may be seen in early lesions and severe pain is
associated with extensive denudation.
• Lesions usually heal without scarring (except in secondary
infections & mechanical denudation). None the less post
inflammatory hyperpigmentation is usually present at sites
of healed lesions for some time.

Histology
Acantholytic blister formed in suprabasal (deep) layer Pemphigus vulgaris
of epidermis
The single layer of intact basal cells that form blister base
has been likened to a row of tombstone

• Eyelid Pemphigus

Immuno - Pathology
Direct immunofluorescence microscopy (DTM) of lesion
or intact patient skin shows deposits oflgG on the
surface of keratinocytes in a fish net like pattern; in
contrast complement components are typically found
in lesional (but not uninvolved) skin.
• Indirect immuno fluorescence microscopy (using monkey
oesophagus) demonstrates circulating autoantibodies
against cell surface antigen.

• Auto antibodies (IgG) are directed against Ds gs


(desmogleins), transmembran e desmosomal
glycoprotein that belong to cadhein supergene family

HELP LINE NO. 9391567707


TOPIC 5: PEMPHIGUS DERMATOLOGY 39

Skin lesions appear as


thin walled flaccid blisters
filled with clear fluid
easily rupture
causing painful erosions.

Erosions in the skin folds


=
develop into vegetative lesions which are granular and crusty
looking
=
known as pemphigus vegetans acantholysis in Pemphigus Vulgaris

PV - Layer of basal cells still attached to the underlying


connective tissue. A feature of the intraepithelial vesicle
formation.
Pemphigus vulgaris Often called “tombstones”.

Oral ulcerations –pemphigus vulgaris PV - acantholysis - Tzank cells (epithelial cells which are
smaller, rounder)

102. An Autoimmune Disease is all except:


A. Pemphigus vulgaris
B. Psoriasis
C. Lichen Planus
D. Acne vulgaris
Ans D PV - Direct immunofluorescence: IgG deposition in
.........(AIPGMEE - 2000) intercellular areas

• Pemphagus vulgaris is an autoimmune disease.


• This disorder is characterised by loss of cohesion between
epidermal cells.

• In pemphigus vulgaris auto-antibodies are directed against


a desmosomal cadherin molecule2 (desmoglein 3) that
forms a complex with a protein found in both desmosomal
and adherens junctions.
Anti desmoglein 3 antibodies are responsible for dys- PV - Direct immunofluorescence: IgG deposition in
adhesion of epidermal cells intercellular areas

Psoriasis is a genetically determined disorder, but its origin is


unknown
• Licheplanus is an inflammatory disorder of skin with
unknown origin
• Acne vulgaris too, has no autoimmune basis

suprabasal separation of the epithelial cells intraepithelial


vesicle -PV

HELP LINE NO. 9391567707


TOPIC 6: SYPHILIS DERMATOLOGY 40

104. Drug induced pemphigus is seen in A/E • In burn - subepidermal bulla can seen be cause of damage
A. Penicillin to basal cells arid basement npmbrane.
B. Phenopthelein
C. Iodine 112. In pemphigus vulgaris, antibodies are present
D. Frusemide against:
D A. Basement membrane
.........(AIPGMEE - 1994) B. Intercellular substance
C. Cell nucleus
Drugs inducing Pemphigus is D. Keratin
Penicillin B
Iodine
.........(PGI - June -2000)
Phenolpthelin

105. Acantholysis is seen in • Circulating auto-antibodies can be demonstrated in 80 to


A. Epidermis 90% of PV patients by indirect immunofluoresence
B. Epidermo-Dermal Junction microscopy. Patients with pemphigus valgaris, have Ig auto-
C. Dermis antibodies directed against desmogleins, transmembrane
D. In all the layers of Skin desmosomal glycoprotein that belong to cadherin
A supergene family of calcium dependent adhesion
.........(AIPGMEE - 1995) molecules.
AIPGMEE - 2003
TOPIC 6: SYPHILIS
Acantholysis
• is loss of cohesion between epidermal cells. It is 113. Characteristic feature of early congenital syphilis
hallmark of Pemphigus Vulgaris Q is:
• It is due to destruction of intercellular substance A. Microcephaly
• Acantholytic cell is derived fom Stratum basale
B. Saddle nose
St. granulosum Between st. basale and prickle C. Interstitial keratitis with saber skin
cell laver D. Vesicular rash with bulla over palms and soles
In P . foliaceous & P . In P . vulgaris and P . vegetans D
erythematosus .........(PGI - 1999 - Dec)
• Diagnosis confirmed by skin biopsy and demonstration of
acantholytic cells in Tzanck smears • The cutaneous lesions in congenital syphilis are vary
characteristic. Within 7 days of birth — bullous eruption
108. A 25 year old female has palatal ulcers & skin (SYPHILITIC PEMPHIGUS) symmetrical and bilateral on
blisters, most likely diagnosis is: front of wrists, palms, soles, ankles and later on in any
A. Pemphigus vulgaris
part of the body.
B. Pemphigus follacious
• Later on maculo-papular rash all over or on the
C. Dermatitis herpetiformis
D. Pemphigoid buttocks, where it may resemble NAPKIN RASH .
A Interstitial keratitis, sabre tibia, saddle nose are late (after
.........(AIPGMEE - 1997) 2 yrs) features.

109. Intraepidermal blister seen in 114. Which is not a manifestation of secondary syphilis:
A. Pemphigus vulgaris A. Vesicle and bullae
B. Paraneoplastic pempigus B. Hyperpigmented macules
C. Bullous pemphigoid C. Nodules
D. Dermatitis herpetiformis D. Rashes
E. Epidermolysis bullosa acquisita A
A and B .........(AIIMS PGMEE - FEB - 1997)
.........(PGI - DEC 2005), PGI - JUNE 2004 AIIMS PGMEE - JUNE - 1997

111. Sub-epidemal splitting is not found in:


• Bullae and vesicles are seen in congenital syphilis not in
A. Bullous pemphigoid
secondary syphilis.
B. Pemphigus foliaceus
C. Dermatitis herpetiformis
D. Burns 116. Multiple, painful ulcers over glans penis without
B induration is suggestive of
.........(PGI - June -1999) A. LGV
B. Granuloma inguinale
• In pemphigus foliaceus, there is subcorneal epidermal C. Chancroid
split. D. Secondary syphilis
Nikolsky’s sign ‘Positive’ C
.........(AIIMS PGMEE - NOV - 1993)
• In bullous pemphigoid, Dermatitis herpetiformis -
there is subepidermal pressure bulla. Differential diagnosis of genital ulcer:

HELP LINE NO. 9391567707


TOPIC 6: SYPHILIS DERMATOLOGY 41

Features Disease
Syphilis Herpes Chancroid LGV Donovanosis
No. of lesions Single Multiple Multiple Single Variable
Pain No Yes Yes Variable No
Base Non Red Red Non Red
Vascular ' vascular
Induration Yes No No Variable Yes
Lymphadenop Yes Yes Yes Yes No (Pseudo -
athy (Suppurativ (Suppur buboes)
e) ative)

Syphilis ulcer with indurated base

Lymphogranuloma venereum: large left inguinal bubo

Chancroid usually starts as a small papule that rapidly becomes


pustular and eventually ulcerates. The ulcer enlarges,
develops ragged undermined borders, and is surrounded • Condylomata lata.
by a rim of erythema.
Unlike syphilis, lesions are tender and the border of the ulcer
is not indurated

Chancroid -lesions are tender and the border of the ulcer is


not indurated

Characteristic lesions of chancroid. The bubo on the right


side drained spontaneously. The bubo in the left inguinal
canal required needle aspiration

117. A 24 year old male presents to a STD clinic with a


single painless ulcer on external genitalia. The choice
Chancroid of laboratory test to look for the etiological agent
• Large fluctuant buboes should be drained with the would be:
patient under local anesthesia and a large-gauge A. Scrappings from ulcer for culture on chocolate agar with
needle inserted through surrounding healthy skin. antibiotic supplement.
The insertion site should be superior or lateral to B. Serology for detection of specific IgM antibodies.
the bubo to prevent chronic drainage from the site C. Scrappings from ulcer for dark field microscopy.

HELP LINE NO. 9391567707


TOPIC 6: SYPHILIS DERMATOLOGY 42

D. Scrappings from ulcer for tissue culture 120. Primary bullous lesions is seen in which type of
C syphilis
..........(AIIMS PGMEE MAY - 2003) A. Primary
B. Secondary
118. Condylomata lata is seen in: C. Tertiary
A. Primary syphilis D. Congential
B. Secondary syphilis D
C. Congenital syphilis .........(AIPGMEE - 1994)
D. Tertiary syphilis
D 121. All of the following are true about syphilis except:
.........(AIIMS PGMEE NOV - 1999) A. VDRL is sensitive but NOT specific
B. Infection leads to life long immunity
gummas observed in tertiary syphilis C. IgM & lgA
D. Treponoma pallidium when inocculated in rabbit produces
progressive disease
B and C
.........(AIPGMEE - 1998)

122. Sec. syphilis,true about rash is:


A. Pruritic
B. Vesicular
C. Asymptomatic
D. Tender
C
.........(PGI - 1998 - Dec)
• condylomata lata. The lesions resemble genital warts
(condylomata acuminata).
• Fluids exuding from these lesions are highly
infectious

• In secondary syphilis - rash is asymptomatic. .


• Rash is characteristically
— Non tender
— Non pruritic
— Maculopapular rash
— Reddish. Coppery colour

119. A young man aged 19 years develops a painless


penile ulcer 9 days after sexual intercourse with a
professional sex worker;likely diagnosis is;
A. Chancroid
B. Herpes
C. Chancre
D. Traumatic ulcer — Generalised and bil. Symmetrical
C — Pleomorphic appearance
.........(AIPGMEE - 2001) — Induration and infiltration
— Adenopathy
• Chancre or syphillitic ulcer in the primary stage, presents
as a painless, indurated papule which becomes eroded • This syphilis patient presented with a “roseola rash”,
later on. Also the appearance of Chancre fall within the similar to that of viral eczema , which developed on
incubation period which is between 9 to 90 days. her buttocks and legs during the secondary stage of the
disease

HELP LINE NO. 9391567707


TOPIC 6: SYPHILIS DERMATOLOGY 43

• Secondary syphilis is the most contagious of all the stages, gumma of nose due to a long standing tertiary syphilitic
and is characterized by the spread of the Treponema
pallidum bacteria throughout the body, causing systemic
symptoms that includes cutaneous lesions such as the
skin rash seen on this patient’s right thigh.

• Note symmetrically hyperpigmented scaly minimally elevated


papules and plaques. The rash varies substantially — it
may be flat or raised. Involves palms and soles.
124. Jarish herxheimer reaction is seen in early cases of:
A. Syphilis
B. Gonorrhoea
C. LGV
D. Granuloma iguinale
A
.........(PGI - June -1998)

• Jarish hexheimer reaction is seen in early cases of syphilis.


• It is due to sudden release of spirochetal lytic products
and lasts 12-72 hours.
123. Secondary syphilis manifested by: It is characterised by - fever, myalgia and exacerbation of
A. Painless lymphadenopathy. lesions.
B. Pruritic rash.
C. Mucosal erosion. 125. In secondary syphilis all are seen except:
D. Asymptomatic rash. A. Condyloma lata
E. Mostly asymptomatic B. Interstitial keratitis
C and D C. Arthritis
..........(PGI - DEC 2003) D. Proteinuria
B
These serpiginous, nodular ulcerative lesions are due to late .........(PGI - June -1998)
syphilitic disease

signs and symptoms of late stage syphilis include not being


able to coordinate muscle movements, par alysis,
numbness, gradual blindness and dementia.

• Interstitial keratitis is a feature of late congenital syphilis.


chancre is usually firm, round, small, and painless. It
Condyloma lata, arthritis, proteinuria - all found in secondary
appears at the spot where the T. pallidum bacteria
syphilis.
entered the body. The chancre lasts 3 to 6 weeks,
• secondary syphiliti c rash covering his back
and it heals without treatment
representing the systemic spread of the Treponema
pallidum bacteria.

chancre located on the penile shaft due to a primary


syphilitic infection

primary anorectal syphilitic chancre during the primary


stage of the disease

HELP LINE NO. 9391567707


TOPIC 7: ACNE DERMATOLOGY 44

Latent syphilis is further subdivided into 2 stages.


Early latent syphilis (Early relapsing syphilis)
• This stage occurs during the first two years of infection
• In this stage the patient remains infective.
• In this stage the patient remains asymptomatic but
about 25 % of patients will have one or more relapses
and they develop the infectious skin lesion of secondary
syphilis or chancre redux.
• patient presented with a primary syphilitic chancre of the
Late latent syphilis
lip.
• This stage occurs after 2 years
• There are no relapses in this stage.
• The patient becomes non infective
Course of the disease after latent syphilis
• About l/3rd of untreated patients will slowly progress
from this stage to tertiary syphilis.
• The rest will remain asymptomatic.

128. All are features of secondary syphilis Except:


A. Papulosquamous lesion
B. Bullous lesion
C. Nodular lesion
D. Lichenoid type lesion
Tertiary Syphilis (Stage Four) B
.........(AIIMS PGMEE - JUNE 1998)
126. Mucous lesions are seen in:
A. Sec. Syphilis • The skin rashes in the Secondary stage of syphilis
B. Dermatitis herpetiformis consists of
C. Psoriasis Macule
D. Pemphigus Papule
E. Porphyria Papulosquamous
A and D Pustules
.........(PGI - June -2002) Lichenoid eruptions
• Vesico Bullous lesions do not occur in secondary
• The protean menifestations of the secondary stage of syphilis
syphilis usually include localized or diffuse symmetric • Bullous lesions are seen only in Congenital syphilis.
mucocutaneous lesions & gener alized nontender
lymphadenopathy. TOPIC 7: ACNE

127. ‘Chancre redux’ is a clinical feature of: 129. Treatment of choice of Nodulocystic Acne is:
A. Early relapsing syphlis A. Erythromycin
B. Late syphilis B. Isoretinoin
C. Chancroid C. Tetracycline
D. Recurrent herpes simplex infection D. PUVA
A B
.........(AIIMS PGMEE - MAY 2006) .........(AIIMS PGMEE - MAY 1995)
AIIMS PGMEE - MAY – 1994, AIIMS PGMEE MAY – 2002
• Chancre redux is the appearance of relapsing lesion AIPGMEE - 1994
at the site of the healed lesion.
• An important point to remember is that the lesion does Treatment for Acne
not occur due to reinfection, but it is the relapse
of the original infection. TOPICAL ORAL
• Chancre redux Antimicrobial Antimicrobial
Suppose a person presents with painless chancre. On t/t Benzoyl Peroxide Tetraycline
the chancre disappears i.e. the person is cured. Tetracycline Minocycline
After a certain period of time the patient presents again Erythromycin Erythromycin
with the same lesion at the same site, but he does not Clindamycin
give any H/O of exposure to the organism. So, it is a case Azelaic alid
of relapse not reinfection. Comedolytic Sebum Suppressive
• Chancre redux are seen in latent syphilis Tretinoin Isotretinoin
Isoretinoin Spironolactone
Latent syphilis Salicylic acid Cyproterone &
• This stage occurs between the secondary and tertiary Ethinylestranol
stage of syphilis.
• In this stage the features of secondary syphilis have Stages of acne. (A) Normal follicle; (B) open comedo
resolved i.e. t he patient is asympto matic bu t (blackhead); (C) closed comedo (whitehead); (D) papule;
serologically positive. (E) pustule

HELP LINE NO. 9391567707


TOPIC 7: ACNE DERMATOLOGY 45

• Azelaic acid is available as a 20 percent cream , which is


applied twice daily to a clean, dry affected area. The agent
is fairly well tolerated, with only about 5 percent of patients
complaining of tr ansient cutaneous irritation and
• Severe nodulocystic acne not responsive to oral antibiotics, erythema.This rate is lower than the incidence of such
hormonal therapy or topical therapy may be treated with complaints reported for benzoyl peroxide and tretinoin.
synthetic retinoid, Isoretinoin”. Because azelaic acid decreases pigmentation, it
should be used with caution in patients with darker
complexions

• Salicylic Acid
Salicylic acid is an ingredient of various over-the-counter
preparations. It is available at a concentration of 0.5 or 2
percent in a number of creams and lotions.
This agent inhibits comedogenesis by promoting the
desquamation of follicular epithelium. It has been shown
to be as effective as benzoyl peroxide in the treatment
• Retinoids, which are derivatives of vitamin A , function of comedonal acne
by slowing the desquamation process, thereby decreasing Salicylic acid is well tolerated and should be applied once
the number of comedones and microcomedones. Retinoids or twice daily.
are the most effective comedolytic agents in use
Benzoyl Peroxide
• Benzoyl peroxide, available over the counter and by
prescription
• This agent has bactericidal a nd comedolytic
properties. It is the topical agent most effective against
P. acnes,7 with bacteriostatic activity superior to that of
topical antibiotics.8 It also functions as a mild comedolytic
agent by increasing epithelial cell turnover with
desquamation.
• Benzoyl peroxide can be obtained in various concentrations
(2.5 to 10 percent), although little evidence exists that
efficacy is dependent on the dose.8 This agent comes in
water-based or alcohol-based gels. The water-based
formulations are less drying than the alcohol-based
preparations. Benzoyl peroxide gels are applied once or
twice daily.
• Skin irritation is the most common side effect of
benzoyl peroxide. This effect occurs more often at
higher concentrations and tends to decrease with
continued use. Contact allergy occurs in 1 to 2 percent
of patients.9 Patients using benzoyl peroxide formulations
for the first time should be instructed to test for allergic
dermatitis by applying a small amount of the agent in the
antecubital area before using it on the face.

• Nodulocystic form of acne is a severe variant of the acne.


They can be very disabling and disfiguring. Nodules and
cysts subsides giving rise to scars.
• Drug of choice for nodulocystic acne is – Isoretinoin
More on t/t of Acne -
a) Local therpy- Frequent washing with soaps & water
b) Topical agents - widely used agents are -
i) Benzoyl peroxide
ii) Vitamin A
• Azelaic Acid iii) Antibiotics - clindamycin & erythromycin
• Azelaic acid is a decarboxylic acid that was first investigated
in the 1970s as a treatment for hyperpigmentation and Drug od chioce is istotretnoin (Synth. Retnoic Acid)
was coincidentally found to be an effective acne • Most common side effect of retinoids is Skin Rash
treatment.14
• azelaic acid is used for the treatment of mild to moderate • Oral therapy - Major systemic modalities are –
inflammatory acne. Although its exact mechanism of action i) Broad spectrum antibiotics
is unknown, this agent has antibacterial and antikeratinizing • Tetracycline
activity, and it appears to be as effective as benzoyl • Minocycline
peroxide or tretinoin (Retin-A) in the treatment of mild • Doxycycline
to moderate acne. ii) Oestrogen - This decreases sebum production

HELP LINE NO. 9391567707


TOPIC 7: ACNE DERMATOLOGY 46

• Glucocorticoids - They cause reduction in plasma Open comedo (blackhe ad) is a 0.1- t o 3.0-mm
androgen levels and hence decrease sebum production noninflammatory lesion that looks like a black dot
used in severly affected patients
• Isotretinoin - Used for nodulocystic acne

132. Acne Vulgaris is due to involvement of:


A. Sebaceous Gland
B. Pilosebaeous Gland
C. Eccrine Gland
D. Apocrine Gland
B
.........(AIIMS PGMEE NOV - 2001)

Acne vulgaris is due to obstruction of Pilosebaceous Duct


Lesions consists of
— Comedone
Papules – Comedone
Pustules – The cells lining the sebaceous duct proliferate
Cysts excessively in acne (cornification) and may block the
sebaceous duct forming a comedone. These may be
• Acne has equal sex incidence so small that they are not visible to the naked eye
• Most common site — Face (microcomedones).

Normal pilosebaceous unit.

Primary lesion of acne vulgaris is the microcomedo


microscopic, bulging mass that results from a combination Closed comedones are whiteheads; the follicle is completely
of blocked
hyperproliferative corneocytes and sebum
leads to follicular plugging.

Open comedones are blackheads; black because of surface


pigment (melanin) rather than dirt
Closed comedo (whitehead) is the first visible acne lesion • Comedonal acne.
a noninflammatory lesion
evolves from the microcomedo
appears as a white dot ranging from 0.1 to 3.0 mm in
diameter

HELP LINE NO. 9391567707


TOPIC 7: ACNE DERMATOLOGY 47

Papulopustular acne. with the production of irritating fatty acids


that produces an inflammatory reaction.

Aggravating factors include


hormones (e .g., testosterone, pr ogesterone,
glucocorticoids), drugs (e.g., lithium)
occupational exposures (e.g., grease).
Dietary factors (e.g., chocolate, nuts)
=
do not contribute to acne.
• Nodulocystic acne Acne rosacea
an inflammatory disease of the pilosebaceous units
on the face
in middle – aged individuals.

Acne : (mild) - non-cystic acne


Acne rosacea

• Acne : (Moderate Severity) - some cysts


• Phymatous rosacea

• Disfiguring rhinophyma
Acne : (Severe) - cysts

133. A 24-year-old unmarried woman has multiple


nodular, cystic, pustular and comadonic lesions on
face, upper back and shoulders for 2 years. The drug
of choice for her treatment would be:
pathogenesis of the obstructive type (comedones)
A. Acitretin
related to plugging of the outlet of a hair follicle
B. Isotretinoin
by keratin debris.
C. Doxycycline
D. Azithromycin
inflammatory type is associated with
B
abnormal kertinization of the follicular epithelium
.........(AIPGMEE - 2006)
increased sebum production (androgen controlled)
bacterial lipase (derived from Propionibacterium acnes)
• Acne, grade I; multiple open comedones

HELP LINE NO. 9391567707


TOPIC 7: ACNE DERMATOLOGY 48

C. Systemic lupus erythematosus.


D. Polymorphic light eruption
B
..........(AIPGMEE - 2005)

‘Rosacea is a chronic inflammatory disorder of the skin of


the facial convexities characterized by persistent
erythema and telangiectasias punctuated by acute
episodes of swelling, papules and pustules’

• Acne, grade II; closed comedones 136. Features of acne vuigaris:


A. Increased Sebum secretion
B. Not seen after 20 years
C. Increased Androgen production causes
D. Rx is penicillin for 1 week.
A and C
.........(PGI - JUNE 2006)

Acne vulgaris is a chronic inflammatory disease of the


pilosebaceous unit.
It is characterized by: Sebum production
Hypercornification of the pilosebaceous duct.
Abnormality of the microbial flora epecillally colonization of
the duct with P. acnes & Production of inflammation.
This condition usually starts in adolescence &
• Acne, grade III; papulopustules frequently resolves by mid twenties.
A peak in prevalence & severity occurs between 14 & 17
yrs in females & 16 & 19 yrs in males.
Patients with acne vulgaris, the free testosterone
level (androgen) is above normal.

137. Treatment of acne:


A. 13 - cis retinol
B. Minocycline
C. Erythromycin
D. Dapsone
E. Rifampicin
A,B and C
.........(PGI - JUNE 2005)
• Acne, grade IV; multiple open comedones, closed
Treatment for acne :
comedones, and papulopustules, plus cysts
Topical
Antimicrobial Comedolytic Antimierobial
Benzoyl peroxide Tretinoin Tetracycline
Tetracycline Isotretinoin or Minocycline
Erythromycin 13-cisretionic acid Doxycycline
Azelaic acid Adapalene Erythromycin
139. Causative factor for acne are following except:
A. Androgen
B. Only food
C. Bacterial contamination
D. Hypercornification of duct
B
.........(PGI - June -2000)
Acne with reactive hyperpigmentation; before treatment vs
after treatment • Acne vulgaris is of unknown cause and is apparently
activated by androgens in those who are geneticaly
134. A 40-year-old woman presents with a 2-year history predisposed.
of erythematous papulopustular lesions on the • Pathogenesis : Seborrhoea, hyperkeratosis of the pilo-
convexities of the face.There is a background of sebaceous ostia, plugging of infundibulum of the follicles,
erythema and telangiectasia.The most likely diagnosis retention of sebum, overgrowth of acnebacillus (propioni
in the patient is: bacterium acnes) with resultant release of and irritation
A. Acne vulgaris. by accumulated fatty acids and foreign body
B. Rosacea reaction to extra follicular sebum.

HELP LINE NO. 9391567707


TOPIC 8: TUBERCULOSIS SKIN DERMATOLOGY 49

140. Comedones are seen in: 141. A 19-year-old girl has multiple, papulopustular,
A. Pityriasis erythematous lesion on face and neck. The likely
B. Lichen planus diagnosis is
C. Adenoma sebaceum A. Acne rosacea
D. Acne vulgaris B. Acne vulgaris
D
C. Pityriasis versicolor
.........(AIIMS PGMEE - DEC 1998)
D. Lupus vulgaris
• Primary lesion of Acne Vulgaris is comedone B
• Open comedones are Black Heads .........(AIIMS PGMEE - DEC 1994)
• Closed comedones are White Heads
Comedones can evolve into TOPIC 8: TUBERCULOSIS SKIN
• Papules
• Pustules & 142. Skin tuberculosis which involves skin after involving
• Cysts lymph nodes:
A. Scrofuloderma
B. Lupus vulgaris
C. Lupus erythmatosis
D. Lupus pernio
A
.........(AIIMS PGMEE - FEB - 1997)

143. An 8 year old boy presents with a well defined


annular lesion over the buttock with central scarring
that is gradually progressing over the last 8 months.
The diagnosis is:
Site of Appearance (requires presence of Sebaceous gland) A. Annular psoriasis
• Face(MC) B. Lupus Vulgaris
• Shoulder C. Tinea corporis
• Back & Upper chest D. Chronic granulomatous disease
B
..........(AIIMS PGMEE MAY - 2001)
AIIMS PGMEE - DEC 1998

Annular lesions with central scarring in 8 yr old child strongly


suggests lupus vulgaris.

Lupus Vulgaris Annular Tinea Corporis


• Acne vulgaris dorsi Psoriasis
• Central Scarring is seen • Central • Central clearing
clearing occurs occurs
• Other features • Other features
• Most common • Fungal infection
manifestation of • Single or multiple
cutaneous T.B.,
plaque and
commonly seen in
erythema
children.
• Lesion is usually • Lesion enlarges
erythematous, indurated, with central clearing
plaque

144. A 12 year old boy had a gradually progressive plaque


on a buttock for the last 3 years. The plaque was 15
Plaques in Acne cm in diameter, annular in shape, with crusting and
induration at the periphery and scarring at the
center. The most likely diagnosis.
A. Tinea corporis
B. Granuloma annulare
C. Lupus vulgaris
D. Borderline leprosy
C
.........(AIIMS PGMEE NOV - 2003)

A lesion with central scarring in a 12 year old boy gradually,


progressing over 3 years is quite suggestive of Lupus
vulgaris

HELP LINE NO. 9391567707


TOPIC 8: TUBERCULOSIS SKIN DERMATOLOGY 50

Remember

Feature Disease Investigation


Central clearing Tinea corporis KOH smar Biopsy
Central scarring Lupus vulgaris LD
Central crusting Leishmaniasis body demonstration
Clinical features of Lupus vulgaris
Commonest manifestation of cutaneous tuberculosis
Most frequent age group is children and young adults • Tuberculosis Cutis Orificalis
It usually affects die exposed areas of die body. • This type of T.B. involves mucosa
• Mucosa of Anal region after Gastro intestinal T.B.
Face is the most common site.
• Mucosa of urogenital region after Renal T.B.
The lesion is asymmetrically distributed.
• Mucosa of Oral region after Pulmonary T.B.
Can also affect hands, feets, knees and buttock
The lesion starts as reddish brown macule and enlarges slowly
over months and years to form big patch with a well
defined but irregular margin.

The diagnostic features are: -


1. Persistent, brownish-red, well defined patch with dermal
infiltration
2. Presence of apple jelly nodules
3. On healing a, tissue paper like scarring is produced in Histology of cutaneous tuberculosis: Note the
the centre Langhans’ giant cells
4. Match stick test positive - An apple jelly nodule has no
resitance to pressure by a sharp match stick
5. Long history expanding over years
• Tuberculin test positive. Miliary tubercles in a biopsy
specimen
• Diagnosis of Lupus vulgaris is made by - Biopsy

145. Tuberculides is seen in


A. Lupus vulgaris
B. Scrofuloderma
C. Lichen scrofulosorum • Tuberculosis Verrucosa Cutis
D. Tuberculoris Cutis Orificialis – It is almost similar to lupus vulgaris, the only difference is
B that lesions are more Verrucous and usually located
.........(AIPGMEE - 2007) on the hand or foot

Lupus vulgaris • Tuberculids: Tuberculids are symmetric generalized


• Lupus Vulgaris ◊ It is T.B. of skin exanthems in the skin of tuberculous patients,
possibly resulting from hypersensitivity reactions
to tubercle bacillus. Typically, patients with tuberculids
are in relatively good health and show (1) positive
tuberculin sensitivity, (2) tuberculous involvement (usually
inactive) of viscera or lymph nodes, (3) negative staining
and culture for pathogenic mycobacteria in affected tissue,
and (4) skin lesions that heal with remission or treatment
of TB.

Lupus vulgaris. A hyperkeratotic, crusted, granulomatous


plaque (or plaques) on the face or elsewhere is
characteristic. An apple-jelly color is seen on diascopy
(pressure to remove the blood with a glass slide

• Scrofuloderma T.B. of skin after the


involvement of
underlying structure
such as lymph node
or Joint.

HELP LINE NO. 9391567707


TOPIC 8: TUBERCULOSIS SKIN DERMATOLOGY 51

• Originally, these exanthems were believed secondary to 150. M.C. type of cutaneous T.B. is:
mycobacterial “toxins”; however, recent opinion and A. Lupus vulgaris
identification of mycobacterial DNA by PCR B. Scrofuloderma
amplification reactions in affected tissue suggest that
C. T.B. verruca cutis
they are manifestations of hematogenous spread of bacilli
D. Erythema induratum
in patients with tuberculin immunity.
A
.........(PGI - DEC 2006)
Atypical mycobacterial infection. This fish-tank granuloma
developed several months after the patient cleaned his
• Lupus vulgaris is the most commonly found form of
aquarium. Any hobbies or occupations that bring the cutaneous T.B. in adults
patient in contact with fish are predisposing factors to
this infection by Mycobacterium marinum. 151. Skin manifestation of T.B.
A. Lupus vulgaris
146. Which of the following is/are tuberculides B. Lupus pernio
A. Lichen scrofulosorum C. Scrofuloderma
B. Lichen nitchidus D. Butcher warts
C. Lichen aureus A and C
D. Erythema nodosum .........(PGI - JUNE 2004)
A and D
• Butcher’s wart is caused by HPV-2 & HPV-7. It is seen
.........(PGI - 2000 - Dec)
in occupational handlers of meat, poultry or fish.
• Tuberculjds are lesions of skin or mucous membrane due
to allergic or toxic response of T.B. bacilli or protien
Lesions : Lichen scrofulosorum
— Papular-Necrotic tuberculids
— Acne agminata
— Rosea like lesions
— Erythema Nodosum
— Erythema induratum
• Lupus vulgaris is a localised skin lesion due to reinfection
of T.B.
• In Tuberculids — tuberculin test Negative and no bacilli • Lupus pernio is a particular type of sarcoidosis that involves
isolated from the lesions. the tip of the nose & the earlobes with lesions that are
violaceous in colour rather than red brown.
147. Cutaneous tuberculous secondary to under lying
tissue is called as:
A. Scrofuloderma
B. Lupus vulgaris
C. Tuberculous verrucosa cutis
D. Spina-ventosa
A
.........(AIPGMEE - 1999)

148. True about Lupus vulgaris:


• Erythema nodosum affecting the lower limbs in a
A. Apple jelly nodules at root of the nose
person with sarcoidosis
B. T.B. of skin and mucous membrane
C. Also known as scrofuloderma 152. Tuberculides are seen in
D. ATT helpful A. Lupus vulgaris
A,B and D B. Scrofuloderma
.........(PGI - DEC 2004) C. Lichen scrofulososum
D. Erythema nodosum
C
149. Mycobacterium causing skin ulcer:
.........(AIIMS PGMEE - MAY 2007)
A. M. Smegmatis
B. M. Scrofulaceum
C. M. Ulcerans
D. M. fortuitum
E. M. Marinum
C and E
.........(PGI - DEC 2002)

HELP LINE NO. 9391567707


TOPIC 9: ALOPECIA DERMATOLOGY 52

TOPIC 9: ALOPECIA • Alopecia areata can be classified according to its


pattern. Hair loss most often is localized and patchy
154. A male patient presents with patchy loss of hair on
scalp, eyebrows and beard . He also gives a history Patchy alopecia areata
of rapid greying of hair in a few areas. The likely
diagnosis is:
A. Alopecia areata
B. Anagen effluvium
C. Telogen effluvium
D. Androgenic alopecia
A
.........(AIPGMEE - 2008)

Single or multiple circumscribed smooth patchy hair loss, most


obviously on scalp but frequently involving any hair bearing
skin eg beard, eyebrows, eye lashes, with
pathognomic “exclamation mark ” & “going gray
overnight” phenomenon
• A reticular pattern occurs when hair loss is more
( i.e., white or gray hairs are frequently spared) indicate
extensive and the patches coalesce.
the diagnosis of alopecia areata.
• An ophiasis pattern occurs when the hair loss is
localized to the sides and lower back of the scalp

Features of Alopecia Areata:


• Onset of Alopecia is sudden
• Alopecia is well circumscribed and patchy (round or oval
patches of baldness)
• Lesions usually situated On scalp (Most common site) • Conversely, sisaipho (ophiasis spelled backwards)
• Lesions may involve beard of other hair bearing sites such pattern occurs when hair loss spares the sides and
as eyebrows or eyelashes back of the head (
• Skin within the areas of hair loss is normal Sisaipho pattern of alopecia areata
• Non-inflammed, Non scarring Alopecia
• Hair show the pathognomic ‘Exclamation mark’ sign
• Believed to be an Autoimmune condition

Alopecia areata affecting the beard.

Alopecia totalis

HELP LINE NO. 9391567707


TOPIC 9: ALOPECIA DERMATOLOGY 53

Diffuse alopecia areata 156. In alopecia areata, seen is:


A. Exclamatory mark hair
B. Scaring
C. Fungal infection
D. Traumatic
A
.........(PGI - 1997 - Dec), AIIMS PGMEE - NOV 2005

Exclamation mark is found in cases of Alopecia areata.


Alopecia areata is an autoimmune disorder of skin in which
there is perifollicular accumulation of lymphocytes
• Associated with other autoimmune diseases is seen & loss of hair in sharply defined areas of skin. It is
• A positive family history may be obtained. characterized by round or oval, circumscribed
• Rapid onset graying of hair may be observed ‘going gray patches of alopecia without a ny sign of
ovrenight’ inflammation, scarring or atrophy. In a patch where
• The mechanism of rapid graying is thought to be the the hair is tending to grow back, it is seen typically in the
selective shedding of pigmented hair and retention of shape of an exclamation mark. An exclamation mark
the non pigmented (already grey) hair. hair has a thin lower portion, a thick stump: this is
so, because the newly growing hair is thin & the portion
Corticosteroid injection of old hair which has brocken off is thick.

157. Growth phase of hair is:


A. Anagen
B. Metagen
C. Telogen
D. None
A
.........(PGI - 1997 - Dec), AIIMS PGMEE - MAY 2006

• Anagen phase is the growing phase of hair where hair


root is pigmented, papilla & inferior segment present.
• Telogen is the resting phase of hair & catagen is the
involutionary phase of hair.
• Cicatricial (scarring) alopecia (hair loss) is the term used
for a group of disorders that cause permanent hair loss.
During the active, evolving stage of hair loss, patches of
alopecia commonly appear red and inflamed at the base
of the hair shaft. Sometimes crops of pustules are noted

155. Which of the following is contraindicated in P osterior sca lp with areas of scarring alopecia and
Androgenic Alopecia: erythematous papules and plaques.
A. Testosterone
B. Cyproterone
C. Acetate Finasteride
D. Minoxidil
A
.........(AIPGMEE - 2000)

• Scarring Alopecia Localized areas of scarring alopecia of


the scalp may result from bacterial, viral and fungal
infections. More importantly, lesions of discoid lupus
erythematosus, lichen planus, and pseudopelade
present with this scarring picture.

HELP LINE NO. 9391567707


TOPIC 9: ALOPECIA DERMATOLOGY 54

• An accurate diagnosis is necessary to determine the the involvement of Mucosa ,Thinning of nails - strongly
treatment and biopsy is usually necessary to determine suggests the diagnosis of Lichen Planus
the cause. Evidence of cutaneous disease elsewhere on
the skin, oral or genital mucous membranes, and nails Alopecia
should be looked for carefully. Treatment of discoid lupus
erythematosus of the scalp includes intr alesional Scarring Non Scarring
corticosteroid and, if severe, antimalarials or retinoids 1. Lichen planus 1. Alopecia Areata

2. Cutaneous lupus 2. Endocrine related


Folliculitis decalvans: Pustules on a cicatrizing background. 3. Trauma
3. Linear Scleroderma
4. DLE 4. Tinea Capitis

5. Rare causes (Sarcoidosis &


metastasis)

• Nonscarring diffuse loss: Causes include male-pattern


baldness, female-pattern baldness, telogen effluvium,
anagen effluvium, primary hair shaft abnormalities, and
congenital disorders.
Discoid lupus of the scalp: Follicular hyperkeratosis, atrophy
and scarring present The hair cycle comprises the growth phase (anagen), the
brief transition phase (catagen), and the resting phase
(telogen).

Tufted folliculitis: This can occur after scarring alopecia of


any cause. Fibrosis of the follicles can cause tortional
changes so that more than one hair can grow from the • Telogen effluvium. Any great stress on the body can reset
follicle the hair cycle in a significant portion of the hairs.
• This results in partial hair shedding 2–3 months after the
stressor.

Follicultis decalvans in cicatrical alopecia: Keloids have


been left over much of the scalp.

A thinning across the frontal hair line is typical.


Labor and delivery caused the telogen effluvium in this patient.
Rapid weight loss is another common cause.

Lichenplanus
• Both Leprosy and Lichen planus can cause scarring alopecia

the involvement of
– Mucosa
– Thinning of nails • An anagen effluvium is extensive hair loss caused by
sudden profound disturbances to the matrix cells of the
= hair follicles. Rather than shedding, the hair is lost by
strongly suggests the diagnosis of Lichen Planus fracturing of the hair shafts at the level of the scalp.

HELP LINE NO. 9391567707


TOPIC 9: ALOPECIA DERMATOLOGY 55

Anagen Effluvium vs Telogen Effluvium


• There are multiple differences between an anagen
effluvium and a telogen effluvium.

• In an anagen effluvium, hair loss occurs because the


hair shafts are broken rather than shed. In contrast, the
ends of the hairs that are shed in a telogen effluvium
have a characteristic club shape with unpigmented
proximal ends.

• The hair loss in an anagen effluvium occurs within days or


• The two most common causes of anagen effluvium occur
from cancer chemotherapy and from radiation therapy. weeks of the injury to the follicle. Hair loss in a telogen
• The hair will regrow once the drugs are removed. effluvium typically occurs 3 to 4 months after the
systemic insult.

• Hair growth occurs in no other phase of the hair growth


cycle except in the anagen phase. The hair shaft is
generated by rapid mitotic cell divisions in the hair matrix
to produce hair fiber at a rate of 1 to 1.5cm (approximately
½ inch) per month on the scalp. An anagen effluvium
occurs if there is sufficient injury to the rapidly dividing • An anagen effluvium can involve up to 90% of the
keratinocytes in the hair matrix. The insult damages the hair on the head, whereas a telogen effluvium rarely
keratinocytes and diminishes the metabolic activity of the involves more than 50% of the hair on the head .
growing hair shaft. The stoppage of cell division results in • Since ~90% of the hair on the scalp is in the
growing phase , an anagen effluvium has the
a thin, weakened hair shaft that is susceptible to fracture
potential to cause almost complete alopecia.
with minimal trauma when it reaches the surface of the anagen effluvium
scalp.

• Hair breakage in an anagen effluvium occurs within days


to weeks (typically 1 to 3 weeks) following the insult to
the follicle.

• A close examination of the hair in anagen effluvium will


show that the scalp end of the hair shaft is dystrophic
with a rapidly tapering configuration (bayonet hair).

Telogen Effluvium: a Male-pattern alopecia (Non scarring endocrine )occurring in a


woman who presented with quite severe hirsutism of
Diffuse thinning over the entire scalp with widening of the
several months’ duration. This pattern of hair loss is
part everywhere on the scalp.
strikingly different from the typical female androgenetic
alopecia, and was due to an androgen-secreting
tumor.

HELP LINE NO. 9391567707


TOPIC 9: ALOPECIA DERMATOLOGY 56

Androgenetic alopecia in a woman.-Non scarring endocrine


Note the thinning across the top with preservation of the
hairline.

• Diffuse hair loss for 1 month with a history of enteric fever


before 4 months suggest the diagnosis of Telogen
effuvium.
Moth-eaten alopecia of secondary syphilis –Nonscarring focal
loss Clinical features of Telogen effuvium
• Varying degrees of diffuse hair loss occurring 2-3 months
after the precipitating stimulus

• Severe cases are associated with anemia with Beau’s


lines of the nails

158. Cicatrical alopecia is seen in:


A. Taenia-capitis
B. Psoriasis
C. Discoid lupus erythroamatosis
D. Alopesia-areata
C
.........(AIPGMEE - 1999), PGI - DEC 2004
PGI - DEC 2002, AIIMS PGMEE - NOV 2007

Non-scarring Alopecia is seen in : Scaring alopecia is seen


in :
1. Alopecia Arcata 1. Cutaneous Lupus
2. Tinea capitis 2. Lichen planus Etiology of Telogen effuvium
3. Traumatic alopecia 3. Linear sclerodenna • Every hair follicle passes through 3 phases (anagen,
catagen and Telogen) but adjoining hair follicles are not
In Systemic Lupus Etythematosis, there are two forms
synchronous in their cycle.
of Alopecia,
One is a scarring alopecia (this is seen secondary to
discoid lesions) and the other is non-scarring. • However sometimes the anagen (growing) phase of
the several adjoining hair follicles is aborted and
In Psoriasis, Scalp is involved in almost all cases. these follicles enter catagen (resting) phase at the same
However psoriasis of scalp never causes loss of hair and time and several hair are shed simultaneously.
baldness
Several factors can precipitate Telogen effuvium
164. 22 years women with diffuse hair loss for 1 month,
• Infections —» Typhoid, malaria, Dengue
with past history of enteric fever, before 4 months,
• Childbirth —» Especially prolonged and difficult
likely cause
A. Androgenic alopecia • Surgical trauma — Minor/major surgery
B. Telogen effuvium • Hemorrhage -> Surgical/traumatic
C. Anagen effuvium • Emotional stress — Examinations, marital discord
D. Alopecia areata Treatment
B • Hair fall stops simultaneously in 2-3 months, though hair
.........(AIIMS PGMEE - NOV 2007) growth which follows may be incomplete.

HELP LINE NO. 9391567707


TOPIC 10: SCABIES DERMATOLOGY 57

TOPIC 10: SCABIES Scabies

165. Papulovesicular lesions on face, trunk palm and sole Usually Involves In Infant Involves Nodular scabies
in a 9 month old child is seen in:
• Interdigital space • Involves Scrotum
A. Seborrhic dermatitis • Scalp, face
B. Atopic dermatitis • Anterior wrist • Neck
C. Scabies • Ulnar border of hand • Palm & Sole
D. Drug reaction
C
.........(AIIMS PGMEE - Dec - 1995)

• Papulovescicular lesions on face, trunk, palm and sole


in a 9 month old child Suggests the diagnosis of scabies.
• in infants scabies involves the face (It does not
involves face in Adults)
167. A child has multiple itchy papular lesions on the
• Atopic Dermatitis — genitalia and fingers .similar lesions are also seen in
– It can also present with Papulovesicular lesions on younger brother .Which of the following is most
face and trunk the main differentiating point b/w scabies possible diagnosis?
and atopic dermatitis is that in infant Atopic dermatitis A. Papular urticaria
does not involve Palms & Soles. B. Scabies
C. Atopic dermatitis
D. Allergic contact dermatitis
B
..........(AIIMS PGMEE NOV - 2002)

History and clinical features of the child points toward the


diagnosis of scabies. The points in favour of Scabies are-
1. Itchy papular lesions on genitalia and fingers.
• Similar lesions are also seen in atopic dermatitis but the
most common site of involvement is face.
2. Similar lesions are seen in younger brother.
• This point almost confirms the diagnosis of scabies.
• Scabies is a highly infectious disease and can involve other
members of the family. Other diseases given in the options
166. Nodular scabies is found in:
are not contagious.
A. Web space of finger
A burrow
B. Axilla
a small threadlike curvilinear papule that is virtually
C. Scrotum
pathognomonic of scabies
D. Abdomen
C

.........(AIIMS PGMEE - JUNE - 1997)

• Allergic contact dermatitis -


It develops after contact with an allergen. The patient
will always give h/o contact to an allergen

Scrotum
• Scabies is contagious disease caused by sarcoptis scabiei
hominis mites
• Usual sites are interdigital spaces, anterior aspect of wrist
and ulnar border of hand.

Papular urticaria-
It is a term used to describe a recurrent, disseminated
itchy papular eruption, due to either insect bite or hyper
sensitivity to them

HELP LINE NO. 9391567707


TOPIC 10: SCABIES DERMATOLOGY 58

• Atopic dermatitis-
It causes lesions similar to scabies but is non-contagious
disease and the most common site of involvement is face 169. A 6 month-old infant had itchy erythematous
papules and exudative lesions on the scalp, face,
groins and axillae for one month. She also had
vesicular lesions on the palms. The most likely
diagnosis is:
A. Congenital shypilis
B. Seborrheic dermatitis
C. Scabies
D. Psoriasis
C
.........(AIPGMEE - 2006)

168. Scabies, an infection of the skin caused by Sarcoptes Scabietic papules on the penile shaft and scrotum
scabie, is an example of:
A. Water borne disease
B. Water washed disease
C. Water based disease
D. Water related disease

.........(AIIMS PGMEE NOV - 2002)

Scabies mite scraped from a burrow


Widespread eruption on the back of an infant with scabies

• A typical linear burrow on the flexor forearm 170. An infant presents with itchy lesions over the groin
and prepuce;All of the following are indicated in this
patient except:
A. Bathe and apply scabicidal solution
B. Treatment should be extended to all family members
C. Dispose all clothes by burning
D. Start the patient on IV antibiotics
D
.........(AIPGMEE - 2001)

IV antibiotics need not be given; others are usually


done
This is most likely a case of scabies.
Erythematous papules and papulovesicles on the flexor wrist.
Scabies is caused by mite ‘sarcoptes scabei
• Main complaint is itching
• Most frequently occurs on anterior aspect ofwrisfi, ulnar
border of hand & interdigital webs, but penis, palms &
soles may be involved in infants as in this case.
• Lesion are vesicles & burrows : burrow is the pathognomic
lesion.
• Drug of choice here is Gammabenzene hexachloride
• Scabies on the penile shaft and glans

HELP LINE NO. 9391567707


TOPIC 10: SCABIES DERMATOLOGY 59

Other drugs used include : The particular agents used in the treatment of scabies include
Benzyl benzoate & sulphur ointment :
Permethrin 1. Benzyl benzoate: .. - Irritant to young children.
Monosulphorum & crotomiton 2. Lindane (gammaxene) : - Occasionally irritates.
3. Malathion
There is however no rationale of using IV Antibiotics. 4. Permethrin ; is a new effective agent
5. Monosulfiram : May cause antabuse (disulfiram) like
Nodular scabies in an infant alcohol reaction.
• Note that permethrin cream is less toxic than commonly
used lindane preparations, and is effective against lindane
tolerant infestations.
• a single oral dose of Ivermectin effectively treats
scabies in otherwise healthy persons.

173. Adult scabies is characterized by:


A. Involve palm and sole
171. An 8-month old child presented with itchy, B. Involve face
exudative lesions on the face, palms and soles. The C. Involve anterior abdomen
siblings also have similar complaints. The treatment D. Involve web spaces
of choice is such a patient is: E. Involve genitalia
A. Systemic ampicillin A,D and E
B. Topical betamethasone .........(PGI - DEC 2004)
C. Systemic prednisolone
D. Topical permethrin Seen on the head & neck in babies but rarely in adults.
D Scalp in ad ults who uses topical steroids for
.........(AIPGMEE - 2003) seborrhoeic dermatitis.

Crusted scabies Scabies in the interdigital web spaces

Scabies preparation demonstrating a mite and ova Papulovesicles and nodules on the palm in a patient with
scabies

In crusted scabies, sections show multiple mites (arrows)


within the hyperkeratotic stratum corneum. The epidermis
is spongiotic • In adults other than the elderly, burrows may occur on
the palms in women, but they are less frequently found
on the palms in men.

Scabies on the buttocks

172. Permethrin is used in the treatment of:


A. Scabies
B. Leprosy
C. Body louse 174. A 6-month old infant presented with multiple
D. Leishmaniasis papules and exudative lesions on the face, scalp,
A trunk and few vesicles on the palms and soles for 2
.........(AIPGMEE - 1999) weeks. His mother had history of itchy lesions. The
most likely diagnosis is:

HELP LINE NO. 9391567707


TOPIC 11: ATOPIC DERMATITIS DERMATOLOGY 60

A. Scabies B. Pururitis
B. Infantile eczema C. Morgagnian fold
C. Infantile seborrhoeic dermatitis D. Pityriasis alba
D. Impetigo contagiosa E. Dermographism
A A
.........(AIIMS PGMEE - MAY 2005) .........(PGI - DEC 2004)

• Vesicobullous lesion in infant can be seen in Diagnostic criteria of atopic dermatitis :


Scabies Major
Infantile eczema - An itchy skin condition
Seborrhic dermatitis
Minor
involvement of palms and soles— is a characteristic feature - onset below age of 2 years (not used if child is under 4
of scabies in infants years)
(or parental report of scratching)
Differential diagnosis - History of skin crease involvement of rubbing in a child)
(including cheeks in children under 10 years)
Scabies Infantile eczema - History of a generally dry skin.
Characteristic distribution: Lesions Spares palms and soles - Personal history of other atopic disease (or history
present on palms, soles and of any atopic disease in a first degree relative in children
genitalia under 4 years.)
Burrows present Absent - Visible flexural dermatitis (or dermatitis of cheeks /
Typical lesions in family member May have atopic diathesis in forehead & other limbs in children under 4 years
family
H/O asthma not relevant History of Asthma • Typical atopic dermatitis on the face of an infant
Infantile Seborrhic dermatitis Infantile eczema
Begin in infants <3 months In infants > 3 months
Asymptomatic Extremely itchy
Scalp, major flexures (axillae, Face, other parts of the body
groins) Family or personal history of
atopy.

175. Ivermectin in indicated in the treatment of:


A. Syphilis
B. Scabies
C. Tuberculosis
D. Dermatophytosis
B
.........(AIIMS PGMEE - MAY 2006) — To diagnose atopic dermatitis, at least one major plus three
or more minor criteria should be present.
• Ivermectin is the only drug effective orally in scabies Dermographism consists of local erythema due to capillary
and pediculosis. vasodilatation, followed by oedema & a surrounding flare
Single 0.2mg/kg orally cures most patients. due to axon reflex induced dilatation of arterioles.
• Ivermectin is basically an antihelminthic drug used
traditionally to treat onchocerciasis. Flexural involvement in childhood atopic dermatitis
• The mechanism of action of Ivermectin is by inhibition of
Glutamate gated chloride ion channels & other
ligand gated chloride channels, such as those gated
by the neurotransmitter gamma amino acid (GABA)
present in invertebrate nerve and muscle cells.
• This binding leads to an increase in the permeability of
the cell m embr ane to chloride ions with
hyperpolarization of the nerve or muscle cell, resulting
in paralysis and death of parasite.

Scabicides include: Dirty neck sign in chronic atopic dermatitis.


5% permethrin
25% Benzyl benzoate
1% Gamma Benzene hexachloride
10% Crotamiton Ivermectin

TOPIC 11: ATOPIC DERMATITIS

176. Minor clinical feature in diagnosis of atopic


dermatitis:
A. Dry skin Irritation around mouth of an infant with atopic dermatitis

HELP LINE NO. 9391567707


TOPIC 11: ATOPIC DERMATITIS DERMATOLOGY 61

177. Kaposi vericelliform lesion seen in


A. Atopic dermatitis
B. Darriers disease
C. Lichen planus
D. Varicella
A and B
.........(PGI - DEC 2005) • (b) Patch test, positive reaction. In this case, allergen
4 (paraphenylened iamine) reacted.
Paraphenylenediamene is a black dye and it is the only
patch that leaves a black color . In addition to the black
coloration, multiple vesicles and a surrounding erythema
are seen

178. Skin test can be done for which hypersensitivlty


reactions:
A. I
• In extreme sensitivity in Patch test , bullous test reactions
B. II
may occur even to standard test agents, in this case to
C. III
nickel.
D. IV
E. V
A and B
.........(PGI - June -2001)

179. Air-borne contact dermatitis can be diagnosed by:


A. Skin biopsy
B. Patch test
C. Prick test
D. Estimation of serum IgE levels
B
.........(AIIMS PGMEE - MAY 2006) Patch test allergens; Patch tests on the back; Patch test
results for contact dermatitis
• Air borne contact dermatitis is an allergic contact
dermatitis
• Patch test confirms the diagnosis of .allergic contact
dermatitis, as it detects a type IV hypersensitivity
reaction to exogenous allergens.
• Patch test is very simple technique which consists of • Most common allergen causing air-borne contact dermatitis
applying the suspected agent or its chemical ingredients in India - Parthenium hysterophorus (Congress grass)
in specified concentration on the normal looking skin on
the back of the patients, under separate patches for 48
hours and looking if this application produces dermatitis
on the area.
• After confirming the cause of contact dermatitis the
patient should be instructed to avoid contact with the
agent.
• Patch testing. Patch tests detect type IV reactions to
external allergens. Batteries of common allergens pertinent
to the clinical situation are applied to the back for
48 hours. In this case there were multiple reactions to
medicament agents including several topical steroids 180. A3 year old child has eczematous dermatitis on
extensor surfaces. His mother has a history of
Bronchial asthma. Diagnosis should be

HELP LINE NO. 9391567707


TOPIC 11: ATOPIC DERMATITIS DERMATOLOGY 62

A. Atopic dermatitis severe in the rainy season and improved completely


B. Contapt dermatitis in winter. The most likely diagnosis is
C. Seborrhic dermatitis A. Insect bite hypersensitivity
D. Infantile eczematous dermatitis B. Scabies
A C. Urticaria
.........(AIIMS PGMEE - MAY 2007), AIPGMEE - 2007 D. Atopic dermatitis
A
181. Most common cause of plant induced dermatitis in .........(AIIMS PGMEE - MAY 2004)
India:
A. Poison ivy Atopic dermatitis -
B. Parthenium The following points rule out atopic dermatitis.
C. Ragweed • Atopic dermatitis worsen in winters
D. Cotton fibres • Age group —
B Atopic dermatitis first appears in infants and then it
.........(AIIMS PGMEE - MAY 2008) continues in the childhood and adult phase. The lesions
AIIMS PGMEE - DEC 1994 become more and more lichenified.
AIPGMEE - 1999 It is rare for atopic dermatitis to present at the age of 27
years with H/O 3 years. The patient will give h/o the
“Most common cause of plant induced dermatitis is disease from infancy.
parthenium.”
Parthenium hysterophorus (congress grass, congress
weed carrot weed, wild fever few, the “scourge of India”)
• It is an exotic weed that was accidently introduced in
India in 1956 through imported food grains.
• It has become a common weed causing dermatitis of
epidemic proportions. c
• The epithet “congress weed” refers to the U.S. congress
(who allocated the shipment for Pune, India).
• In Pune it found an ecological niche without natural
enemies and spread rapidly along the canal banks, roads
and railway tracks to become a major field weed. Urticaria -
Presence of itchy excoriated papules rules out the diagnosis
182. A 9-year-old boy has multiple itchy erythematosus of urticaria.
wheals all over the body for 2 days. There is no The urticarial lesions are itchy erythematous papules.
respiratory difficulty. Which is the best treatment?
A. Antihelminthics
B. Systemic corticosteroids
C. Antihistamines
D. Adrenaline
C
.........(AIIMS PGMEE - MAY 2004)

• Multiple itchy erythematous wheals all over the body


indicate urticaria.
• T/T of urticaria -
Scabies •
The basic pathogenesis in the case of urticaria is release
Presence of lesions on the face rules out the diagnosis of
ofhistamine.
scabies. In scabies in adults lesions are never present on
face. So we are left with insect bite hypersensitivity. The
Acute urticaria is a self limiting disease and it requires
chronicity and seasonal variation can be attributed to insect
only antihistamine for tit.
bite in rainy season every year.
The antihistamines commonly used are –
• Hydroxyzine
184. A 55-year-old male, with uncontrolled diabetes
• Diphenhydramine (sedating antihistamine)
mellitus and hypertension, developed severe air-borne
contact dermatitis. The most appropriate drug for
• Loratadine
his treatment would be
• Fexofenadine (non sedating antihistamine)
A. Systemic corticosteroids
• Certrizine
B. Thalidomide
C. Azathioprine
For rapid relief of acute severe urticaria with laryngopharyngeal
D. Cyclosporine
edema .5 to lml of epinephrine should
C
A short burst of cortcosteroids is reserved for very
.........(AIIMS PGMEE - MAY 2004)
severe episodes of urticaria.
The drug of choice for allergic contact dermatitis is
183. Rakesh, a 27-year-old man had itchy, excoriated
corticost eroid, but corticos teroids are
papules on the forehead and the exposed parts of
conraindicated in diabetics and hypertensives. So we
the arms and legs for 3 years. The disease was most
have to use a corticosteroid sparing agent in this patient.

HELP LINE NO. 9391567707


TOPIC 12: CONTACT DERMATITIS DERMATOLOGY 63

Azathioprine is a corticosteroid sparing agent which is used


in patients with allergic contact dermatitis. It does not
have any serious adverse effects.
Hypertension is a common adverse effect of cyclosporine,
hence it cannot be used in this patient.

185. A 3 year old child has eczematous dermatitis on


extensor surfaces. His mother has a history of
Bronchial asthma. Diagnosis should be
A. Atopic dermatitis Nickel dermatitis from contact with nickel plated paint spray
B. Contact dermatitis gun.
C. Seborrhic dermatitis
D. Infantile eczematous dermatitis
A
.........(AIIMS PGMEE - NOV 2006)

TOPIC 12: CONTACT DERMATITIS

187. Most Common cause of allergic contact dermatitis


in Indian females is: Hand dermatitis from sticking plaster (left) and shoes (right).
A. Vegetables
B. Nail polish remover
C. Nail polish
D. Detergents
D
.........(AIIMS PGMEE JUNE - 2000)
• Garlic allergy. Allergy to foods may cause a hand dermatitis.
Detergents Here the offending food was garlic
• Chemicals causing allergic contact dermatitis:
– Metals, especially Nickel (MC) and chromium
– Rubber
– Organic dyes
– Plastics and resins
– Preservative in ointment & cosmetics
– Plants, especially Primula
– Drugs applied topically

• Contact allergic dermatitis due to eyedrop preservatives

Perfume and medicament allergy.

Contact dermatitis due to cosmetic sensitivity (left) and plants


(right)

• Neomycin allergy. (a) Allergic contact dermatitis to


neomycin in eardrops is not uncommon.

• Metals, especially nickel and chromium occur in traces


in Jewellery but these are found also in less likely places
such as detergent (Ni) and cement (cr)
• Nickel dermatitis from contact with jeans studs
(left) and necklace (right)

HELP LINE NO. 9391567707


TOPIC 12: CONTACT DERMATITIS DERMATOLOGY 64

This woman applied neomycin, hoping to cure a rash about winters.The most appropriate test to diagnose the
the mouth. Instead, a rash ten times worse developed condition would be:
A. Skin biopsy
B. Estimation of IgE levels in blood
C. Patch test
D. Intradermal prick test
C
.........(AIPGMEE - 2006)

190. A 25 year old man presents with recurrent episodes


of flexural eczema, contact urticaria, recurrent skin
• Hydrocortisone allergy. Surprisingly enough, allergic contact infections and severe abdominal cramps and
dermatitis may occur from topically applied topical steroids. diarrhoea upon taking sea foods. He is suffering from:
A. Seborrheic dermatitis
B. Atopic dermatitis
C. Airborne contact dermatitis
D. Nummular dermatitis
B
.........(AIPGMEE - 2004)

Food allergens exacerbates skin rashes in atleast a subset of


patients with atopic dermatitis and characteristic ‘-’ clinical
features may be seen following oral challenge with
certain foods (eg. seafood) in sensitive individuals .
Atopic dermatitis
Atopic dermatitis (AD) is the cutaneous expression of the
atopic state, characterized by a fimily History of asthma,
Hand dermatitis from a detergent hay fever or dermatitis in up to 70% of patients. It is
known as “an itch that rashes”.

Dennys Morgan line: an extra fold of skin beneath lower


eyelid

192. Patch test is read after:


A. 2 hours
B. 2 day
• Erythema multiformelike reaction that developed C. 2 weeks
D. 4 weeks
acutely following hair dying
B
.........(AIPGMEE - 1999)

Patch test:
• Is designed to document sensitivity to a specific antigen.
• It is done to confirm disorder of skin resulting from a delayed
hypersensitivity type reaction. \
e.g. Contact dermatitis.
• In this procedure, a series of suspected contact allergens
are applied to the patient’s back with adhesive tape, and
are allowed to Remain in contact with the skin.
• Result is read after 48 hours,2 when dressings are removed
and the area is examined for evidence of delayed
hypersensitivity reactions (erythema, edema, or papulo-
Allergic contact dermatitis to nickel in a necklace.
vesicles).

193. Atopic dermatitis is diagnosed by:


A. Patch test
B. Clinical examination
C. Wood’s lamp
D. ­ lgE
B
.........(AIPGMEE - 1999)

188. A 45-year-old farmer has itchy erythematous Atopsy is a genetically determined disorder in which there is
popular lesions on face, neck, ‘V area of chest, dorsum an:
of hands and forearms for 3 years. The lesions are - Increased liability to form reagin antibodies (IgE).
more severe in summers and improve by 75% in - Increased susceptibility to certain diseases like asthma,

HELP LINE NO. 9391567707


TOPIC 13: LGV DERMATOLOGY 65

hay fever, dermatitis. 196. Genital elephantiasis is caused by:


Atopic dermatitis : is a very common, extremely itchyQ disorder A. Donovanosis
of unknown cause that charachterstically B. Congenital syphilis
but not invariably affects the face and flexures of infants, C. Herpes genaitalis
children, adolescents and young adults. D. Lymphogranuloma venerum
Diagnosis is essentially clinical D
.........(AIPGMEE - 2002)
TOPIC 13: LGV
• Late complication of LGV is Genital elephantiasis due
194. Lymphogranuloma venerum is caused by: to lymphatic involvement.
A. Chlamydia trachomatis A small percentage of cases of LGV in men present
B. Hemophillus ducrei as chronic progressive , infiltrative, ulcerative or fistular
C. Human T cell lymphoma virus type II lesions of the penis, urethra or scrotum. Associated
D. Donovania granulomatis lymphatic obstruction may produce elephantiasis.
A • Pseudo elephantiasis is found in Donovanosis.
.........(AIIMS PGMEE NOV - 1999)
• The tertiary stage of the LGV is termed genitoanorectal
syndrome. Women are more likely to present in this stage.
Symptoms include fever, pain, tenesmus, pruritus, and
purulent or bloody diarrhea. These symptoms are associated
with proctocolitis, abscesses, and fistulas.

• Lymphogranuloma venerum is caused by Chlamydia


trachomatis type I, II and III.
• Other frequently asked questions
Disease Organism
• Chancroid(soft chancre) Hemophylus ducrei
• Granuloma inguinale 197. Bubos with multiple sinuses discharging into
• Chancre Calymatobacter granulomatis, inguinal lymph nodes are seen in
• LGV T . pallidum A. Chancroid
Chlamydia trachomatis type I, II and B. Granuloma Inguinale
III C. LGV
D. Syphilis
195. A 30 year old male patient has a large, spreading
and exuberant ulcer with bright red granulation C
tissue over the glands penis. T here was no .........(AIPGMEE - 1994)
lymphadenop athy. The most likel y causative
organism is;
A. Treponema pallidum
B. Herpes simplex virus type 1
C. Herpes simplex virus type 2
D. Calymmatobacterium granulomatis
D
.........(AIIMS PGMEE NOV - 2003)
Lymphogranuloma Venerum (LGV) is a sexually transmitted
Bright red painless ulcer with no lymphadenopathy suggest disease caused by chlamydia trachomatis (serotypes LI,
the diagno sis of Donov anosis (caused by LII & LIII) and involves primarily the lymphatic system.
Calymmatobacterium granulomatis).
• The most characteristic feature of the disease is inguinal
Features of genital ulcer due to Donovanosis
lymphadenopathy, which begins 1-6 weeks after the
• The early primary lesion is a papule
primary genital lesion. Patient presents with a painful,
• The no. of lesions can be variable
tender swelling in the groin.
• The edges are elevated and irregular
• The lesion is red and velvety and bleeds readily The swelling is made up of multiple lymphnodes which
• There is firm induration and pain is uncommon are initially discrete, but later become matted to form a
• Absence of lymphadenopathy, pseudobuboes are seen firm, elongated sausage shaped, fixed mass - the inguinal
Diagnosis - It is made by demonstration of Donovan bubo.
bodies’” within large mononuclear cells. • This is most commonly unilaterally and more common in
males.
T/T->
Doxycycline — Drug of choice in developed countries • The nodes get attached to overlying skin and to deeper
Erythromycin — used as an alternative in pregnant women tissues. Suppuration occurs in the nodes with formation
Chloramphemicol —» Drug of choice in some developing of multiple abscess which may rupture producing multiple
countries. discharging sinuses.
For the differential diagnosis of genital ulcers

HELP LINE NO. 9391567707


TOPIC 14: CHANCROID TOPIC 15: DERMATITIS HERPETIFORMIS DERMATOLOGY 66

• Enlargement of nodes both above and below the inguinal A. Herpes genitalis
ligament may produce a groove in the bubo – sign B. LGV
of groove. (This often thought to be pathognomic of C. Primary chancre
L.G.V.) D. Chancroid
D
.........(AIPGMEE - 2008)

198. Lymphadenopathy is seen is A/E


A. Syphlis 1st Stage
B. Donovanosis
C. LGV 201. Chancroid may be caused by:
D. Chancroid A. T.Pallidium
B B. G.donovani
.........(AIPGMEE - 1995) C. Chlamydiatrachomatis
D. Haemophilus ducreyi
Gr anula in guinale or donov ano sis is a chro nic D
granulomatous bacterial infection caused by a gram .........(AIPGMEE - 1999)
negative bacterium ‘Calymmato bacterium granulomatis’.
There is no lymph node involvement, however lesion in 202. 20 year old male develops multiple ulcer over
inguinal region simulates lymphadenopathy. -Pseudobubo prepuce and glans, which are painful along with
(Behl) suppurative inguinal lymphadenopathy, 5 days after
having sexual intercourse with a lady, most probable
199. Frie test is done in diagnosis is:
A. Donovanosis A. Molluscum contagiosum
B. LGV B. Anaerobic streptococci
C. Syphillis C. Herpes
D. Leprosy D. Chancroid
B D
.........(AIPGMEE - 1996) .........(AIIMS PGMEE - SEP 1996)

• Frie’s test is an intradermal test for demonstration of


hypersensitivity to chlamydial antigens, the causative
agents of LGV.
• The crude chlamydial antigen obtained from bubo pus or
cultures is inoculated intradermaly in the forearm with a
control on the other side.
• Induration of 7 mm or more in 2 - 5 days is considered
positive.
• Due to frequent false positive reactions this test is not
recommended now.

Features of LGV

A Asymptomatic lesion
B Bubo 203. Painful lymphadenopathy is seen In
C Chlamydia trachomatis is causative A. Donovanosis
agent B. Syphilis
D Doxy Cycline is DOC C. Chancroid
D. Herpes simplex
E Estheiomine
E. Gonorrhaea
F Frei'sT est C and D
G Groove's Sign .........(PGI - DEC 2002)
TOPIC 14: CHANCROID
TOPIC 15: DERMATITIS HERPETIFORMIS
200. A hetero sexual male presents with multiple non -
204. Drug of choice for dermatitis herpetifonnis is:
indurated painful ulcers with undermined edges and
A. Dapsone
enlarged lymph nodes 5 days after exposure. Most
B. Griseofulvin
likely diagnosis is

HELP LINE NO. 9391567707


TOPIC 15: DERMATITIS HERPETIFORMIS DERMATOLOGY 67

C. Rifampicin
D. Ketoconazole
A
.........(AIIMS PGMEE - JUNE - 1997), AIIMS PGMEE - MAY
2005

• The drug of choice for Dermatitis herpetiforms is —


Dapsone (50-200 mg/day)
• Dermatitis herpetiformis is a bullous lesion in which there
is Gluten Hypersensitivity.

Treatment —
• Dapsone
• Gluten free diet
Immunofluorescence showing immunoglobulin A at the
Vitamins & minerals
dermoepidermal junction (direct immunofluorescence
stain).
205. A 28 y ear old pt. has multi ple grouped
papilovesi cular lesions on bo th ELBOWS,
knees,buttocks and upper back associated with
severe itching. The most likely diagnosis is:
A. Pemphigus vulgaris
B. Bullous pemphigoid
C. Dermatitis herpetiformis
D. Herpes zoster
C
.........(AIIMS PGMEE NOV - 2002)

Light micrograph shows neutrophils in the dermal papillae,


with fibrin deposition, neutrophil fragments, and edema
206. A 30 year old male had severely itchy papula-
vesicular lesions on extremities, knees, elbows and
buttocks for one year. Direct immunofluorescence
staining of the lesions showed IgA deposition at
dermoepidermal junction. The most probable
diagnosis is:
A. Pemphigus vulgaris
B. Bullous pemphigoid
C. Dermatitis herpetiformis
Papillary microabscesses form and progress to subepidermal D. Nummular eczema
vacuolization and vesicle formation in the lamina lucida, C
the weakest portion of the dermoepidermal junction .........(AIPGMEE - 2004)

Dermatitis herpetiformis
an immunologic vesicular disease
characterized by the presence of
IgA immunocomplexes
type II immunocomplexes reaction
location at the tips of the dermal papilla
producing subepidermal vesicles with neutrophils
has a strong associated with celiac disease.

Classic vesicles of dermatitis herpetiformis. Dermatitis herpetiformis

HELP LINE NO. 9391567707


TOPIC 16: PITYRIASIS ROSEA DERMATOLOGY 68

DH on buttocks

DH on face 207. What can pt with gluten sensitive hypersensitivity


consume as food:
A. Rice
B. Barley
C. Oat
D. Corn
E. Rye
A
.........(PGI - DEC 2006)

• Diet restrictions in gluten sensitive enteropathy :


- Wheat
- barley
DH on chest - rye
- in smaller amounts oats.
• Rice is safe.

TOPIC 16: PITYRIASIS ROSEA

209. The following statement is true for Pityriasis Rosea:


A. Self limiting
B. Chronic relapsing
C. Life threatening infection
D. Caused by dermatophytes
A
.........(AIPGMEE - 2007), AIIMS PGMEE - NOV 2006
DH on the top of the thigh
210. A 16-year-old boy presented with
asymptomatic,multiple, erythematous, annular
lesions with a collarette of scales at the periphery of
the lesions present on the trunk. The most likely
diagnosis is:
A. Pityriasis versicolor.
B. Pityriasis alba
C. Pityriasis rosea
D. Pityriasis rubra pilaris
C
..........(AIPGMEE - 2005), AIPGMEE - 2002

DH Pityriasis Rosae is a self limiting populosquamous disorder :


Immune complexes at the tips of the dermal papilla • The etiology is unknown, a virus has often been
subepidermal vesicles with neutrophils incriminated.
Dermatitis herpetiformis
Clinical features
• Usually occurs between the ages of 10-35 years
• The incidence is lowest in summer.
• Mildly itchy rash, usually on the trunk.

IgA granular basement membrane zone with stippling


in dermal papillae

HELP LINE NO. 9391567707


TOPIC 16: PITYRIASIS ROSEA DERMATOLOGY 69

Morphology of the lesions Pityriasis Rosea - inverse distribution on arms


Herald patch Children with dark skin often present with an inverse
• This is the first lesion to be seen, present in about 80% distribution
of the patients with pityriasis Rosae. of lesions in pityriasis rosea.
• The lesion is oval or round with central wrinkled salmon
coloured area and darker edge. There is & collarette of Rather than appearing on the trunk,
scales, which are attached just within leading edge and =
free towards the centre. they primarily are distributed on the arms
and legs

Herald patch of P.rosea

Inverse form of pityriasis rosea, with peripheral distribution

Herald patch with collarette of scale at the margin.


Typical oblong trunk lesions of pityriasis rosea.

Pityriasis rosea
Characteristic FEATURES Classic pityriasis rosea of the lower abdomen with associated
oval shaped plaques herald patch
of confluent scaling papules, distributed along the
dermatomes

lesions may be more difficult to see in children with darker


skin
pigmentation and as noted earlier,
=often have a
more papulofollicular appearance.

In this child, the large herald patch is very prominent.

salmon-colored maculopapular lesions.


The most striking feature is the arrangement of the lesions
such that the long axis is parallel to the lines of
cleavage.
Lesions aligning along Langer’s lines.
“Christmas tree” distribution.

Secondary lesions
• They begin as scaly papules and enlarge to form annular

HELP LINE NO. 9391567707


TOPIC 16: PITYRIASIS ROSEA DERMATOLOGY 70

plaques but secondary lesions are smaller, less scaly and


less erythematous than herald patch.

Sites of prediliction
• Mainly seen on the trunk.

• The configuration of the lesion of the pityriasis


Rosae is often suggestive of the diagnosis, the
longitudinal axis of the patches runs downwards and Papulosquamous leisions in Pytyriasis rosea
outwards from the spine, along the lines of the ribs.
This appearance has been referred to as the “fir tree” or
“christmas tree” appearance.
• Sometimes in (20% patients ) lesions occ ur
predominantly on extremities and neck (inverse
pattern).

Evolution of Pitvriasis
Herald patch Treatment
↓ • The disease is self limiting, so only symptomatic tit
(Phase of evolution) 7-10 days is required
- Itchy lesions — Antihistamines + lactocalamine, mild
Many similar lesions which emerge slowly
to moderate topical steroids
↓ - Very scaly/erythematous lesions — Coal tar in
(Phase of resolution) 2-10 weeks petroleum
Mild pigmentary changes - Recalcitrant lesions — Sunlight PUVA

Fades slowly 213. Which of the following statements is true regarding
Pityriasis rosea Pityriasis Rubra Pilaris:
Pityriasis A. Isolated patches of normal skin are found
B. Cephalocaudal distribution
Predominant trunk Predominant face & scalp C. I.V. cyclosporine is effective
Involvement involvement with late D. More common in females
involvement of whole body E. Oral cyclosporine is effective
↓ ↓
↓ A
• Brown, scaly macules • Erythematous • Orange hue Erythema .........(PGI - 2001 - Dec)
• more common in annular lesions with • Thickened palmswith
cushing’s syndrome peripheral fine bran
yellowish discolouration • Pityriasis Rubra pilaris is a chronic cutaneous disorder
like scales
• Follicular eruption characterised by
↓ (i) Follicular, conical pinkish papules covered with scales or
↓ ↓
P. rubra pilaris horny plugs with hair curled on top usually found on the
P. versicolor P. rosea
back of fingers and hands.
(ii) Later generalised erythema and scaling develop.
Eruption rarely becomes erythematous.
• The fir tree pattern of oval lesions along dermotonal lines.
Trunk, Neck and extremities are commonly involved,
• Generalised but characteristic ‘skip’ areas of normal skin,
• Wax-like keratoderma

• Pityriasis rubra pilaris.


– Confluent orange-colored palmar hyperkeratosis.

Pityriasis rosea in the groin. The condition prefers sun-


protected areas

HELP LINE NO. 9391567707


TOPIC 17: TINEA VERSICOLOR DERMATOLOGY 71

Pityriasis rubra pilaris Pityriasis rubra pilaris


‘Islands of sparing’.. Confluent lesions of acute onset in a child.
• g) Confluent and follicular pityriasis rubra pilaris.

Pityriasis rubra pilaris


• Psoriasiform lesions

Pityriasis rubra pilaris


The Koebner reaction also occurs in this condition, as
shown here in a teenaged patient

Pityriasis rubra pilaris


Follicular papules.

Rx.
Isotretinoin or acetretin
Methotrexate
Pityriasis rubra pilaris Cyclosporine is used for transplant rejection to suppress
Localized PRP in a child immunity.
• Skin biopsy is required for diagnosis of pityriasis rubra
pilaris.

TOPIC 17 : TINEA VERSICOLOR

214. All of the following drugs are effective in the


treatment of pityriasis versicolor except:
A. Selenium sulphide.
Pityriasis rubra pilaris B. Ketoconazole.
Confluent lesions of acute onset in a child. C. Griseofulvin.
D. Clotrimazole
C ..........(AIPGMEE - 2005)

215. A 10 year old child has hypopigmented, multiple


scaly patch over face. The likely diagnosis is:
A. Pityriasis alba
B. Tinea versicolor
C. Leprosy
D. Indeterminate leprosy
A .........(AIIMS PGMEE JUNE - 1999)

216. An adult presents with oval scaly hypopigmented


macule over the chest and the back. The diagnosis
is:

HELP LINE NO. 9391567707


TOPIC 18: PITYRIASIS ALBA DERMATOLOGY 72

A. Leprosy
B. Lupus vulgaris
C. Pityriasis versicolor
D. Lichen Planus
C
.........(AIIMS PGMEE MAY - 2001)

217. The following drug is indicated in the treatment of


pityriasis versicolar:
A. Ketoconazole.
B. Metronidazole.
C. Griseofulvin.
D. Chloroquine
Hypopigmented Scaly lesions on check in child has clinched
A the diagnosis in favour of pityriasis alba.
..........(AIIMS PGMEE MAY - 2003), AIPGMEE - 2002

Treatment of Pitvriasis Versico lor:


• Topical Imidazole cream & shampoo {Ketoconazole,
Miconazole, Clotrimazole, Econazole, Itraconazole).
• Whitfield’s ointment (3% salicylic acid + 6% Benzoic
Acid)
• Selenium disulphide & Sodium trisulphate shampoo.

219. Wood’s lamp used in sis of:


A. P. Versicolor
B. Vitiligo
C. Porphyria 221. A 5 year boy has multiple asymptomatic oval and
D. Psoriasis circular faintly hypopigmented macules with fine
E. Lichen planus scaling on his face. The most probable clinical
A,B and C diagnosis is:
.........(PGI - DEC 2006) A. Ptyriasis versicolor
B. Indeterminate leprosy
C. Ptyriasis alba
• Wood’s light is a source of UV light from which virtually all
D. Acrofacial vitiligo
variable rays have been excluded by a wood’s (nickel oxide)
C
filter. Its uses are :
.........(AIPGMEE - 2003)
- Fungal infections like Tinea capitis-green fluorescence ; AIIMS PGMEE - DEC 1998, AIIMS PGMEE - SEP 1996
Pityriasts versicolor -yellow.
- Bacterial infections like Erythrasma, acne-coral pink. P . alba (P . simplex) P . Versicolour Indeterminate
Pseudomonas pyocyanea-yellow green. leprosy
- Infestation-scabies, porphyria. • Primarily in young children • Rare in children but • more common in
- Pigmentary disorders : - Vitiligo before the age of puberty common in young children
children
- Detection of Ash leaf macules in tuberous sclerosis.
• Macules usually located on • Macules appear on
- Drugs and chemicals : - Detection of tetracycline/ cheeks but may present any upper trunk &
Mepacrine in tissues. whese on face shoulder face
- Detection of fluorescent contact or photosensitizers on involvement is rare
Hands & Lower limbs
the skin. are usually not involved
- Tumours : Sq. cell Ca of skin-red fluorescence. • More frequent in children in • Systemic steroids • More frequent in
- Miscellaneous: Lipofuscins in sweat from patients with hill stations aggravate lesion patient belonging to
high leprosy
chromhidrosis. prevelence state as
Tamil Nadu
TOPIC 18: PITYRIASIS ALBA • Reccurrent • Recurrent
• Scaly lesion • Scaly lesion • Non scaly lesions
220. Babloo,5 years, Presents with small hypopigmented • Non anaesthetic • Non anaesthetic • Anaesthetic/ Non
scaly macules on his cheek. Some of his classmates anaesthetic lesion
also have similar lesion. Which of the following is the
most probable diagnosis: 222. True about pltrylasis alba:
A. Pityriasis rosea A. No active treatment required
B. Pityriasis versicolor B. Common in elderly
C. Indeterminate leprosy C. Variant of vitiligo
D. Pityriasis alba D. Common over the face
E. Presents as scaly, whitish macules
D
A,D and E
.........(AIIMS PGMEE NOV - 2000)
.........(PGI - June -2001)

HELP LINE NO. 9391567707


TOPIC 19: TUBEROUS SCLEROSIS DERMATOLOGY 73

TOPIC 19: TUBEROUS SCLEROSIS

225. Babloo, a 4 year old boy, presents with history of


seizures, on examination there is hypopigmented
patches on his face. There is mental retardation also.
What is most likely diagnosis:
A. Neurofibromatosis
B. Tuberous sclerosis
C. Sturge weber syndrome
D. Incontinentia Pigmenti
B
• Sturge weber Hyperpigmented
.........(AIIMS PGMEE NOV - 2000)
syndrome capillary Angiomas
Neuro cutaneous disorders can cause seizures and mental involving upper face.
retardation

Only Tuberous sclerosis gives hypopigmented lesions on


skin.
All the other neurocutaneous disorder mentioned in
question cause hyper pigmentation.
Conditions Skin Lesions
• Tuberous Sclerosis • Ash leaf shaped
hypopigmented patch
• Shagreen patch
• SWS -Typically the patient presents at birth with a
• Adenoma Sebaceum “Port Wine Nevus” - a reddish-brown or pink
discoloration of the face, often following the
distribution of the trigeminal nerve. Intracranially,
ipsilateral to the facial nevus, there is abnormal
circulation that leads to 1) cerebral dysfunction; 2)
electrical instability (seizures); and 3) cerebral
cortical atrophy. Seizures usually present within the
first two years of life. Typically the occipital lobes
are affected first, and most severely, but the disease
may also involve the parietal and temporal lobes,
and (rarely) the frontal lobe.

Shagreen patch

• Port wine stain is a vascular birthmark caused by


abnormal development of blood vessels in the skin. It can
occur anywhere on the body, but in SWS it affects the
skin around the forehead and/or scalp.
• As well as the port wine stain affecting the skin, it will
also involve an extra layer of blood vessels over the surface
Adenoma Sebaceum of the brain (angioma).
• The angioma may be associated with seizures (fits
or convulsions) and developmental problems.

• SEGA –Subependymal giant cell astrocytoma in a 16-


year-old boy with a long history of adenoma sebaceum,
seizures, and psychomotor developmental delay

HELP LINE NO. 9391567707


TOPIC 19: TUBEROUS SCLEROSIS DERMATOLOGY 74

Port wine stain – Like dermatomes, they are linear on the limbs and
circumferential on the trunk.

– Unlike dermatomes, Blaschko’s lines do not correspond to


innervation patterns or spinal cord levels

• Neurofibromatosis • Cafe au lait patches


(Flat light brown macules all over
skin surface, trunk in particular and
in axilla)

Incontinenti pigmentosa -Blaschko’s lines


IP in an affected female with Stage III “rash”

• Cafe au lait patch

An adult with reticulated pigmentation patterns

• Incontinenta • Only girls present with this


pigmenti (It is a X linked disorder • Initially there are
dominant condition It is widespread blistering eruptions
usually lethal in utero in which are replaced by
males) Hyperpigmentation.
226. A patient had seven irregular hyperpigmented
macules on the trunk and mult iple small
hyperpigmented macules in the axillae and groins
since early childhooD) There were no other skin
lesions. Which is the most likely investigation to
support the diagnosis?
A. Slit-lamp examination of eye
B. Measurement of intraocular tension
C. Examination of fundus
D. Retinal artery angiography
A
.........(AIPGMEE - 2006)

• IP manifests in stages that evolve sequentially . 227. All of the following are seen in Tuberous Sclerosis,
– The onset and duration of each stage vary among except;
individuals, and not all individuals experience all four stages. A. IRIS Nodule
– The skin abnormalities that define each stage occur along B. Renal Cortical Cyst
lines of embryonic and fetal skin development known as C. Rhabdomyoma of heart and lung
Blaschko’s lines Blaschko’s lines correspond with cell D. Adnoma Sebaceum
migration or growth pathways that are established during A
embryogenesis. .........(AIPGMEE - 2000)

HELP LINE NO. 9391567707


MINOR TOPICS DERMATOLOGY 75

Iris Nodule is a feature of Neurofibromatosis and not tuberous


sclerosis

TOPIC 21: ACANTHOSIS NIGRICANS


Tuberous sclerosis is characterized by:
Skin (Cutaneous) Involvement 229. Underlying internal malignancy is not shown by
A. Acanthosis nigricans & Annular erythema
• Adenoma sebaceum
B. Bullous pyoderma & migratory necrotIzing erythema
• Facial angiofibroma.
C. Granuloma annulare
• Ash-leaf shaped hypopigmented
D. Erythema gyratum repens
macules C
• Shagreen patches .yellowish .........(AIPGMEE - 1996)
thickenning of skin over Lumbosacral region of back.
• Depigmented nevi • Granuloma annulare is a chronic skin disease
consisting of a rash with reddish bumps arranged
Neurological Features in a circle or ring . Granuloma annulare is different from
• Epilepsy, Seizures warts, and cryotherapy treatment will not work. It most
• Mental retardation Q often affects children, young and older adults and it is
• Hydrocephalus Q slightly more common in females (60/40 ratio).
• Presence of subependymal nodules with tram track • Causes
calcification on neuroimaging is characteristic. • The condition is usually seen in otherwise healthy people.
Occasionally, it may be associated with diabetes or
Tram track calcification –Sturge weber syndrome thyroid disease.

Granuloma annulare is associated with Diabetes millitus not


with internal malignancy. All other conditions mentioned
Benign Neoplasms are associated with malignancy.
Rhabdomyoma of myocardium
• Angiomyoma of kidney/liver/ pancreas/adrenals.
Ependymomas and childhood Subependymal giant cell
Astrocytoma.

Nonobstructive ventricular rhabdomyomas in a patient with


tuberous sclerosis.

Glucagonoma syndrome — necrolytic migratory erythema


• alpha cell tumor of the pancreas; occasionally no neoplasm
found
• abdomen, thighs and buttocks
• patchy erythema with flaccid vesicles and bullae
• glossitis, angular cheilitis, normocytic anemia, low amino
acid levels in serum

228. ‘Adenoma sebaceum’ is a feature of -


A. Neurofibromatosis
B. Tuberous sclerosis
C. Xanthomatosis
D. Incontinentia pigmenti
B
.........(AIIMS PGMEE - NOV 2005)

HELP LINE NO. 9391567707


MINOR TOPICS DERMATOLOGY 76

migratory necrotIzing erythema in Glucaganoma Acanthosis nigricans (AN)


• velvety thickeni ng and darkening
(hyperpigmentation) of the skin, especially on the
nape of the neck, axillae and other body folds

• underlying causes may be hereditary or acquired, and


• Erythema gyratum repens as presenting sign of include:
squamous cell carcinoma of lung in smoker • obesity; drugs; “malignant” acanthosis nigricans; hereditary,
benign AN
• hyperinsulinemia is a common denominator

230. Following are signs of internal malignancy except:


A. Tuberous sclerosis
B. Acanthosis nigricans
C. Clubbing
D. Derrnatomyositis
A
.........(PGI - JUNE 1997)

• Tuberous sclerosis is an autosomal dominant disorder,


also known as epiloia. It occurs in 1: 15,000 persons in
general population.
Erythema gyratum repens The characteristic clinical features are known as ‘vogt’s
• “wood-grain” pattern triad’, which consist of:
• wavy, erythematous, urticarial bands with scale - Mental retardation - mild to severe
• slowly migrate - Convulsions
• breast, sto mach, bladder, prosta te, cervix; - Adenoma sebaceum presents on face & also on rest of
occasionally no CA the body.
- The distribution on face is usually of butterfly type.
- It is not a sign of internal malignancy.

• Acanthosis nigricans can be a reflection of an internal


malignancy, most commonly of the
GIT & it appears as velvety hyperpigmentation, primarily in
flexural areas. In the majority of patients, acanthosis
nigricans is associated with obesity, but it may be a
reflection of an endocrinopathy such as acromegaly,
cushing’s syndrome, the stein-leventhal syndrome,
or insulin-resistant diabetes mellitus.
Sweet’s syndrome (acute febrile neutrophilic dermatosis)
• painful red plaques and papules - face, neck, upper
chest, arms, legs
• usually females (4:1); fever, leukocytosis prominent
• Associations, benign: URIs, strep, RA, Crohns, sarcoidosis,
Behcet’s, pregnancy
• Association, malignant: AML, myelo dysplasis,
lymphoma—may follow by months to years
• Rx: prednisone

Clubbing.
It is found in patients with primary & metastatic lung
cancers & mesothelioma.

• Dermatomyositis is associated with malignancies like


Bronchogenic & breast ca.

HELP LINE NO. 9391567707


MINOR TOPICS DERMATOLOGY 77

E. Prophyria cutanea tarda


A and E
.........(PGI - June -2001)

SLE and porphyria cutanea tarda and liver spots are


photosensitive.

231. True about acanthosis nigricans:


A. Most commonly seen in obesity
B. Seen in axilla
C. It signifies internal malignancy
D. It is associated with insulin resistance
E. Seen in old age Morphea present with solitary plaque or multiple plaque
A,B,C and D or linear lesions or small guttate lesions.
.........(PGI - JUNE 2003)

• Acanthosis n igricans is a velvety th ickening and


pigmentation of major flexors, particularly in axilla.

• Calcinosis cutis — Localised or diffused calcification, lesions


like scleroderma and dermatomyositis.
The most common form of acanthosis nigrican is weight
dependent mild acanthosis nigricans (obesity associated).
• It is also a/w - insulin resistance, gastric malignancy,
metastases.
• Two types of acanthosis nigricans :
(i) Juvenile type : It is benign, a/w endocrine, metabolic or
digestive disorder, (ii) Adult type : Poor prognosis, a/w
abdominal malignancy.
• Liver spot or Lentigo senilis occur on senile skin as irregular-
TOPIC 22: DLE shaped, pigmented macule shortly increasing in size and
not formed on exposed parts of body’
232. All are manifestat|ons of SLE except
D. AP view in full expiration
A. Lesions resemblig Chr. DLE
B. Butterfly rash
C. Photosensitivity
D. Constitutional symptom
None
.........(PGI - June -1998)

• In SLE - RASH IS NOT ITCHY OR PAINFUL.


Other photosensitive skin ksions :
Lichen planus, psoriasis, vitiligo, seborrheic dermatitis. Darrier’s.
disease,, solar urticaria, pemphigus, drugs, herpes, food
preservatives, chemical etc.

233. Which of the following are photosensitive diseases:


A. SLE
B. Liver spots
C. Calcinosis cutis
D. Morphea

HELP LINE NO. 9391567707


MINOR TOPICS DERMATOLOGY 78

234. True about drug induced SLE is: • Ulcerovegetative type (most common): These lesions
A. CNS manifestation are common develop from the nodular type and consist of large, usually
B. Renal involvement is common painless, spreading, exuberant ulcers. The ulcers have
C. Antihistone antibodies are found clean, friable bases with distinct, raised, rolled margins.
D. All with antibodies progress to lupus The ulcers are typically beefy red in appearance and bleed
C easily. Autoinoculation is a common feature, resulting in
.........(PGI - June -2000) lesions on adjacent skin.

• All patients with drug indeced SLE have Anti-nuclear anti-


bodies (ANA) and most have antibodies to histones.
• Clinical Feature : Most common is systemic complains and
arthralgia, polyarthritis and pleuropericarditis occur in 25
to 50% cases.

• Other features :
— Anemia, leuk openia, ACL L A, thromb ocytopenia,
cryoglobulins, rheumatoid factor, False positive • Nodular type: Soft, often pruritic, red nodules arise at
VDRL, positive direct coomb’s test etc. can occur. the site of inoculation and eventually ulcerate and present
• features RARE in drug induced SLE are : a bright red granulating surface. (A nodule may be
— CNS and renal involvement mistaken for a lymph node [a pseudobubo].)
— ds DNA and hypocomplementemia • Cicatricial type: Dry ulcers evolve into cicatricial
plaques and may be associated with lymphedema.
235. A girl, aged 19, presents with arthritis and a • Hypertrophic or verrucous type (relatively rare): This
photosensitive rash on the cheek; Likely diagnosis proliferative reaction with formation of large
is: vegetating masses may resemble genital warts.
A. SLE
B. Chloasma Beefy-red penile ulcers.
C. Steven Johnsons syndrome Granuloma inguinale
D. Lymes disease
A
.........(AIPGMEE - 2001)

Rash in SLE is a Malar butterfly Rash which is :


- fixed, erythematous.
- flat or raised present over the cheeks and bridge of nose.
- photosensitive • The regional lymph nodes are not involved in Donovanosis.
Its presence along with symptoms suggestive of systemic Pseudobuboes. Penile donovanosis seen as an ulcerating.
involvement such as arthritis, makes diagnosis clear as SLE.
Nodular lesion with marked tissue destruction. There is a
large, inguinal ulcer indicating soft tissue breakdown in
the area secondary to donovanosis. This is known as a
‘pseudobubo,’

TOPIC 23: DONOVANIOSIS

236. Pseudo Bubo are seen in:


A. Donovanosis
Swelling of the labia minora resulting in elephantiasis (pseudo-
B. Chancroid elephantiasi s) in a patient who pr esented with
C. LGV
longstanding donovanosis.
D. Syphilis
A
.........(AIIMS PGMEE - Dec - 1995), AIPGMEE - 1996

• Granuloma inguinale (GI, or donovanosis) is primarily a


sexually transmitted disease in which characteristic
intracellular inclusions called Donovan bodies may be seen.
– Granuloma inguinale (donovanosis) usually manifests as
genital lesions, which are indolent, progressive,
ulcerative, and granulomatous.

Four varieties of granuloma inguinale (donovanosis) skin lesions


occur.

HELP LINE NO. 9391567707


MINOR TOPICS DERMATOLOGY 79

238. Donovanosis is caused by: Presentation with a solitary lesion in the genital skin that
A. Calymmatobacterium granulomatosis heals with hyperpigmentation. Soon after ingestion
B. T. pertunae of paracetamol, one of the most commonly implicated
C. Chlamydia trachomatis agents for FDE, suggests a diagnosis of Fixed Drug
D. Haemophillus-ducreyi Eruption (FDE).
A
.........(AIPGMEE - 1997)

Donovanosis is synonymous with Granuloma inguinale.


It is caused by bacteria known as calymmobacterium
granulomatis

TOPIC 24: DRUG REACTION

239. Gold poisoning leading to exfoiiative dermatitis is


treated with:
A. Chloroquine
B. Steroid
Fixed drug eruption (FDE) is an adverse cutaneous reaction
C. Antibiotics
to an ingested drug with characteristic clinical features.
D. Antihistaminics
Characteristic features of fixed drug eruptions - Fitzpatrick’s
B
Lesions develop soon after ingestion of the offending agent
.........(AIIMS PGMEE - Dec - 1995)
Occur from 30 men to 8 hours after ingestion of drug (in
previously sensitized individuals)
Exfoliative dermatitis diffuse skin involvement.
Genital skin is the most commonly involved site -
Fitzpatrick’s

• Exfoliative dermatitis close-up view showing


erythema and scaling.

Nevertheless any site may be involved, conjunctive,


oropharynx, perioral or periorbital regions.
Lesions begin as macules, become edematous forming plaques
and may further evolve into vesicles and bullae and then
into erosions
Most commonly lesions are solitary but they may be
multiple.
• Lesions heal with hyperpigmentation.
‘After healing lesions shows dark brown with violet hue
• Topical steroids are the primary category of postinflammatory hyperpigmentation’
medications used to treat exfoliative dermatitis (ED). Usually asymptomatic.
A sedative antihistamine may be a useful adjunct for pruritic May be pruiritic, painful or burning. Painful when eroded
patients, since it helps patients to sleep at night, thus
limiting nocturnal scratching and excoriations. Antimicrobial • Lesions persist if drug is continued and resolve days to
agents often are used if an infection is suspected to be weeks after drug is discontinued. Lesion recur at same
precipitating or complicating exfoliative dermatitis. Other site every time the drug is taken FDE occur repeatedly at
drugs specifically indicated for management of underlying the identical skin site (i.e. fixed) within hours of ingestion.
etiology of exfoliative dermatitis may be necessary. Antiinflammatory agents such as salicylates, NSAID’s
(includingparacetamol), Phenylbutazone and phenacetin
240. A 27-year-old sexually active male develops a are important implicated agents.
vesiculobullous lesion on the glans soon after taking
tablet paracetamol for fever. The lesion healed with 241. Exfoliative dermatitis can be due to all the following
hyperpigmentation. The most likely diagnosis is:
diseases, EXCEPT:
A. Behcet’s syndrome.
A. Drug hypersensitivity
B. Herpes genitalis.
B. Pityriasis rubra
C. Fixed drug eruption.
C. Pityriasis rosea
D. Pemphigus vulgaris
D. Psoriasis
C
C
..........(AIPGMEE - 2005)
.........(AIPGMEE - 2002)

HELP LINE NO. 9391567707


MINOR TOPICS DERMATOLOGY 80

Recessive epidermolysis bullosa


• The slightest abrasion on the skin would tear the epidermis
from the underlying dermis, and this was true of the
mucosa as well. The child had broad areas of dermis that
were exposed, and the child was treated as a burn victim

Exfoliative Dermatitis (Erythroderma) is involvement of


skin with erythema & scaling. It is caused by:
1. Drugs
2. Skin disease - Eczema
- Psoriasis
- Pityriasis rubra pilaris
- Pemphigus foliceaus
• Lamellar ichthyosis.
- Icthyosiform dermatosis
– The abnormal stratum corneum has produced what
3. Malignancies - Lymphoma& leukemia
appears as very thick scale on the skin, and with an
- Hodgkins ds
abnormal barrier layer these patients commonly get
Mycoses fungoides
secondary staphylococcal and yeast infections. Note that
4. Idiopathic
there are other associated ectodermal problems
such as the teeth, hair, and eyelids
TOPIC 25: EPIDERMOLYSIS BULLOSA

242. Genetic predisposition is seen in which disease:


A. Lichen planus
B. Bullous pemphigoid . ,
C. Pemphigus vulgaris
D. Epidermolysis Bullosa
D
.........(PGI - JUNE 1997)

• Lichen planus is an inflammatory disorder of skin of


unknown origin but with a prominent immunopathogenetic
component. It is characterized by an eruption of variable
extent of typical mauve of pink flat topped itchy papules.

• Epidermolysis bullosa is a group of genetically


inherited diseases in which the patient shows an
enhanced tendency to develop blisters on areas
• Lichen planus. The polygonal papules on the dorsum
subjected to a minor trauma. These diseases are thus
of the foot are typical of the chronic form of the
called mechano-bullous diseases.
disorder, and the lesions are commonly itchy. Dystrophic
nail changes are common in lichen planus.
The blisters are usually situated on points such as
the toes, fingers, ankles, knees, wrists & occasionally
on the face as well. Areas of the body having a bone
underlying the skin are the most frequently involved sites.
• Rest of the conditions mentioned here, are auto-immune
diseases & have an immunological background. -

Recessive epidermolysis bullosa


• A baby with recessive epidermolysis bullosa. This patient
links adequate numbers of anchoring fibrils (collagen
type 7) to attach epidermis to the underlying dermis

HELP LINE NO. 9391567707


MINOR TOPICS DERMATOLOGY 81

• Koebner’s phenomenon in lichen planus. Typical lesions Epidermolysis bullosa is a group of genetically determined
may occur in a scratch when the disease is active, as here skin fragility disorders characterized by blistering of the
along the lines of bramble scratches. skin & mucosa following mild trauma.
Genetic linkage analysis shows to be mutations in the
basal keratin pair with aggregation of keratin
filament clumps in basal epidermis.
It is characterized by the formation of blisters that
heal without clinically significant scarring or milia
formation.

Bullous pemphigoid is a subepidermal blistering disease


mainly occuring in the over 60s. It has also an
immunological background. In this condition, there is a
circulating antibody directed to the epidermal
basement membrane zone in 85-90% of patients which
can be detected using the immunofluorescence method.
• Deposition of immunoglobulins (IgG) in the surface
keratinocytes throughout the epidermis in & around
lesions are seen in immunobullous diseases like
pemphigus vulgaris

Antibodies of the IgG type & the complement


component C 3 are also deposited in the subepidermal zone
around the lesions.

244. In an 8 day old child with no history of


consanguinity in the parents, the mother reports
blisters and bleeding off of the skin at the site of
handling and pressure. There was a similar history in
the previous child which proved to be fatal. The
diagnosis is:
A. Bullous pemphigoid
• Pemphigus vulgaris causes intraepidermal blistering
B. Congenital syphilis
disease because of a loosening of desmosomal links
C. Congenital epidermolysis bullosa
between epidermal cells caused by immunological attack.
D. Letterer - Swiwe disease
C
.........(AIIMS PGMEE MAY - 2001)

Presence of family history and peeling of skin at the .site of


handling confirms the diagnosis of conge nital
epidermolysis Bullosa

Epidermnlvsis Bnllosa :
• Genetically inherited disease
• Characterized by blistering on minimal trauma.
• There are several forms of Epidermolysis Bullosa
• Simple Epidermolysis Bullosa -
• Junctional Epidermolysis Bullosa
243. Features of epidermolysis bullosa: • Dystrophic Epidermolysis Bullosa
A. Decreased Adhesion fibrils • T/t —> Avoidance of trauma
B. Present as miliary & scaring
C. IgG deposits in epidermo-dermal junction Pemphigoid — The commo n age of present ation of
A pemphigoid is usually 20-30 yrs.
.........(PGI - JUNE 2006)

HELP LINE NO. 9391567707


MINOR TOPICS DERMATOLOGY 82

Erythema muliforme is typically associated with vesico bullous


lesions .
Erythema muliforme is a short lived, inflammatory reaction
of skin and mucosae, occurring in response to a variety of
antigenic stimuli resulting in scatterred lesions at the
dermoepidermal junction.

Congenital syphilis — it presents with vesicobullous


lesions in a child but there is no family history.

Individual lesions are red to purple maculopapules some of


which may become annular or target like lesions and
may blister (Vesico bullous)
• The face and upper limbs are prferentially involved
• The disorder may be precepitated by- infections (eg
Herpes Simplex-Most common).
- drugs
- UV rays (Sun exposure)
Letter Siewe disease — Many other systemic features such - Ulcerative colitis and crohn’s disease
as hepatomegaly, bone infiltration are prominent
features besides skin lesions

TOPIC 26: ERYTHEMA MULTIFORME

245. ‘Target lesions” are characteristics of:


A. Dermatitis herpetifonnis
B. Sulpha drug reaction
C. Lichen planus
D. Erythema multiforme
D
E. multiforme is associated with:
.........(AIIMS PGMEE - JUNE - 1997), AIPGMEE - 1998
- Malignancy (Carcinoma & Lymphomas)
• Target lesions (Bull’s eye) are characteristic of - Collagen vascular diseases (SLE, desmatomyositis,
erythema multiforme. Periarteritis nodosa)
• Target lesions are seen on hands and feet consist of red - Infections (e.g. herpes, mycoplasma, histoplasmosis,
typhoid, leprosy)
ring with central pale or purple areas.
- Drugs (sulfonamides, penicillin, salicyclates, barbiturates,
hydantoins, antimalarials).
Multiforme may be a sign of UC and CD., both of which are
premalignant conditions.

TOPIC 27: GONORRHEA

248. The syndromic management of urethral discharge


includes treatment of:
Precipitating factors for erythema multiforme are A. Neisseria gonorrhoeae and herpes genitalis
• Herpes Simplex (Most common) B. Chlamydia trachomatis and herpes ganitalis
• Ulcerative colitis & crohn’s disease C. Neisseria gonorrhoeae and Chlamydia trachomatis
• Ultraviolet light D. Syphilis and chancroid
• Sulfonamide C
> Extreme form of this disease is known as Steven .........(AIPGMEE - 2003)
Johnson’s syndrome

246. Regarding Erythema Multiforme, all are true except:


A. No vesicles
B. Target lesions are seen
C. Involves face and neck regions
D. Is a sign of internal malignancy
A
.........(AIPGMEE - 2000)

HELP LINE NO. 9391567707


MINOR TOPICS DERMATOLOGY 83

The new management for complains ofurethral discharge • Kaposis varicelliform Eruption — manifest as either
involves a combined modality of treatment for Neisseria — Eczema herpeticum or
gonorrhoeae and Ch. Trachomatis as most cases are due — Eczema vaccinatum
to co-infection with both pathogens • Eczema herpeticum is caused by herpes simplex hominis
virus (HSV-1) in primary infection
Initial treatment

Treat gonorrhea Plus Treat chlamydial


infection
- Cefotaxime - Azithromycin
- Ceftrioxone - Azithromycin
- Fluoroqunilone
249. Gonococcus resistant structure is
Eczema vaccinatum is caused by vaccinia virus due to
A. Urethra
inadvertent vaccination of small pox with live vaccinia virus.
B. Testis
• Both of them are complication of atopic dermatitis
C. Fallopian Tube
D. Ampulla
B
.........(AIPGMEE - 1996), AIPGMEE - 1998)

TOPIC 28: HSV

251. The most frequent cause of recurrent genital


ulceration in a sexually active male is:
A. Herpes genitalis
B. Aphthous ulcer
C. Syphilis
D. Chancroid
A
.........(AIPGMEE - 2003)

Herpes ulcers are painful, vesicular, multiple and are associated 253. A patient has Bullous Lesion; on Tzank smear shows
with firm tender lymphadenopathy A. langerhans celts are seen
B. Acantholysis
C. leucocytosis
D. Absens of melanin pigment
B
.........(AIPGMEE - 1996)

Acantholysis is loss of cohesion between Epidermal cells


found in patients of Pemphigus Vulgaris
(bullous lesions are characteristic in pemphigus vulgaris)
Herpes simplex is the most common cause of genital ulcer in
developing countries.
Herpes 62%
Chancroid 12-20%
Syphilis 13%
LGV and Granuloma inguinale
252. Eczema herpeticum is caused by: TOPIC 29: MOLLUSCUM CONTAGIOSUM
A. Herpes simplex virus
B. Varicella 254. A3 year old female child develops umblicated
C. CMV nodules over face following a trivial viral infection,
D. HPV the probable diagnosis is:
E. HSV-6 A. Lichen planus
A B. Chicken Pox
.........(PGI - 2000 - Dec) C. Scabies
D. Molluscum contagiosum
D
.........(AIIMS PGMEE JUNE - 1999)

Molluscum Contagiosum
• is caused by Pox virus
• It presents in children as pearly papule with central
umbilication usually on face.

HELP LINE NO. 9391567707


MINOR TOPICS DERMATOLOGY 84

Other commonly occuring urticarias


1. Cold urticaria are of 2 types
• a) Idiopathic Acquired cold urticaria
PPT by exposure to
• cold water
• cold winds
• A.C. room
• Ice cream, cold drinks

b) Familial cold urticaria


• Huge urticaria with fever & leucocytosis on exposure to
Diagnosis — Molluscum bodies on Giemsa or gram stain is cold occurs in families
characteristic.
• T/t — most of the lesions resolves spontaneously the Dermographism
aim is to prevent the spread to others in the community; • In many pts with urticaria and in a few people without,
curettage, cautery, strong salicylic Acids are used for this. firm presure over a track with blunt object such as a key
or the wrong end of a pencil over the skin of the back will
produce first blanching, then redness, then wheal.
• This phenomenon which is an exaggeration of the
normal triple response is known as Dermographism.

255. A child has umblicated nodule on face, the diagnosis


is
A. Seborrhic dermatitis
B. Chicken Pox
C. Tinea Solar Urticaria — Urticaria develops after few minutes on
D. Molluscum contagiosum exposure to sun.
D
.........(AIIMS PGMEE - JUNE 1998), AIIMS PGMEE - DEC 1998

TOPIC 30: URTICARIA

257. A patient gets recurrent urticaria while doing


exercise and on exposure to sunlight. Which of the
following is the most likely cause:
A. Chronic idiopathic urticaria Solar urticaria
B. Universal sermographism • Urticaria developing within 1-30 minutes of exposure to
C. Cholinergic urticaria sunlight
D. Photo dermatitis • They disappear within 24 hours
C • Lesions are usually seen on the ‘V area of the neck and
.........(AIIMS PGMEE NOV - 2000) the arms.
• Solar urticaria may be the presenting sign in-
• The symptomatic profile of the pt. matches with that of
a) Porphyria cutanea tarda
cholinergic urticaria b) Systemic lupus erythematosis
• Cholinergic urticaria c) Erythropoietic protoporphyria
It is used for those condition in which the attacks are
precipitated on exposure to heat from sunlight or • Drug induced urticaria — Drugs causing urticaria -
other sources, physical exertion and emotional Aspirin, opioids
tension, the pt is bllieved to be hypersensitive to
acetylocholine liberated under the influence of any stimulus
which induces sweating.

HELP LINE NO. 9391567707


MINOR TOPICS DERMATOLOGY 85

258. A 22 year old woman developed small itchy wheals Food


after physical exertion, walking in the sun, eating Food additives Drugs Saliva Grass - Pollen
hot spicy food and when she was angry. The most
likely diagnosis is: • Drugs precipitating urticaria
A. Chronic idiopathic utricaria - Penicillin (60%) Sulfonamides (15%) Aspirin (15%)
B. Heat urticaria Chloramphenicol (5%) Phenytoin (5%)
C. Solar urticaria
D. Cholinergic urticaria • Difference b/w Urticaria and Angioedema
D Angioedema is frequently associated with urticaria, but the
.........(AIIMS PGMEE NOV - 2003) two may occur independently. Angioedema is a reaction
similar to urticaria except that it occurs in deeper tissues
Urticaria and is clinically characterized by asymmetric swelling of
• It consists of raised erythematous skin lesions, that are tissues.
markedly pruritic evanescent and are generally worsened
by scratching.
Cholinergic urticaria (Micropapular urticaria)
• Urticaria develops in response to a rise in the core body
temperature within 2-30 minutes. The lesion lasts for 20-
90 minutes
It is associated with cholinergic sympathetic fibres. They are
provoked by generalized heating
Exercise
Emotional stress • Difference b/w Acute and chronic urticaria
Hot food Acute urticaria persists for < 6 weeks Chronic urticaria
lasts for more than 6-8 weeks
Localized Heat urticaria
• Very rare type of urticaria that is seen after application • Pathogenesis of urticaria
of local heat Histamine plays the key role
• Lesions appear within 2-5 minutes of contact and lasts for T/t ofurticaria-
about an hour. In both acute and chronic urticaria antihistamines are the
• They can be provoked by applying a test tube full of basic drugs.
water at 40°C to the forearm for five minutes. Drug mostly used is hydroxyzine.
In acute urticaria with laryngopharyngeal edema - 0.5
to 1 ml o f epinephrine shoul d be inj ected
subcutaneously.

259. A 5 year old male child has multiple hyperprigmented


macules over the trunk. On rubbing the lesion with
the rounded end of a pen, he developed urticarial
wheal, confined to the border of the lesion. The most
likely diagnosis is:
A. Fixed drug eruption
B. Lichen planus
C. Urticaria pigmentosa
D. Urticarial vasculitis
C
.........(AIPGMEE - 2004)
Cold urticaria
• It is of two types:
Idiopathic Secondary
Idiopathic cold urticaria
• Probably transmitted as an autosomal dominant trait.
Symptoms appear soon after birth and may persist life
long
• Urticaria occurs in response to drop in the body ‘ Urticaria pigmentosa is a familial cutaneous disorder
temperatur e precipitated by g eneralized characterised by generalised distribution of red
environmental rather than local cooling. brown macules.
• Symptoms appear within 3 hrs of exposure Each lesion represents a collection of mast cells in the
dermis with hyperpigmentation of overlying epidermis.
Acquired or secondary cold urticaria -
• Urticaria precipitated by cold contact be it cold water,
cold air, cold drinks or eatables.
• Condition is more common in young adults.
Contact urticaria-
• Urticaria at the site of contact
• Commonly found on the hands or around the mouth
• Agents responsible are -

HELP LINE NO. 9391567707


MINOR TOPICS DERMATOLOGY 86

• The most characteristic features is that these lesions 261. In which of the following conditions Parakeratosis
urticate on scratching. most frequently occurs?
A. Actinic keratoses
B. Seborrheic keratoses
C. Molluscum contagiosum
D. Basal cell carcinoma
A
.........(AIPGMEE - 2006)

• Typical actinic keratosis with downward budding or


elongation of rete pegs (arrows 1), atypia of the
basal layer that extends along the rete pegs (arrows
2) acanthos is, marked hyperkera tosis, and
Darier sign parakeratosis (arrows 3).
• One can demonstrate the presence of mast cells by • In addition there is a chronic inflammatory infiltrate at the
rubbing one of the brown patches. Within a few minutes, base of the epidermis
the rubbed area becomes reddened, swollen and itchy.
This is known as Darier sign, and confirms the presence of
mastocytosis.

TOPIC 32: CHLOASMA

Extensive release of histamine from mast cell degranulation 262. A 23 year old lady develops brown macular lesions
may result in - Headache, flushing diarrhea and pruritis. over bridge of Nose and cheeks following exposure
to light. The probable dignosis is:
TOPIC 31: ACTINITIC KERATOSIS A. Chloasma
B. Acne rosacea
260. Actinic keratosis is seen in: C. Systemic lupus erythematosis
A. Basal cell carcinoma D. Photodermatitis
B. Squamous cell carcinoma A
C. Malignant melanoma .........(AIIMS PGMEE JUNE - 1999), AIPGMEE - 2001
D. Epithelial cell carcinoma
B Chloasma shows light brown pigmented rash. Rash in SLE is
.........(AIIMS PGMEE MAY - 2002) erythematous & not light brown pigmented.

• Actinic keratosis
• are pre malignant lesions occuring on the sun exposed
part of the body in persons of fair complexion

It is considered premalignant but only 1:1000 lesions per


year progress to become squamous cell carcinoma” TOPIC 33: DARIER’S DISEASE

264. Dyskeratosis is characteristic feature of:


A. Darier’s ds
B. Pemphigus vulgaris
C. Psoriasis
D. Bowen’s disease
E. Haikey-Hailey ds
A,D and E
.........(PGI - 2000 - Dec)

Dyskeratosis it is abnormal keratinization occurring prematurely


within individual cells or groups of cells below the stratum
• The lesions are pink or slightly hyperpigmented patches,
granulosum
tender and feels like sand paper when finger is drawn
It may be :
over them
— Benign e.g.
Darrier’s disease
Chronic benign pemphigus of Hailey and Hailey
— Malignant as in epithelioma like :
• Bowen’s disease
• Pagets disease
• Squamous cell Ca

HELP LINE NO. 9391567707


MINOR TOPICS DERMATOLOGY 87

• Skin biopsy from Darier’s disease reveals a


characteristic dyskeratosis with suprabasal
acantholysis secondary to breakdown of the
desmosome adhesion junctions. The epidermis
shows hyperproliferative budding beneath the
dyskeratotic plug.

TOPIC 34: EXANTHEMS CHILDHOOD

266. After 3 days of fever patient developed maculo


erythematous rash that lasted for 48 hrs. The most
• Painful erosions in the flexures characterize Hailey- likely diagnosis is:
Hailey disease. The skin is more fragile than in A. Fifth disease
Darier’s disease. B. Rubella
• Flexural skin may become rather thickened and C. Measles
papillomatous, but patients do not develop the D. Roseola infantum
hyperkeratotic papules that are seen in Darier’s D
disease. .........(AIPGMEE - 2002)

265. Kaposi’s varicelliform eruption seen in


A. Darrier disease • Roseola infantum is the sixth of the traditional
B. Varicella exanthems of childhood. The condition is an acute
C. Pityriasis rosea benign disease of childhood characterized by a
D. Atopic dermatitis history of a prodromal febrile illness lasting
E. Mumps approximately 3 days, followed by defervescence
A and D and the appearance of a faint pink maculopapular
.........(PGI - JUNE 2004) rash.

• Kaposi’s varicelliform eruption refers to widespread • Since identification of the etiologic agent human
cutaneous infection with a virus which normally causes herpesvirus type 6 (HHV-6 ), infection has been
localized or mild vesicular eruption, occurring in a patient documented without the characteristic fever or rash. The
with pre-existing skin disease. virus may present as an acute febrile illness associated
with respiratory or gastrointestinal
symptomatology. In one prospective cohort, 93%
of newly acquired infections were symptomatic, with fever,
fussiness, diarrhea, and rash as the most distinguishing
features

Causes are :
- Infections : HSV (MC), Coxsackie A-16,Vaccinia, Small pox
vaccination
- predisposing factors :
Atopic eczema
Darier’s disease, pemphigus foliaceus, benign familial
pemphigus, icthyosis vulgaris, allergic contact
dermatitis, congenital ichyosiform erythoderma, mycosis
fungoides, sezary syndrome & other inflammatory
• Discrete rose-pink m acules/maculopapules
dermatoses.
characteristic of roseola infantum.

HELP LINE NO. 9391567707


MINOR TOPICS DERMATOLOGY 88

TOPIC 35: FORDYCE SPOT

268. Fordyce spots are


A. Ectopic sebaceous glands
B. Ectopic eccrine glands
C. Ectopic apocrine glands
D. Ectopic mucosal glands
A
.........(AIPGMEE - 1995)
267. Morbilliform eruptions is seen in:
A. Scarlet fever
B. Rubella
C. Toxic shock syndrome
D. Measles
E. Mumps
B and D
.........(PGI - 2001 - Dec)
269. Regarding Fordyce spots:
A. Represent internal maliganancy
B. Ectopic sebaceous glands
C. Present in axillae
D. Found in healthy people
E. Are erythematous
B and D
.........(PGI - June -2002)

• Fordyce’s spots are ectopic sebaceous glands, which


have no erythemous halos & are found in the mouth of
healthy individuals. They are usually confused with Koplik’s
spots of measles.
• Fordyce’s disease is a chronic but benign malady affecting
the mucous membrane of lips (vermilion portion), &
• Causes of Morbilliform rash (erythematous macule and sometimes, the buccal mucosa. It is characterized by
papule) : discrete, pin-head sized, superficial light or yellow milium
— Drugs like, maculo-papular lesions without any subjective
— Viral : Measles sensations.
Rubella
Erythema infectiosum It is more common in males than females.,
EBV, Echovirus, coxsackie virus, adenovirus, early HIV The condition usually appears at puberty.
— Bacterial : Typhoid It is a type of sebaceous, naevoid condition resembling
Early secondary syphilis milium.
Early rickettsia No treatment usually necessary.
Early meningococcus
— Acute Graft versus host disease

— Kawasaki’s disease
• In scarlet fever, Toxic shock syndrome, also Kawasaki’s
disease there is scarlatiniform eruptions.

• Fox-fordyce’s disease is a chronic itching papular eruption


of the axillae & pubes. It occurs due to obstruction of
the apocrine glands, an endocrine disorder or a toxic
menifestation.
Treatment is X-ray therapy & OCR

• Fordyce’s spot is not a sign of internal malignancy. The


different skin manifestations of internal malignancy
are :
- acquired ichthyosis
- acanthosis
- dermatomyositis
- erythema-gyratum reprens
- necrolytic migratory erythema
• Mumps — usual presentation is bilateral painful parotitis - bullous pemphigoid
without rash. - skin metastases

HELP LINE NO. 9391567707


MINOR TOPICS DERMATOLOGY 89

TOPIC 36: IMPETIGO

270. False statement about impetigo:


A. Mostly caused by staphylococcus or streptococcus or both
B. It predisposes to glomerulonephritis
C. Produces scar on healing
D. Eryihromycin is drug of choice
E. It is infectious lesion
C and D
.........(PGI - JUNE 2003)

• Impetigo is a common superficial bacterial infection of skin


caused by group A P - hemolytic streptococci or S. aureus. Piedra is a fungal infection of the hair forming superficial
S. aureus causes bullous impetigo. Nonbullous impetigo nodules of the shaft.
most often caused by p – hemolytic streptococci, but S.
aureus can secondarily infect impetiginous lesions. S.
aureus has increased in incidence as cause.

Light microscopy of the typical nodules of white piedra (x200)

White piedra is caused by the yeasts of the henus Trichos


poron (T.mucoides, T.asahii, T. cutaneum, T. asteroides,
and T. inkin) It is more common in temperature and
semitropical regions, but has been reported worldwide.
It may affect not only the sclap, but also the eyebrows,
eyelashes, beard, axillary, groin, and pubic hairs. It was
It occurs most commonly in young children with poor
also described in monkeys and horses.
hygiene and affects moistened or damaged skin ,
often mucocutaneous borders.
Black piedra occurs more frequently in tropical countries in
South America, Africa and Asia, affects mainly the hair of
• Lesions : Honey coloured crusted papules, plaques or
the scalp, but also beard and moustache, it has also been
bullae, pustules. Despite blistering, redness, underlying
described in primates. Among some Indians in the Amazon
skin heals without scaring.
region it is considered a sign of beauty. The causative
agent is the fungus Piedraia hortai.

Light microscopy with the dark nodules of black piedra (x70)


• Multiple viable bacteria are present in lesions, spreading
by close contact. Shaving or cutting the affected hairs, if possible. Piedra hortae
• Lesions are not painful, no fever. is sensitive to terbinafine and this was successfully when
• Glomerulonephritis can be a sequalae of it but not used orally.
Rheumatic fever.
- Impetigo herpetiformis is the synonym of generalized
Treatment : pustular psoriasis of pregnancy
Topical Mupirocin or sodium fusidate is effective.
• Orally - Dicloxacillin or cephalexin for both the organism.

271. Which of the following arc bacterial infections of


skin:
A. Pyoderma gangrenosum
B. Piedra
C. Impetigo contagiosum
D. Impetigo herpetiformis TOPIC 37: MELANOMA
E. Ecthyma
C and E 272. Melanocytes are present In:
.........(PGI - JUNE 2005) A. Stratum corneum
B. Stratum basale
- Pyoderma gangrenosum is a rare, serious ulcerative C. Stratum granulosum
disorder that is often due to serious underlying systemic D. Derm
diseases like ulcerative colitis, crohn’s disease, Rheumatoid B
atthritis or Myeloma. .........(PGI - 1999 - Dec)

HELP LINE NO. 9391567707


MINOR TOPICS DERMATOLOGY 90

• Melanocytes are present in the basal layers of epiderm is


(stratum basale)
Langerhans cell found in epidermis
Keratinocytes make up 90% of epidermal cells produced
by division cells of basal layer.

273. A child presents with a solitary white well defined


patch on his right thigh. What is the diagnosis:
A. Piebaldism
B. Acral vitiligo
C. Nevus achromicus
D. Albinism
C
.........(AIIMS PGMEE NOV - 2000) Vitiligo
• Hypopigmented lesion usually present at birth
• Presence of single hypopigmented area on the thigh • Symmetrical lesions
suggests nevus Achromicus. • It does not remain static,
Nevus Achromicus It increases in size.
• Single hypopigmented lesion
• Strictly unilateral
• Present at birth but does not increase in size
• May be mistaken for nevus Anaemicus but does not lack
the erythema response.

Albinism
• Diffuse hypopigmentation occurs.
Nevus Anaemicus
• Unilateral hypopigmented lesion present at birth
• does not increase in size
• Almost simulate to nevus Achromicus but the
difference is whenever the area of lesion is massaged
with hand, the lesion fails to develop erythema
which appears in adjoining area.
• Rare in India

• Although all the diseases given in the question cause


hypopigmentation, only nevus Achromicus is strictly
unilateral. All the other diseases either produce symmetrical
areas of Hypopigmentation or they produce diffuse lesions.

TOPIC 38: NEVUS

274. Hypopigmented patches seen in


Nevus anemicus. The edges can be obliterated with pressure, A. Naevus depigmentosa
verifying that this white lesion is caused by decreased B. Freckles
blood content, not decreased pigment C. Naevus Ito
D. Naevus Ota
E. Naevus Anemicus
A
.........(PGI - DEC 2005), PGI - JUNE 2004

• Nevus Anaemicus

Piebaldism
• White forelock with areas of hypopigmentation
• Symmetric involvement of central forehead, ventral
trunk and mid regions of upper & lower extremities.

HELP LINE NO. 9391567707


MINOR TOPICS DERMATOLOGY 91

TOPIC 39: PEMPHIGOID • Early blistering in a patient with a background eczematous


component due to bullous pemphigoid.
276. A 85 yr old woman with Nikolsky sign-ve, blisters
on thigh & trunk, lesions come on & off. What is the
cause:
A. Pemphigus vulgaris;
B. Pemphigoid
C. Lichen planus
D. Dermatitis herpetiformis
B
.........(PGI - June -2000)

• Bullous pemphigoid
• Etiology and pathogenesis • Like the eczematous lesions, blisters may occur in annular
• This is the most common of the immunobullous disorders, arrangements in pemphigoid, as shown here on the thighs
in which the targets for immunological attack are 240kDa
and 180kDa hemidesmosome proteins known as the
bullous pemphigoid antigen.
• These are derived from keratinocytes, and are mainly
found within the basal pole of the basal cells, with a small
amount extracellularly in the lamina lucida. Binding of
antibodies, activ ation of compl ement, and
chemoattraction of neutrophils and eosinophils all
contribute to the blistering process.
• Pathologically, blisters form at the dermo-epidermal
junction, with positive DIF at this level
• Cell-poor’ pemphigoid is a variant with minimal inflammatory
• This is primarily a disorder of elderly subjects. Most develop component. In this case there are tense unilocular
a widespread rash with tense, unilocular blisters of blisters but no erythema or eczematous component.
diameter 1–2cm, which contain clear, straw-colored fluid, In some patients there is a mixture of inflamed and non-
or are hemorrhagic , inflamed lesions
• A pompholyx pattern of vesicles occurs on the palms and
soles in 25% of patients.
• A preceding urticated eczematous phase, often with
rather annular lesions, is common and may precede
blistering by several months.
• Oral lesions are uncommon, occurring in about 10–20%
of patients, and usually mild if they do occur.

Bullous pemphigoid. Annular eczematous lesions are a


precursor phase in many patients , but will demonstrate
positive immunofluorescence if the diagnosis is suspected
and appropriate samples are taken
Hemorrhagic blisters are a common feature in pemphigoid,

• Localized bullous pemphigoid. This patient had a crop of


blisters on the foot, with posi tive direct • Pompholyx-like lesions on the sole in a patient with bullous
immunofluorescence for IgG and C3. pemphigoid
• She was treated with topical steroids only and never
developed more widespread disease.

• Nikolsky in Pemphigus but not Pemphigoid

HELP LINE NO. 9391567707


MINOR TOPICS DERMATOLOGY 92

277. A 40 year old male had multiple blisters over the Dermis
trunk and extremities. Direct immunoflurescence
studies showed linear IgG deposits along the • The level of cleavage in blistering disorder
basement membrane. Which of the following is the
most likely diagnosis Blister level Condition
A. Pemphigus vulgaris Subcorneal Bullous Impetigo
B. Pemphigus foliaceous Intraepidermal Pemphigus
C. BullousPemphigoid Subepidermal Pemphigoid
D. Dermatitis herpetiformis Dermatitis
C herpetiformis
.........(AIIMS PGMEE - NOV 2004) Burns
Subcorneal -Bullous Impetigo
Bullous pemphigoid
an immunologic vesicular disease
IgG antibodies directed against the basement membrane
vesicles are in a subepidermal location
Nikolsky’s sign is negative
acantholysis is not present.

• Bullous impetigo is at the mild end of a spectrum of


blistering skin diseases caused by a staphylococcal
exfoliative toxin that, at the other extreme, is
represented by widespread painful blistering and superficial
denudation (the staphylococcal scalded skin syndrome).
• In bullous impetigo, the exfoliative toxins are restricted
to the area of infection, and bacteria can be cultured
from the blister contents
Immunofluorescence reveals
IgG and complement deposited in a linear fashion • In bullous impetigo the subcorneal bulla contains a
along the basal plasma membrane of basal few acantholytic cells together with neutrophils and
keratinocytes. some Gram-positive cocci.
The antigen is a normal component of the hemidesmosome.

Pemphigus vulgaris affecting the lip

Usually clear, tense bullae on an erythematous base

• Pemphigus affecting the tongue

Generalised Bullous pemphigoid

HELP LINE NO. 9391567707


MINOR TOPICS DERMATOLOGY 93

Pemphigus affects other mucous membranes, in this case Pruritic, papulovesicular lesion symmetrically distributed on
causing a ‘red eye’. scalp, buttocks, extremities, with granular deposition of
IgG at epidermal-dermal junctional

Dermatitis herpetiformis
• Symmetric vesiculation, crusts and erosions are
distributed over the extensor areas of the elbows, knees,
buttocks, shoulders and scalp, with a tendency to grouping
of individual lesions.

Blisters in pemphigus are fragile, as they are due to separation


between epidermal cells. Often, the presenting feature
is erosions rather than clinically apparent blisters.

Dermatitis herpetiformis
• The scalp is relatively commonly involved in pemphigus
vulgaris, erosions at this site often developing prominent IgA granular basement membrane zone with stippling
crusting in dermal papillae

Bullous pemphigoid. Tense, fluid-filled, and hemorrhagic bullae


on an erythematous base were seen on the trunk and
extremities. Some of the bullae have ruptured and left
a scab with crusting
autoimmune disease with intraepidermal acantholysis and
blister formation.

Direct Immunofluorescence of Pemphigoid Skin Biopsy


Pemphigus - intraepidermal
Indirect Immunofluorescence of Pemphigus Serum in Calcium IgG linear basement membrane zone (20x) and C3 linear
Buffer on Intact Human Skin basement membrane zone
IgG cell surface (intercellular substance) staining

Summary
Subepidermal Pemphigoid
• Skin antibodies: Two types (intercellular and basement
Dermatitis herpetiformis
membrane).
Burns

HELP LINE NO. 9391567707


MINOR TOPICS DERMATOLOGY 94

Intercellular cement (pemphigus)


Antigen: 130 kDa desmoglein 3 Pemphigus Pemphigoid
Disease: Antibody found in over 90% of sera from active •Mucosa involved •Mucosa not involved
form of Pemphigus vulgaris

Basement membrane (pemphigoid):

Antigen: 180 kDa transmembrane protein (BP180) Pemphigus Pemphigoid


Disease: Antibody occurs in about 70% of patients
with blistering or bullous pemphigoid •Acantholysis present •Acantholysis absent

Bullous pemphigoid

• Pemphigoid is commonly seen in old age (60-80 yrs) Pemphigus Pemphigoid


as bullous eruptions on lower limb.
• Both pemphigous and pemphigoid give bullous lesions but •Involves upper part of body Involved lower part of body
they differ in other characteristic.

Pemphigus Pemphigoid
•Nikolsky sign present • Nikolsky sign absent
•Bullae are intrepidermal & •Bullae are supepidermal &
flaccid tense
•Mucosa involved •Mucosa not involved
•Acantholysis present •Acantholysis absent
•Involves upper part of body •Involved lower part of body
•Age of involvement 40-60 yrs •Age of involvement 60-80 yrs
•Prognosis poor •Prognosis good TOPIC 40: PORPHYRIA

278. Porphyrins are synthesized mainly in


Pemphigus Pemphigoid A. Liver & Spleen
•Nikolsky sign present • Nikolsky sign absent B. Bone marrow and speen
C. Spleen
D. Liver & Bone marrow
D
.........(AIIMS PGMEE - Dec - 1995)

• Porphyrin is synthesized in liver and Bone Marrow.

279. A 40 year old farmer with a history of recurrent


attacks of porphyria complains of itching when
Pemphigus Pemphigoid exposes to the sun and maculopapular rash on sun
•Bullae are intrepidermal & • Bullae are supepidermal & exposed areas, his symptoms are exaggerated in the
flaccid tense summer. The diagnosis is:
A. Seborrheic dermatitis
B. Contact dermatitis
C. Psoriasis
D. Porphyria cutanea tarda
D
.........(AIIMS PGMEE MAY - 2001)

Sun induced maculopapular lesions, h/o recurrent attacks


of porphyria are classical features of porphyria cutanea
tarda.

HELP LINE NO. 9391567707


MINOR TOPICS DERMATOLOGY 95

Porphyria Cutanea Tarda Contact dermatitis — The lesions are usually


• Commonest Porphyria erythematous vesicles and bullae and there will h/
• Sun induced subepidermal blisters on face and hands. O exposure to allergens.
• Mostly seen in alcoholics with liver damage
• Estrogens (steroids) also aggravate it
• T/t - Venesection or chloroquine

Porphyria Cutanea Tarda

TOPIC 41: PURPURA

280. Palpable purpura Is seen In A/E:


A. Wegeners GN
B. ITP
C. HSP
D. Serum sickness
B
.........(PGI - 1999 - Dec), AIPGMEE - 2000)

• In ITP, there is NON- palpable purpura, found.


• HSP, Wegener’s granulomatosis, Serum sickness - can
cause palpable purpura due to associated vasculitis
Porphyria
• In acute intermittent porphyria skin lesions are absent Note :- Causes of palpable purpura are :
Other cutaneous porphyrias are : 1. Vasculitic : Leukoclastic vasculitis, PAN
• Erythropoietic porphyria 2. Embolic :
• Variegate porphyria — Acute meningococcemia
— Disseminated Gonococcal infection
Seborrhic Dermatitis present as greasy scales and has no — Rocky mountain spotted fever
relation with sunlight. — Ecthyma gangrenosum.

Psoriasis presents as erythematous plaques topped by


silvery scales, Exposure to sunlight can aggravate TOPIC 42: REITER SYNDROME
psoriasis although in majority it has beneficial effect.
282. A 27-year-old male had burning micturition and
urethral discharge. After 4 weeks he developed joint
pains involving both the knees and ankles, redness
of the eye and skin lesion. The most probable clinical
diagnosis is:
A. Psoriasis vulgaris
B. Reiter’s syndrome
C. Behcet’s syndrome
D. Sarcoidosis
B
.........(AIIMS PGMEE - MAY 2005)

• Occurence of Arthritis and conjunctivitis following urinary


tract infection suggests the diagnosis of Reiter’s syndrome.

HELP LINE NO. 9391567707


MINOR TOPICS DERMATOLOGY 96

• Reiter’s syndrome refers to the triad of


Reactive arthritis
Conjunctivitis
Urethritis

Features of Reactive arthritis


• Asymmetric, oligoarticular inflammatory arthritis of lower
extremities.
Arthritis most commonly affects the knees and the ankles.
cultures of synovium and synovial fluid are sterile therefore
the term reactive arthritis. • Laboratory diagnosis -Mild anemia Modest leucocytosis
• Enthesitis (plantar fascia inserts onto the inferior aspect Moderate to marked thrombocytosis - Elevated E.S.R.
of calcaneus, this produces heel pain) Also know
• Dactylitis (usually in the form of sausage toes).
• Associated with antecedent genitourinary and gastro- Clinical feature Definition
intestinal infections. Genitourinary infections with Recurrent oral ulceration Minor aphthous, major aphthous or
chlamydia trachoatis and gastrointestinal infections with herpetiform ulcerations abserved by
shigella, salmonella and yersinia species is common. physical or patient that recurred at
• Axial skeleton involvement characterized by unilateral least three times over a 12 month
period.
sacroilitis. This is seen in 20% cases.
Plus two of the following criterias
Extraarticular involvement which includes - Recurrent genital ulceration Aphthous ulceration or scarring
Conjunctivitis observed by patient or physician.
Anterior uveitis Occular lesions Anterior uveitis, posterior uveitis or
Oral ulcer cells in vitreous on slit lamp
Circinate balanitis examination, or retinal vasculitis
observed by ophthalmologist
Skin lesions

283. Keratoderma blenorrhagicum is seen in:


A. Psoriasis
B. Reiter’s syndrome
C. Syphilis
D. Disseminated gonococcal infection
B
.........(AIIMS PGMEE - MAY 2008)

• Keratoderma blenorrhagicum is a feature of Reiter’s disease


Keratoderima blenorrhagicum is the skin lesion that
develops in Reiters syndrome. It begins as dull red papule
or raised sore that quickly forms a crusty yellow surface.
Keratoderma blenarrhogicum (papulosquamous rash of soles These lesions often cluster together forming scaly plaques
and palms) and pustules.

Keratoderma blenorrhagicum freqeuently affects


- Soles of the feet
- Skin on the top of the feet
- Legs
- Bands, fingers, nails and scalps

HELP LINE NO. 9391567707


MINOR TOPICS DERMATOLOGY 97

TOPIC 43: SPOROTRICHOSIS

284. Linear lesion is seen in:


A. Sporotrichosis
B. Lichen planus
C. Psoriasis
D. Pemphigus
A
.........(PGI - 1997 - Dec)

Sq. cell epithelioma may arise as such or may complicate


dermatosis like keratosis horn, leukoplakia, chronic ulcer,
lupus vulgaris etc.
• But lichen planus, DLE & dyskeratosis congenita can
cause oral cancers, but is still controversial.

287. Predisposing factors for skin ca are


A. Lichen planus
B. Bowen’s disease
Sporotrichosis is caused by sporothrix schenckii & is C. Psoriasis
characterized by the development on the skin, in D. Behcet’s disease
subcutaneous tissues & in lymph nodes, or nodules E. UV rays
which soften & break down to form indolent ulcers. A,B and E
Infection acquired through thorn pricks or other minor .........(PGI - June -2002)
injuries. The fungus spreads from the primary site through
lymphatics but seldom extends beyond the regional lymph • Risk factors of melanoma :
nodes. - Sun exposure
- Family H/O melanoma
- Persistently changing mole
- Presence of clinically atypical mole
- Immunosuppression

285. Which of the foUowings arc fungal infection of skin:


A. Sporotrichosis
B. Molluscum contagiosum
C. Madura foot
D. Tinea • Risk factors for basal cell & squamous cell carcinoma
E. Erysipelas :
A,C and D - Exposure to UV light principally UV-B
..........(PGI - JUNE 2005) - Male sex
- Older age
TOPIC 44: SQUAMOUS CELL CARCINOMA
- Celtic descent
286. Changes of squamous cell carcinoma are seen in: - Fair complexion
A. Seborrhoeic keratosis - Outdoor occupation & a tendency of easy sun burn
B. Bowen’s disease - Chronic arsenic exposure (arsenical keratosis)
C. Lichen planus - Exposure to Cyclic aromatic hydrocarbons in tar, soot or
D. DLE shale.
B - Cigarette smoking (risk of lip or oral ca)
.........(PGI - JUNE 1997) - Human Papilloma viral infection-acts as co-carcinogens.
- Immunosuppresion.
• Bowen’s disease is a rare slowly growing tumour in the
epidermis. - HIV infection.
Tumour cells grow sidewards & upwards remaining within - Ionizing radiations, thermal burns
the epidermis for years; the tumour may later develop - Certain scars & chronic ulcerations
into a basal or squamous cell epitheliomata. - Heritable conditions like albinism, xeroderma pigmentosa
Hence it is usually considered as a pre-cancerous - Mutations in tumor suppressor gene, PATCHED, may lead
dermatosis to BCC.

HELP LINE NO. 9391567707


MINOR TOPICS DERMATOLOGY 98

- Premalignant conditions like Actinic keratosis, Bowen’s • Erysipelas is a c utaneous aff ection, cau sed by
disease, Erythroplasia of Queyrat, Radiodermatitis, streptococcus.
porokeratosis (of Mibelli), Sebaceous epidermal naevus (of - it is an acute inflammation of Lymphatics of skin
Jadassohn organoid naevus), Lichen planus. etc. - Milian’s ear sign is positive
Gorlin PTCH, Transmembrane receptor 9q22.3 Basal cell
syndrome protein= for sonic hedgehog (shh), skin
patched involved in early carcinoma
development through
repression of action of
smoothened
TOPIC 45: TSS

288. Staphylococcus causes A/E:


A. Scarlet fever
B. TSS • MILIAN’S EAR SIGN - Erysi pelas can sp read to
C. Carbuncle pinna(cuticular affection), where as cellulitis cannot.
D. Sycosis barbae
A Impetigo is a common superficial bacterial infection of
.........(PGI - 1999 - Dec) skin caused by - group A Beta hemolytic Streptococci, or
Staphylococcus
• Scarlet fever is caused by streptococcus Lancefield • Clinical disease is marked by isolated pustules which become
group -A. encrested and rupture.

Ecthyma is a variant of impetigo that generally occurs on


the lower extremity and causes punched out ulcerative
lesions. Etiology is similar.

Sycosis barbae is the chronic foliculitis due to staphylococci in


beard region.

• Scalded Skin Syndrome is an exfoliative skin disease in


which the outer layer of epidermis gets separated from
the underlying tissues. Severe form of staphylococcal scaled
skin syndrome (SSSS) is known as ‘Ritter’s disease’ in
new bom and Toxic Epidermal Necrolysis n older
patients.

TSS, carbuncle are caused by staphylococci.

• SSSS is caused by Exfoliative toxin (ET) or exfoliatin


elaborated by staphylococcus.

289. All of the following diseases may be caused by


staphylococcus except:
A. Impetigo
B. Erysipelas
C. Ecthyma
D. Scalded skin syndrome
B
.........(AIPGMEE - 1997)

HELP LINE NO. 9391567707


MINOR TOPICS DERMATOLOGY 99

B. Herpes
TOPIC 46: ACTINOMYCOSIS C. Chicken pox
D. Measles
290. Discharging sinus is seen in A
A. Syphilis .........(AIIMS PGMEE - FEB - 1997)
B. Herpes
C. Actinomycosis • “Apthous ulcers appear as crops of between one and many
D. Molluscum Contagiosum ulcers on a cyclical basis.
C The ulcers are upto 0.5 cm across, round or oval in shape,
.........(AIPGMEE - 1996) with a yellow base and a red erythematous margin.
They are distinctly painful, occur in the unkeratinised
Actinomycosis is clinically characterized by induration mucosa of the cheek, lips, soft palate and floor of
& sinus formation the mouth and normally heal with in 10 to 14 days .
• Actinomyces isreli is causative agent The frequenc y of the ulcer ative epi sodes v aries
• Facio Cervical type is commenest type considerably. They occur more frequently in women than
Lower iaw is the most common site to be involved. men and may then coincide with the second half of the
• Penicillin G is the drug of choice. menstrual cycle.”

TOPIC 49: BALANOPHTHITIS

TOPIC 47: AMYLOIDOSIS 293. Recurrent Balanopthitis seen in:


A. DM
291. “Pinch” purpura is diagnostic of: B. Herpes simplex
A. Systemic primary amyloidosis C. Smoking
B. Secondary systemic amyloidosis D. Alcohol
C. Idiopathic thrombocytopenic purpura E. Bad hygiene
D. Drug induced purpura A
A .........(PGI - DEC 2002)
.........(AIIMS PGMEE - MAY 2005)

• In primary systemic amyloidosis amyloid deposition occur


in the wall of the blood vessels, this leads to an increase
in skin fragility. As a result, petechiae andpurpura develop
in clinically normal skin as well as in lesional skin following
minor trauma. Therefore the skin lesions in case of primary
systemic amyloidosis are called pinch purpura. These
lesions are pink in colour and translucent.
TOPIC 50: BURNS
• Common locations are
- face (especially the periorbital and perioral regions) 294. Which layer of skin causes vesicular changes in case
- flexural areas. of burn:
A. Basal layer
B. Papillary layer
C. Epidermis
D. Dermis
D
.........(AIIMS PGMEE - JUNE - 1997)

TOPIC 51: CANDIDA

295. Treatment of both partners Is recomended In A/E:


A. Candida infection
B. Gardenella
TOPIC 48: APTHOUS ULCER
C. Herpes
D. Trichomonas vaginalis
292. Recurrent oral ulcers with pain and erythematous
B
halo around them, the diagnosis is:
.........(PGI - 1999 - Dec)
A. Aphthous ulcer

HELP LINE NO. 9391567707


MINOR TOPICS DERMATOLOGY 100
— Treatment of sexual partner’s is very important for treating Characteristic dermatological feature and the absence of
the sexually transmitted disease. systemic features sugges t the diagno sis of
— For vulvovaginal candidiasis treatment is given to the sexual dermatomyositis.
partner if candidial dermatitis of penis is detected. Harrison states
— In case of Trichomonal vaginitis examination for STD; “The cutaneous signs of dermatomyositis may preceede
treatment with metronidazol, 2 gm orally single dose given or follow the development oftnyositis by weeks to
to the sexual partner. years.”
— In case of Bacterial vaginosis (Gardenerella), Sometimes a variant of dermatomyositis occur which is
examination for S.T.D. if normal, no treatment is required char acterized by absence of muscle involvement:
to sexual partner. Treatment of a male partner with (dermatomyositis sine myositis) The dermatological
metronidazole does not prevent recurrence of bacterial features of derma tomyositis are “Purpl e red
vaginosis, even though new sexual partners have been discolouration of eyelids, sometimes associated with
implicated as risk factors for recurence. scaling {heliotrope erythema)
— In Herpes infection partner should also be treated.
Condom use can prevent transmission of infection of the • Gottron’s patches -1/3"1 of patients with dermatomyositis
partner. have violaceous, flat topped papules over the distal
interphalangeal joint
TOPIC 52: CHICKEN POX • These lesions are pathognomic of dermatomyositis.

296. In Tzank smear multinucleated cells are seen in:


A. Chicken pox
B. Psoriasis
C. Molluscum contagiosum
D. Pemphigus vulgaris
A
.........(PGI - 1997 - Dec)

• Chicken pox or varicella which is caused by Human Herpes


virus (HHV-3) type 3 is diagnosed by demonstrating
multinucleated giant cells & type A intranuclear inclusion
bodies in smears prepared by scraping the base of the Erythema and scaling involving dorsal part of the
early vesicles (Tzank smear) & stained with toluidine fingres can occur in SLE too . The important feature
blue, Giemsa or papanicolou stain. to distinguish b/w these two is that SLE spares the
• In pemphigus, the pemphigus cell is a separate, detached interphalangeal joint.
& rounded cell with a large, pyknotic nucleus & a deeply Periungual telengiectasia.
staining cytoplasm which is pushed to the periphery.
Poikiloderma - Areas of hypo pigmentation,
hyperpigmentation, mild atrophy and telangiectasia.
Usually develope in patients with long standing disease.
Poikiloderma is rare in both SLE and scleroderma and
thus can serve as clinical sign that serves to distinguish
dermatomyositis from these two conditions.

TOPIC 53: CONDYLOMA ACCUMINATA

297. Podophyllum resin is indicated in the treatment of:


A. Psoriasis.
B. Pemphigus.
C. Condyloma acuminata.
D. Condylomata lata
C
..........(AIIMS PGMEE MAY - 2003)
TOPIC 55: ERYSIPELOID
TOPIC 54: DERMATOMYOSITIS
299. Erysipeloid is transmitted by:
298. A 40 year old woman presented with a 8 month A. Droplet
history of erythema and swelling of the periobital B. Feco-oral
region and papules and plaques on the dorsolateral C. Mosquito bite
aspect of forearms and knuckles with ragged cuticles. D. Contact with animal
There was no muscle weakness. The most likely D
diagnosis is - .........(PGI - June -1999)
A. Systemic lupus erythematosus
B. Dermatomyositis • Erysipeloid is transmitted by contact with animal.
C. Systemic sclerosis • It iscasuedby
D. Mixed connective tissue disorder — Erysipelothrix rhusiopathiae and
B — Bacillus erysipiletus sui
.........(AIIMS PGMEE - NOV 2004)

HELP LINE NO. 9391567707


MINOR TOPICS DERMATOLOGY 101
TOPIC 58: KWASHIORKAR SKIN LEISION

302. Flaky paint appearance of skin is seen in


A. Dermatitis
B. Pellagra
C. Marasmus
D. Kwashiorkar
D
.........(AIPGMEE - 1995)

Flaky paint dermatosis represents skin changes in


Kwashiorkar. Large areas of skin show erythema followed
by hyperpigmentation, Hyperpigmented patches may
TOPIC 56: HERPES ZOSTER
desquammate to expose raw hypopigmented skin .
It gives appearance of old paint flaking off the surface of
300. A 45 year old male has multiple grouped vesicular
the wood — Flaky paint dermatosis.
lesions present on the T10 segment dermatome
associated with pain The most likely diagnosis is -
A. Herpes zoster
B. Dermatitis herpetiformis
C. Herpes simplex
D. Scabies
A
.........(AIIMS PGMEE NOV - 2002)

TOPIC 59: LEISHMANIASIS

303. A, 20 yrs old, male pt,from Jaipur, presents with


an erythematous lesion on the cheek with central
crusting;likely diagnosis is:
A. SLE
B. Lupus vulgaris
C. Chilblains
D. Cutaneous leishmaniasis
D
TOPIC 57: HISTOLOGY OF SKIN .........(AIPGMEE - 2001)

301. Which layer of epidermis is underdeveloped in the The first clinical manifestation is usually a papule at the site of
VLB W infants in the initial 7 days? sandfly bite.
A. Stratum germinativum. Most skin lesions evolve from papular to nodular to ulcerative
B. Stratum granulosum lesions with a char acteristic ce ntr al depression
C. Stratum lucidum surrounded by raised indurated border. Q
D. Stratum corneum Lesions due to Leishmania tropica are characteristically dry &
D have a prominent central crusting
.........(AIIMS PGMEE NOV - 2002) Also in favour is the fact that dermal Leishmaniasis is
endemic in North West Rajasthan.
‘ “Stratum corneum is permeable in preterm infants and
becomes similar to the adults and in infants after two to
three weeks of post natal maturation”

TOPIC 60: MACULE

304. Neither raised nor depressed is:


A. Macule
B. Plaque
C. Nodule
D. Papule
A
.........(PGI - JUNE 1997)

HELP LINE NO. 9391567707


MINOR TOPICS DERMATOLOGY 102
• A macule is a small sized, not raised, circumscribed lesion
with alteration in colour.

Two types of macule:


a) Erythematous & b) pigmentary When of large size, this
alteration in colour in called a patch or a plaque or sheet
of erythema or pigmentation.

• Language of Dermatology

• Primary Lesions A patch is a large spot that is


-macule not palpable &
-patch that is > 1 cm
-papule A patch arises
-plaque de novo or
-nodule through coalescence of macules
-tumor
-vesicle
-bulla
-pustule
-cyst

• Secondary Lesions
-scale
-crust
A papule is a
-excoriation
small superficial bump that is elevated & that is < 1 cm
-lichenification
-maceration
-fissure
-erosion
-ulcer

• Distinct Lesions
-wheal/hive
-burrow
-comedone
-atrophy
-keloid
-fibrosis
-petechiae • Measles –Papular rash
-telangiectasis
-milium

Macule
A flat spot of skin color change
Nevi, roseola
Nevus

Eczema –Papular leision

Roseola
Nodule
A solid elevation of the skin
which extends into the dermis or subcutaneous tissue
larger than 0.5 cm in diameter.
eg
Swollen glands

• Nodular leision

HELP LINE NO. 9391567707


MINOR TOPICS DERMATOLOGY 103
A plaque is a large superficial bump that is elevated & > 1
cm
A plaque arises de novo or through coalescence of papules

A pustule is pus containing bubble often categorized


according to whether or not they are related to hair
follicles:
-follicular - generally indicative of local infection
Plaques in Acne - folliculitis - superficial, generally multiple
- furuncle - deeper form of folliculitis
- carbuncle - deeper, multiple follicles coalescing
-nonfollicular - may indicate systemic infection

Plaque in Chicken pox A cyst is a


sac containing fluid or semisolid material, ie. cell or cell
products

A tumor is a large bump with a significant deep component


& is > 1 cm • Secondary leisions

Scale is the
accumulation or excess shedding of the stratum corneum
Scale
very important in the differential diagnosis
its presence indicates that the epidermis is involved
Scale is typically present
A vesicle is a small fluid-filled bubble that is usually superficial where there is epidermal inflammation
& that is < 0.5 cm ie. psoriasis, tinea, eczema

Blister ,Second degree burn -Vescicular


Scale in Psoriasis

A bulla is a large fluid-filled bubble that is superficial or deep


& that is > 0.5 cm

HELP LINE NO. 9391567707


MINOR TOPICS DERMATOLOGY 104
Crust is dried exudate A fissure is a linear crack in the skin; often very painful
(ie. blood, serum, pus) on the skin surface Athlete’s foot
Seborrhea, impetigo anal fissure

Impetigo

Fissure in Atheletes foot

• Excoriation is a loss of skin due to scratching or picking

An erosion is a superficial open wound with loss of epidermis


or mucosa only

An ulcer is a deep open wound with partial or complete loss


of the dermis or submucosa
Lichenification
an increase in skin lines & creases from chronic rubbing

Decubitus ulcer

Maceration is raw, wet tissue

Stasis ulcer

HELP LINE NO. 9391567707


MINOR TOPICS DERMATOLOGY 105
• Specific leisions Fibrosis or sclerosis describes
dermal scarring/thickening reactions
A wheal or hive describes a short lived (< 24 hours),
edematous, well circumscribed papule or plaque
seen in urticaria

Petechiae or purpura or ecchymosis


describes red blood cells that are outside the vessel walls
& areas are nonblanchable

A burrow
a small threadlike curvilinear papule that is virtually
pathognomonic of scabies

Telangiectasis describes dilated superficial dermal vessels

• Atrophy
– a thinning of the epidermal and/or dermal tissue

• A milium
– a small superficial cyst containing keratin (usually <1-2 mm
in size)

TOPIC 61: MYCOSIS FUNGOIDES

Atrophy with arterial insufficiency 305. Pautrier’s micro-abscess is a histological feature of-
A. Sarcoidosis
B. Tuberculosis
C. Mycosis fungoides
D. Pityriasis Lichenoides Chronica
C
.........(AIIMS PGMEE - NOV 2005)

TOPIC 62: PEDICULOSIS


A keloid
- overgrows the original wound boundaries 306. Treatment of choice of Pediculosis corporis is
- chronic in nature A. 3 applications of Y-Benzene hexachloride
A hypertrophic scar B. One application of Benzyl benzoate
on the other hand does not overgrow the wound C. 4 applications of Benzyl benzoate
boundaries D. Disinfection of clothes only
D
.........(AIIMS PGMEE - NOV - 1993)

• Pediculosis corporis (Body louse) is caused by pediculus


humanus humanus
• It lives in the seams of clothes
Clinical feature –
• The main symptom is severe itching and the lesions consists
of diffusely scattered excoriations and
papules
• T/T - t/t is mainly directed towards disinfecting all the
clothes including the beddings of the patient.

HELP LINE NO. 9391567707


MINOR TOPICS DERMATOLOGY 106
• Methods used for disinfection of clothes TOPIC 66: VON RECHLINGHAUSEN DISEASE
1. Tumble drying - most effective method, it kills both
lice & eggs. 310. Lisch nodule is seen in:
2. High temp, laundering A. Von Recklinghausens disease
3. Drycleaning of outer garments B. Lupus vulgaris.
C. Leprosy
• If large number of patients are affected, as in epidemic, D. LGV
insecticides are used for mass disinfection of clothes. A
.........(PGI - June -1999)
Insecticide most commonly used now-a-days is - Permethrin
because of the increasing resistance to DDT & Lindane.
• Lisch nodule is seen in von Recklinghausen’s disease.
• Pediculosis (louse infestations) is of three types:
Head lice (pediculus capitis humanis)
Body lice (pediculus humanus)
Crab or public lice (pthirus pubis)
• Chronic infestation of body louse results in
postinflammatory hyperpigmentation and thickening of skin
known as Vagabond’s disease
TOPIC 67: WARTS
TOPIC 63: PELLAGRA
311. Skin hazards of swimming are:
307. Casal’s paint necklace is caused by: A. Verrucae
A. Lichen planus B. Pyoderma gangrenosum
B. Pellagra C. M. marinum infection
C. Pernicious anemia . D. M.ulcerans infection
D. SLE D. M.ulcerans infection
B A and C
.........(PGI - JUNE 1997) .........(PGI - June -2001)

• Skin hazards of swimming are :


• Niacin deficiency results in pellagra. Dermatitis, Dementia
- Verrucae (plantar warts)
& Diarrhoea (3D) forms the main clinical presentation of
- Mycobacterium marinum infection-leads to swimming pool
pellagra. Dermatitis occurs in the form of pigmented scaly granuloma.
cracked skin on parts exposed to sunlight, such as neck • Myco. ulcerans cause buruli ulcer.
(casal necklace), back of hands (pellagrous glove), & feet • Adenovirus can cause swimming pool conjunctivitis,
(pellagrous boot). but herpes can’t as it is capsulated and sensitive to
chlorination.
TOPIC 64: PEUTZ JEGHER SYNDROME Swimmer’s ear is otits externa
Swimmer’s itch is caused by S. mansoni and S. jape
308. Oral examination is done in case of:
A. Peutz jegher syndrome TOPIC 68: XERODERMA PIGMENTOSUM
B. Psoriasis
C. Beri-beri 312. Genodermal disease that can cause skin malignancy
D. Plummer vinson syndrome are:
A A. Xeroderma pigmentosa.
B. Neurofibromatosis.
.........(PGI - JUNE 1997)
C. Actinic keratosis.
D. Porphyria cutanea tarda
Peutz-jegher syndrome consists of: A
• Familial intestinal hamartomatous polyposis affecting the ..........(PGI - DEC 2003)
jejunum, where it is a cause of haemorrhage &often
intussusception; • Xeroderma pigmentosa is a rare autosomal recessive
• Melanosis of the oral mucous membrane & the lips. neurocutaneous disorder. In addition to skin lesions,
So, oral examination in helpful in peutz-jegher syndrome. patients may show :
- Progressive mental deterioration, microcephaly, ataxia,
TOPIC 65: VITILIGO peripheral neuropathy, spasticity, choreoathetosis,
Hypogonadism and nerve deafness.
309. Vitiligo all are true except • Inheritable conditions a/w skin cancer :
A. Genetic predisposition present - Albinism
B. Narrow range UV-B is very effective - Xeroderma pigmentosum
C. Topical steroid used for localised lesion - Congenital nevus.
- BCC nevus syndrome
D. Leuckotrichia has good prognosis
• Actinic keratosis is a premalignant skin lesion.
D
• Neurofibromatosis can undergo malignant changes to
.........(AIIMS PGMEE - NOV 2007)
sarcoma & CNS tumours.

HELP LINE NO. 9391567707

You might also like